0% found this document useful (0 votes)
48 views148 pages

Neurology

Uploaded by

Albert Thaw Tun
Copyright
© © All Rights Reserved
We take content rights seriously. If you suspect this is your content, claim it here.
Available Formats
Download as PDF, TXT or read online on Scribd
0% found this document useful (0 votes)
48 views148 pages

Neurology

Uploaded by

Albert Thaw Tun
Copyright
© © All Rights Reserved
We take content rights seriously. If you suspect this is your content, claim it here.
Available Formats
Download as PDF, TXT or read online on Scribd
You are on page 1/ 148

[1]

A 55-year-old builder comes to see you about his ongoing back pain. He was diagnosed several months ago with a
prolapsed intervertebral disc which is still causing him a lot of pain, limiting his ability to work.
He is currently on 75 mg of amitriptyline which has been up titrated over several weeks; he reports he has not noticed much
improvement. Prior to amitriptyline, he was on pregabalin which was stopped due to side effects.
What is the next appropriate step in his management?
🅐 Add duloxetine
🅑 Add gabapentin
🅒 Increase amitriptyline
🅓 Stop amitriptyline and start duloxetine
🅔 Stop amitriptyline and start gabapentin

Option4

[2]
A 20-year-old man presents to the neurology clinic with a 6 month history of deteriorating gait.
On examination he has a wide based gait, with past pointing and high arched feet. Knee and ankler eflexes are absent, but
he has an extensor plantar response bilaterally. Fundoscopy reveals a pale optic disc. There is no impairment of cognition.
🅐 Wilson's disease
🅑 Friedrich's ataxia
🅒 Charcot-Marie-Tooth disease
🅓 Motor neuron disease
🅔 Bardet-Biedl syndrome

Option2

[3]
A 50-year-old man presents with a 2-month history of a progressive headache that is worse in the morning, nausea and
reduced appetite. He adds that he has also been bumping into hanging objects more often.
On examination of his cranial nerves, he is found to have a left superior homonymous quadrantanopia. His visual acuity is
normal.
It is suspected that he might have a space-occupying lesion and an urgent MRI brain has been arranged.
Based on the ophthalmological finding, where is the lesion likely to be located?
🅐 Left parietal lobe
🅑 Left temporal lobe
🅒 Right occipital lobe
🅓 Right parietal lobe
🅔 Right temporal lobe

Option5
[4]
A 27-year-old woman with a background of dystonia of unknown cause has attended the neurology clinic with her mother
for review.
For the past 12 months, she has been having increasingly frequent and worsening muscle rigidity and spasms that have
been difficult to control.
Today she is being considered for insertion of an intrathecal baclofen pump to try and improveher symptoms.
Which receptor does it act on?
🅐 Gama aminobutyric acid (GABA)
🅑 Muscarinic acetylcholine (mACh)
🅒 N-methyl-D-aspertate (NMDA)
🅓 Serotonin-3 (5-HT3)
🅔 amino-3-hydroxy-5-methyl-4-isoxazolepropionic acid (AMPA)

Option1

[5]
A 65-year-old man presents with several months of progressive weight loss, increased difficulty breathing and generalised
fatigue. He is a cigarette smoker with a 50-pack/year history. Over the past few months, he has also noticed increasing
difficulty climbing stairs, although this improves as he continues to climb. Additionally, he mentions a dry mouth, as well as
impotence.
On examination, he appears cachectic. He has pelvic girdle weakness that improves with a short period of exercise. He is
hyporeflexive; again, these improve with exercise.
Which antibodies are most associated with this condition?
🅐 Anti-AChR antibodies
🅑 Anti-Jo-1 antibodies
🅒 Anti-Mi-2 antibodies
🅓 Anti-MuSK antibodies
🅔 Anti-VGCC antibodies

Option5

[6]
Which of the following is least recognized as a cause of autonomic neuropathy?
🅐 Guillain-Barre Syndrome
🅑 New variant CJD
🅒 Diabetes
🅓 Parkinson's
🅔 HIV

Option2
[7]
A 30-year-old lady with no past medical history presents to the emergency department with a 2-day history of pins and
needles in the lower limbs and progressive walking difficulties. She offers a history of a diarrhoeal illness 1 week ago. On
examination, there is a loss of pinprick sensation in the lower limbs from mid-thigh distally and in the upper limbs from MCP
joints distally. There is a bilateral weakness of ankle dorsiflexion to 3/5 and knee flexion and extension is weak to 4/5
bilaterally. Power in upper and lower limbs is otherwise normal. Knee and ankle deep tendon reflexes are absent.
What is the most likely diagnosis?
🅐 Multiple sclerosis
🅑 Guillain-Barre Syndrome
🅒 Functional neurological syndrome
🅓 Chronic inflammatory demyelinating polyneuropathy
🅔 Botulism

Option2

[8]
A patient presenting to the emergency department undergoes a CT head scan. The report describes a hypodense collection
around the convexity of the brain that is not limited to suture lines.
What is the most likely radiological diagnosis?
🅐 Subarachnoid haemorrhage
🅑 Extradural haematoma
🅒 Acute subdural haematoma
🅓 Chronic subdural haematoma
🅔 Intracerebral haematoma

Option4

[9]
A 52-year-old man presents to the emergency department with a sudden, severe headache that started whilst at work 12
hours ago. He has no significant past medical history. He admits to being a frequent user of cocaine.
On examination, he appears to be in significant pain and distress. There are no signs of confusion. Neurological assessment
is unremarkable. An urgent CT scan of the head is requested, which shows no acute abnormalities.
What is the most appropriate next step in investigation?
🅐 CT angiogram of the head
🅑 Lumbar puncture
🅒 MRI scan of the head
🅓 Temporal artery biopsy
🅔 Urine drug screen

Option2

[10]
Which one of the following is most associated with a good prognosis in Guillain-Barre syndrome?
🅐 Age > 40 years
🅑 Female sex
🅒 No history of a diarrheal illness
🅓 High anti-GM1 antibody titre
🅔 Low peak expiratory flow rate

Option3
[11]
A 36-year-old man presents to the emergency department complaining of unsteadiness. He mentions having similar
symptoms several years ago but they resolved without him seeking medical attention.
On examination, he exhibits an ataxic gait. The upper limbs show no abnormalities; however, there is bilateral hypertonia in
the lower limbs, and muscle strength is diminished in all movements, with the flexors predominantly affected. Additionally,
hyperreflexia and clonus are present bilaterally, but sensation remains intact.
What is the most likely diagnosis?
🅐 Friedrich's ataxia
🅑 Multiple sclerosis
🅒 Syringomyelia
🅓 Tabes dorsalis
🅔 Transverse myelitis

Option2

[12]
You are asked to clerk a 72-year-old-man who presented with a fall. He was seen by the stroke team who requested a CT
head. This excluded an intracranial haemorrhage and he was started on aspirin. When you enter the cubicle, you notice the
patient has a right-sided facial droop.
You start taking a history but find it difficult to understand what he says. He is unable to get the words out easily and his
speech is non-fluent as if hesitating before uttering the words.
During the cranial nerve examination, he understood and followed your instructions well. However, he is unable to repeat
words after you.
What type of speech disturbance does this patient have?
🅐 Broca's dysphasia
🅑 Conductive dysphasia
🅒 Dysarthria
🅓 Global aphasia
🅔 Wernicke's dysphasia

Option1

[13]
A 25-year-old male is brought to the emergency department after being struck on the side of the head with a bottle in a
nightclub. According to one of his accompanying friends, he was knocked unconscious initially but then regained
consciousness. An ambulance was called after the patient lost consciousness again. The admission CT head scan shows an
intracranial haemorrhage.
What is the most likely intracranial haemorrhage based on the history?
🅐 Extradural haematoma
🅑 Acute subdural haematoma
🅒 Contusion
🅓 Concussion
🅔 Diffuse axonal injury

Option1

[14]
Which one of the followin gis most likely to be seen following facial nerve paralysis?
🅐 Hyperacusis
🅑 Hyperlacrimation
🅒 Hyperesthesia
🅓 Hyperalgesia
🅔 Hypersalivation

Option1
[15]
A 59-year-old woman presents to the urgent care centre with a 1-day history of facial paralysis. She also complains of some
mild ear pain over the last 2 days. On examination, she has a fixed half-smile on the left side of her face. She is unable to
raise her left eyelid and has increased sensitivity to sound in her left ear. She denies dizziness or vertigo. The remainder of
her cranial nerve examination is normal. ENT examination shows an erythematous left ear canal and vesicles over her soft
palate.
Given the likely diagnosis, which of the following is the most appropriate treatment to initiate?
🅐 Intravenous aciclovir and oral prednisolone
🅑 Intravenous ganciclovir
🅒 Oral aciclovir and prednisolone
🅓 Oral amoxicillin and prednisolone
🅔 Oral prednisolone

Option3

[16]
A 67-year-old man presented to the emergency department with sudden onset double vision on rightward gaze. He had a
background of ischaemic heart disease and hypercholesterolemia. He smoked 10 cigarettes per day.
On examination, he was found to have a normal gait and a normal peripheral neurological examination. Examination of the
cranial the left eye did not adduct on rightward gaze and the right eye exhibited nystagmus. The pupils were equal and
reactive the light.
The emergency department team arrange an urgent MRI of the brain to rule out a possible stroke.
Where is the neurological lesion that would account for the clinical presentation?
🅐 Left medial longitudinal fasciculus
🅑 Left oculomotor nucleus
🅒 Right abducens nucleus
🅓 Right medial longitudinal fasciculus
🅔 Right oculomotor nucleus

Option1

[17]
A 76-year-old man is admitted with a right hemiparesis. He first noticed weakness on his right side around six hours ago. A
CT scan shows an ischaemic stroke and aspirin 300 mg is commenced.
Which one of the following values should not be corrected in acute phase?
🅐 BP 210/110
🅑 Blood glucose 11.2 mmol/l
🅒 Oxygen saturation 93%
🅓 Temp 38.3ºC
🅔 Blood glucose 3.5 mmol/l

Option1
[18]
A 68-year-old man presents to the emergency department following a fall. He says his balance has been terrible recently and
says this is why he fell. He also notes tingling in his fingers and toes. He has a long history of alcohol excess.
On examination, tone and power are normal bilaterally. Knee reflexes appear brisk, and ankle reflexes are both absent.
Plantar reflexes are upgoing bilaterally. His upper limb reflexes arenormal. His pin-prick sensation is intact, but vibration and
proprioception sense are reduced bilaterally in the lower limbs.
What is the most likely diagnosis?
🅐 Alcoholic neuropathy
🅑 Dry beriberi
🅒 Subacute combined degeneration of spinal cord
🅓 Subdural hematoma
🅔 Wernicke's encephalopathy

Option3

[19]
A 40-year-old female presents with progressively increasing headaches associated with tinnitus and occasional visual issues.
The patient reports the headaches are exacerbated by leaning forward and can take several hours lying in bed to resolve.
She has a past medical history of type 2 diabetes and obesity.
Funduscopy reveals swelling of the optic nerves and a raised opening pressure is measured on lumbar puncture.
Which of the following eye defects is most commonly associated with the patient's condition?
🅐 Eye deviation down and out
🅑 Loss of red reflex
🅒 Miosis
🅓 Proptosis
🅔 Relative afferent pupillary defect

Option1

[20]
A 59-year-old woman complains of an urge to constantly move her legs at night due to unpleasant crawling sensations. It is
partially eased when she moves or walks about. This is affecting her sleep and causing her significant distress. She is taking
ferrous fumarate for iron deficiency anaemia and atorvastatin for hypercholesterolaemia.
Given the most likely diagnosis, what should be the treatment?
🅐 Diazepam
🅑 Quinine sulfate
🅒 Ropinirole
🅓 Stopping atorvastatin
🅔 Stopping ferrous fumarate

Option3
[21]
A 28-year-old woman with a history of systemic lupus erythematosus (SLE) presents with jerky, irregular movements which
seem to move from one limb to another. Her symptoms are continuous and there are no other neurological features such as
impairement of consciousness.
Where is the neurological lesion most likely to be?
🅐 Cerebellum
🅑 Temporal lobe
🅒 Hippocampus
🅓 Thalamus
🅔 Caudate nucleus

Option5

[22]
A 34-year-old female with a history of primary generalised epilepsy presents to her GP as she plans to start a family. She
currently takes lamotrigine as monotherapy. What advice should be given regarding the prevention of neural tube defects?
🅐 Folic acid 400 mcg per day once pregnancy has been confirmed
🅑 Folic acid 1 mg per day once pregnancy has been confirmed
🅒 Folic acid 5 mg per day starting now
🅓 Folic acid 10 mg per day starting now
🅔 Folic acid 400 mcg per day starting now

Option3

[23]
A 58-year-old lady presents to clinic with a 6-month history of limb weakness and falls. She has hypercholesterolaemia and
takes simvastatin.
On examination, there is wasting of the intrinsic hand muscles and left biceps with fasciculations in the left biceps and
shoulder girdle. There is a generalised weakness with brisk left biceps and bilateral knee reflexes and a right extensor
plantar response. The sensation is normal throughout. Examination of the cranial nerves was normal.
What is the most likely diagnosis?
🅐 Compressive cervical myelopathy
🅑 Motor neuron disease
🅒 Multifocal motor neuropathy with conduction block
🅓 Myopathy
🅔 Myasthenia gravis

Option2

[24]
A 37-year-old carpenter presents to his GP with a history of clumsiness at work worsening over the last 6 months. He has
noticed 'fumbling' for tools on several occasions and is particularly concerned he has fallen repeatedly due to 'tripping over
his feet'.
He is normally fit and well apart from well controlled type 1 diabetes and has no other symptoms. Insulin is his only
medication. There is no family history of illness.
On examination of this patient's reflexes he has absent ankle jerks and extensor plantars. Sensation is intact.
Which of the following is a possible cause for his symptoms?
🅐 Diabetic neuropathy
🅑 Myasthenia gravis
🅒 Progressive supranuclear palsy
🅓 Motor neuron disease
🅔 Guillain-Barre Syndrome

Option4
[25]
A 19-year-old woman is referred to the neurology outpatient department with 2 years of progressive weakness. She notices
that she finds it difficult to walk upstairs and lift heavy objects over her head. Her mother has commented that her smile
doesn't seem quite as 'bright' as it was previously. She has no significant past medical history and is not on any regular
medications. She does not smoke or drink alcohol.
On examination, she has difficulty closing her eyes and smiling. Power is 4+/5 at the proximal muscles of the upper and
lower limbs. There is no rash.
What is the likely diagnosis?
🅐 Bell's palsy
🅑 Dermatomyositis
🅒 Duchenne muscular dystrophy
🅓 Facioscapulohumeral muscular dystrophy
🅔 Myasthenia gravis

Option4

[26]
A 74-year-old man presents to the emergency department with frank hemoptysis. He is a chronic smoker and states that he
has lost 8 kg of weight over the past two months. He also complains of tingling and numbness of the inner aspect of his right
forearm and little finger.
On examination, there are decreased breath sounds in the upper right lung field. There is wasting of the hypothenar
eminence of his right hand and clawing of the fourth and fifth digits. Observations are within normal limits.
What nerve roots are predominantly responsible for his neurological symptoms?
🅐 C3 and C4
🅑 C4 and C5
🅒 C5 and C6
🅓 C7 and C8
🅔 C8 and T1

Option5

[27]
A 23-year-old man is admitted to the emergency department following a stabbing incident. He has multiple lacerations to
his flank and lower back which have been caused by a knife. After being stabilized he complains of his right leg 'feeling
funny.'
You perform a full neurological examination which reveals the following:
• weakness in extension and flexion of the right knee
• loss of proprioception and vibration below mid thigh in the right leg
• loss of pain sensation below the mid thigh in left leg
Given the likely diagnosis, which of the following best describes the trauma?
🅐 Laceration to right sciatic nerve
🅑 Hemisection of anterior section of spinal cord
🅒 Right-sided lateral hemisection of spinal cord
🅓 Left-sided lateral hemisection of spinal cord
🅔 Posterior disk prolapse

Option3
[28]
A 45-year-old male with a history of alcoholic liver disease presents with increasing confusion. On assessment, he is noted to
be obtunded with a Glasgow coma scale of 14 out of 15, has a wide-based gait and nystagmus. There is no history or signs
of recent trauma.
What is the most specific finding on brain MRI for this patient's condition?
🅐 Left-sided subdural haematoma
🅑 Generalized cerebral atrophy
🅒 Hydrocephalus
🅓 Enhancement of mamillary bodies
🅔 Right-sided cerebellar infarction

Option4

[29]
A 76 year-old man is brought in to the Emergency Department with confusion, a temperature of 39.9 degrees Celsius and
rigidity. From a recent clinic letter you see he lives alone at home and depends on help from his daughter, who has recently
had a prolonged stay in hospital for appendicitis. Upon her discharge she went to see him, found him unwell and called 999.
She had called him and he had been well the day before. He has a background of hypertension and Parkinson's disease for
which he takes ramipril, amlodipine and 'a Parkinson's tablet' which he does not have with him. His chest x-ray and urine dip
are clear and he is unable to give a history. A CT brain shows no acute abnormality.
What is the most appropriate intervention?
🅐 Broad spectrum antibiotics
🅑 Neurosurgical referral
🅒 Bromocriptine
🅓 Intravenous aciclovir
🅔 Cyproheptadine

Option3

[30]
A 21-year-old female is seen in the first seizure clinic in the outpatient department. Both the EEG and MRI brain are normal.
A decision is made not to start her on anti-epileptic medication.
What restrictions on driving should she be informed about?
🅐 No restrictions but inform DVLA
🅑 No restrictions, no need to inform DVLA if not on medication
🅒 Cannot drive for 1 month from date of seizure
🅓 Cannot drive for 6 months from date of seizure
🅔 Cannot drive for 1 year from date of seizure

Option4

[31]
Which of the following visual field changes would be most consistent with a left parietal lobe lesion?
🅐 Right homonymous hemianopia
🅑 Left inferior homonymous quadrantanopia
🅒 Left superior homonymous quadrantanopia
🅓 Right superior homonymous quadrantanopia
🅔 Right inferior homonymous quadrantanopia

Option5
[32]
A 45-year-old woman who had chemotherapy 15 days ago for locally advanced breast cancer presents to the emergency
department with weakness of both legs which has come on over the previous 12 hours. She has been mostly well since
chemotherapy, although does report suffering from painful blisters in a strip across the right side of her chest last week.
On examination, she has a temperature of 37.8ºC and is rather distressed.
Power is 3/5 throughout both legs. There is clonus with upgoing plantars bilaterally, reflexes are brisk. There is loss of fine
touch sensation below the umbilicus.
Examination of the upper limbs and cranial nerves is normal.
Which of the following is the most likely cause of her current presentation?
🅐 Varicella myelitis
🅑 Venous sinus thrombosis
🅒 Guillain-Barre Syndrome
🅓 Vincristine neuropathy
🅔 Cauda equina syndrome

Option1

[33]
A 24-year-old female presents to her general practitioner with three days of tingling in her left arm and double vision. She
had been feeling fatigued for the six months prior to this clinic visit. There no significant past medical history and she is on
no medications. She smokes five cigarettes per day and drinks approximately one bottle of wine per week and works as a
journalist.
Neurological examination revealed reduced sensation in her left upper limb. Upon testing her eye movements, it was noted
that on left lateral gaze, her right eye failed to adduct and her left eye demonstrated nystagmus.
Given the likely diagnosis, where is the anatomical location of the lesion causing the eye signs on examination?
🅐 Mamillary bodies
🅑 Medial longitudinal fasciculus
🅒 Neuromuscular junction
🅓 Optic nerve
🅔 Optic tract

Option2

[34]
A 70-year-old man is investigated for involuntary, jerking movements of his arms. His symptoms seem to resolve when he is
asleep.
Damage to which one of the following structures may lead to hemiballism?
🅐 Subustantia nigra
🅑 Red nucleus
🅒 Subthalamic nucleus
🅓 Globus pallidus
🅔 Frontal lobe

Option3
[35]
A 31-year-old man presents around four weeks after a non-specific viral illness characterised by fever, lethargy and sore
throat. For the past week he has noticed increasing weakness in his legs which has now started to extend to his arms. On
examination he has reduced power, reflexes and slightly reduced sensation in his lower limbs. A few days after admission he
becomes short-of-breath. His forced vital capicity (FVC) starts to fall and he is transferred to ITU.
Given the likely diagnosis, what is the treatment of choice?
🅐 Neostigmine
🅑 Intravenous corticosteroids
🅒 Haemofiltration
🅓 Intravenous immunoglobulin
🅔 Riluzole

Option4

[36]
A 27-year-old man presents to the Emergency Department with 2-day history of severe headache and pyrexia (38.2ºC). A CT
scan is reported as follows: Petechial haemorrhages in the temporal and inferior frontal lobes. No mass effect. Brain
parenchyma otherwise normal.
What is the most likely diagnosis?
🅐 Brain abscess
🅑 Meningococcal meningitis
🅒 Cerebral malaria
🅓 Herpes simplex encephalitis
🅔 New variant CJD

Option4

[37]
A 67-year-old man is diagnosed with an acute ischaemic stroke. On assessment, you identify a right superior visual field
defect. He is aphasic. His speech is fluent and voluminous but makes little sense. His comprehension and production of
written language are also impaired.
What type of aphasia does this represent?
🅐 Broca's aphasia
🅑 Conduction aphasia
🅒 Global aphasia
🅓 Transcortical motor aphasia
🅔 Wernicke's aphasia

Option5

[38]
A 28-year-old man presents with hearing loss which has progressively worsened over the past 6 months. On thing for
examination, air conduction is better compared to bone conduction in both ears, and Weber's test does not lateralise. You
also note a bilateral loss of the corneal reflex.
What is the most likely diagnosis?
🅐 Li-Fraumeni syndrome
🅑 Neurofibromatosis type 1 (NF1)
🅒 Neurofibromatosis type 2 (NF2)
🅓 Tuberous sclerosis
🅔 VonHippel-Lindau syndrome

Option3
[39]
An 84-year-old male presents to the emergency department with a loss of some of his vision and a loss of feeling in his left
arm.
Upon upper and lower limb neurological examination there is nil of note. However, you notice he does not look at you whilst
you are on his left-hand side and talking to him.
Cranial nerve examination reveals a visual field deficit found on direct confrontation, with blind spots in the bottom left
quarter in both eyes.
His capillary blood glucose is 5.6.
His blood results are as follows:
• Hb: 145 g/L (Male: 135-180)
• Platelets: 200 × 10⁹/L (150 - 400)
• WBC: 7.2 × 10⁹/L (4.0 - 11.0)
• Na : 137 mmol/L (135 - 145)
• K : 4.2 mmol/L (3.5 - 5.0)
⁺ 5.1 mmol/L (2.0 - 7.0)
• Urea:

• Creatinine: 90 μmol/L (55 - 120)
• CRP: 3 mg/L (< 5)
Which lobe of the brain has been affected?
🅐 Left frontal
🅑 Left parietal
🅒 Left temporal
🅓 Right parietal
🅔 Right temporal

Option4

[40]
A 40-year-old presents with generalised weakness and visual issues. She reports first noticing some blurring and double
vision 4 days ago. A day later she then developed difficulty getting up from a chair and lifting her arms up which seems to
have spread now to her hands and feet. She reports returning from a hiking tour in Central America 2 weeks ago where she
was treated with antibiotics for an infection.
On exam, she has proximal muscle weakness and an ataxic gait. Her sensation is normal throughout but reflexes are
diminished. Eye weakness with diplopia is also noted.
Given the patient's likely diagnosis, what is the most likely causative pathogen?
🅐 Borrelia burgdoferi
🅑 Campylobacter jejuni
🅒 Cytomegalovirus
🅓 Epstein-Barr virus
🅔 Trypanosoma cruzi

Option2

[41]
Which one of the following conditions is least recognised as a cause of a seventh nerve palsy?
🅐 Acoustic neuroma
🅑 Herpes zoster
🅒 HIV
🅓 Systemic lupus erythematosus
🅔 Diabetes mellitus

Option4
[42]
An 82-year-old man presents with 12 months of progressive loss of sensation in the lower limbs. A viral infection preceded
these symptoms. He has a past medical history of chronic kidney disease stage 5. He lives in an old house that has not
undergone any modernisation.
On examination, there is reduced sensation extending to the knees bilaterally. There is no motorweakness. Blood tests are
performed: HbA1c - 46 mmol/mol (< 48)
What is the most likely cause of the presentation?
🅐 Charcot-Marie-Tooth disease
🅑 Diabetes mellitus
🅒 Guillain-Barre Syndrome
🅓 Lead poisoning
🅔 Uraemic polyneuropathy

Option5

[43]
A 68-year-old woman presents with a two month history of electric shock like pains on the right side of her face. She
describes having around 10-20 episodes a day which, each lasting for around 30-60 seconds. A recent dental check was
normal. Neurological examination is unremarkable.
What is the most suitable first-line management?
🅐 Amitriptyline
🅑 Sodium valproate
🅒 Carbamazepine
🅓 Atenolol
🅔 Zolmitriptan

Option3

[44]
A 35-year-old female is admitted to hospital with hypovolaemic shock. CT abdomen reveals a haemorrhagic lesion in the
right kidney. Following surgery and biopsy this is shown to be an angiomyolipoma.
What is the most likely underlying diagnosis?
🅐 Neurofibromatosis
🅑 Budd-Chiari syndrome
🅒 Hereditary haemorrhagic telangiectasia
🅓 VonHippel-Lindau syndrome
🅔 Tuberous sclerosis

Option5

[45]
A 50-year-old gentleman presents to clinic with a 20-year history of bilateral upper limb tremor. He has no past medical
history. On examination, he has a bilateral symmetrical upper limb postural tremor with no rest or intention tremor. His father
had a similar tremor. He has noticed improvement with alcohol. It has become more severe in the past 2 years and he would
like treatment.
What is the best first line therapy?
🅐 L-dopa
🅑 Deep brain stimulation
🅒 Primidone
🅓 Propranolol
🅔 Botulinum toxin

Option4
[46]
A 50-year-old man presents with right-sided ear pain and facial weakness. He had flu-like symptoms of fever and headache
for three days before the rash appeared.
On examination, there is a right-sided facial nerve palsy. A vesicular rash is seen on otoscopy.
How should this patient be managed?
🅐 Corticosteroids only
🅑 Intravenous aciclovir
🅒 Oral aciclovir and corticosteroids
🅓 Oral aciclovir only
🅔 Reassurance

Option3

[47]
A 40-year-old woman has lost her mobility over the last three years.
On examination of the lower limbs, there is Medical Research Council (MRC) grade 3 power throughout both legs. Tone is
increased and reflexes are brisk. On examination of the upper limbs, there is MRC 5 on the left and 4+ on the right.
Which of the following is most likely to have lead to her leg weakness?
🅐 Multiple sclerosis
🅑 Guillain-Barre Syndrome
🅒 Central pontine myelinolysis
🅓 Charcot-Marie-Tooth disease
🅔 Miller Fisher syndrome

Option1

[48]
A 78-year-old male presents to the emergency department with impairment of his speech. He has a background medical
history of type two diabetes mellitus, hypertension, hypercholesterolemia, and a 50-pack year smoking history.
On examination, his speech is non-fluent. It is halting and effortful. He has difficulty naming examples within a category, he
can only name 4 types of animals in 1 minute. He is unable to repeat the phrases 'baby hippopotamus' and 'British
constitution'. However, he is able to follow 3 stage commands. He has normal tone, power, coordination, and reflexes
throughout.
Neurological imaging reveals an acute localised infarct.
Where is this infarct most likely to be found?
🅐 Angular gyrus
🅑 Arcuate fasciculus
🅒 Inferior frontal gyrus
🅓 Transverse temporal gyri (Heschl's gyri)
🅔 Brodmann area 22 in the superior temporal gyrus

Option3
[49]
A 46-year-old man presents to the emergency department with a painful red eye that has developed over the past few
hours. He describes the vision in the eye as blurred and feels generally nauseous, having vomited twice already. He
describes seeing 'haloes' around lights.
His past medical history includes hypertension, asthma, migraines, and hypercholesterolaemia. He takes ramipril,
amlodipine, atorvastatin, topiramate, montelukast, and inhalers.
What medication is most likely to have precipitated his presentation?
🅐 Amlodipine
🅑 Atorvastatin
🅒 Montelukast
🅓 Rampril
🅔 Topiramate

Option5

[50]
A 24-year-old female presents to her GP due to increased frequency of migraine attacks. She is now having around four
migraines per month.
Which type of medication would it be most appropriate to prescribe to reduce the frequency of migraine attacks?
🅐 Specific 5-HT2 agonist
🅑 5-HT1 antagonist
🅒 Tricyclic antidepressant
🅓 Beta-blocker
🅔 Specific 5-HT1 agonist

Option4

[51]
A 68-year-old man presents to the emergency department with sudden-onset difficulty in speaking and comprehending
language. He also has significant weakness of the right arm. He has a past medical history of diabetes and hypertension.
After examination, a CT head scan shows left-sided cerebral hypoattenuation, indicating ischaemia.
Given the likely diagnosis, he is referred to the appropriate team and spends a considerable amount of time in hospital.
After discharge, the community team follows him up and wishes to measure his level of disability and dependence with
regards to daily living.
Which of the following is best for this purpose?
🅐 Barthel scale
🅑 FAST test
🅒 MUST
🅓 ROSIER scale
🅔 Waterlow score

Option1
[52]
An 83-year-old man is brought to the emergency department with reduced consciousness. His daughter reports that he has
been behaving out of character over the past three weeks, with aggressive outbursts followed by excessive sleepiness. The
daughter also describes an episode of increased confusion 3 days ago which resolved after 24 hours. This morning the
patient was found unresponsive in bed. His past medical history includes atrial fibrillation and recurrent falls. On
examination, he is responding only to pain. There is no obvious focal neurology.
What is the most likely diagnosis?
🅐 Extradural haemorrhage
🅑 Ischaemic stroke
🅒 Space-occupying lesion
🅓 Subarachnoid haemorrhage
🅔 Subdural haemorrhage

Option5

[53]
How long should a patient stop driving for following a stroke?
🅐 No restriction unless physical/visual impairment
🅑 1 month
🅒 3 months
🅓 6 months
🅔 12 months

Option2

[54]
A 65-year-old gentleman is admitted to the medical take with a progressively worsening headache for 3 weeks. On closer
questioning, he reveals it is worse first thing in the morning but also exacerbated by recumbency and coughing. He is also
complaining of intermittent visual disturbances and on fundoscopic examination, there is papilloedema of the right disc but
optic atrophy on the left.
A subsequent CT scan is performed revealing a space occupying lesion.
Where is the spaceoccupying lesion most likely situated?
🅐 Left temporal
🅑 Right frontal
🅒 Right temporal
🅓 Left frontal
🅔 Left parietal

Option4

[55]
Which one of the following infections is most strongly associated with the development of Guillain-Barre syndrome?
🅐 Shigella
🅑 Salmonella
🅒 E. coli H7:O157
🅓 Herpes simplex
🅔 Campylobacter jejuni

Option5
[56]
A 17-year-old man is referred to dermatology. He has around 10 hyperpigmented macules on his torso which vary in size
from 1.5-5 cm in size. His GP also noted some freckles in the groin region. He is also currently under orthopaedic review due
to a worsening scoliosis of the spine. His father suffered from similar problems before having a fatal myocardial infarction
two years ago.
Which chromosome is most likely to have a gene defect?
🅐 Chromosome 4
🅑 Chromosome 11
🅒 Chromosome 16
🅓 Chromosome 17
🅔 Chromosome 22

Option4

[57]
Which one of the following factors indicates a poor prognosis in patients with multiple sclerosis?
🅐 Relapsing-remitting disease
🅑 Presence of sensory symptoms
🅒 Young age of onset
🅓 Male sex
🅔 Long interval between first two relapses

Option4

[58]
A 25-year-old woman presents with recurrent attacks of 'dizziness'. These attacks typically last around 30-60 minutes and
occur every few days or so. During an attack 'the room seems to be spinning' and the patient often feels sick. These
episodes are often accompanied by a 'roaring' sensation in the left ear. Otoscopy is normal but Weber's test localises to the
right ear.
What is the most likely diagnosis?
🅐 Acoustic neuroma
🅑 Vestibular neuritis
🅒 Benign paroxysmal positional vertigo
🅓 Multiple sclerosis
🅔 Meniere's disease

Option5
[59]
A 57-years-old woman presents to the emergency department with a sudden intense episode of dizziness. Her symptoms
began while lying in bed when she turned her head to the side. She experienced two bouts of vomiting accompanied by a
sensation that the world was spinning around her.
She has experienced such episodes in the past lasting around 30-60 seconds before self-resolving. Her past medical history
is significant for migraines and hypertension. She denies tinnitus or any hearing changes.
Her observations are:
• Temperature: 36.9 °C
• Blood pressure: 145/90 mmHg
• Pulse: 95/min.
Which of the following is the most appropriate immediate management for this patient's condition?
🅐 Brandt-Daroff exercise
🅑 Epley manoeuver
🅒 Dix-Hallpike manoeuver
🅓 Refer to ears, nose and throat (ENT) specialist
🅔 Betahistine

Option2

[60]
A 59-year-old man presents with recurrent attacks of vertigo and dizziness. These attacks are often precipitated by a change
in head position and typically last around half a minute. Examination of the cranial nerves and ears is unremarkable. His
blood pressure is 120/78 mmHg sitting and 116/76 mmHg standing.
Given the likely underlying disorder, what is the most appropriate nextstep to help confirm the diagnosis?
🅐 Epley manoeuver
🅑 Tilt table test
🅒 24 hour ECG monitoring
🅓 MRI of th cerebellopontine angle
🅔 Dix-Hallpike manoeuver

Option5

[61]
A 63-year-old woman with motor neuron disease is reviewed in clinic.
Which one of the following interventions will have the greatest effect on survival?
🅐 Regular chest physiotherapy
🅑 Total parental nutrition
🅒 Riluzole
🅓 Antioxidant supplementation
🅔 Non-invasive ventilation

Option5

[62]
A 60-year-old man is diagnosed with Bell's palsy. What is the current evidenced base approach to the management of this
condition?
🅐 Refer for urgent surgical decompression
🅑 Aciclovir
🅒 No treatment
🅓 Aciclovir + prednisolone
🅔 Prednisolone

Option5
[63]
A 30-year-old woman presents with a 2-day history of difficulty with her vision. She is unable to see things to the sides and
feels nauseous and dizziness when trying to look towards the sides. She had an episode of tingling sensation in her left feet
2 months ago. Physical examination show impairment of adduction of both eyes and nystagmus with lateral gaze. An MRI
brain is arranged.
Given the current signs, symptoms and probable underlying diagnosis, where would you expect to find lesions on the MRI
scan?
🅐 Paramedian area of midbrain and pons
🅑 Ventral midbrain at the level of superior colliculus
🅒 Ventral midbrain at the level of inferior colliculus
🅓 Paramedian area of dorsal pons
🅔 Caudal pons at the level of middle cerebellar peduncles

Option1

[64]
A 55-year-old man with motor neurone disease (MND) attends the clinic. He has lost a substantial amount of weight over the
past 3 months. His current BMI is 14.2 kg/m². He has an extremely poor appetite and dysphagia to both solids and liquids.
Pulmonary function tests are as follows:
• Forced vital capacity (sitting) 84% predicted (>80% predicted)
• Forced vital capacity (lying) 80% predicted (>80% predicted)
What is the best form of nutritional support for this patient?
🅐 Nasogastric tube
🅑 Nasojejunal tube
🅒 Percutaneous gastrostomy tube (PEG)
🅓 Radiologicall inserted gastrostomy (RIG)
🅔 Total parental nutrition

Option3

[65]
A 16-year-old girl is seen in the clinic with suspected complex partial seizures following several episodes of impaired
consciousness and automatic features including lip-smacking and chewing. Further investigation confirms the likely
diagnosis, and the child is started on a medication for the condition.
A few days later however the child is rushed into hospital with a several, global rash with blistering and peeling of the skin.
The mucous membranes are also involved and there is evidence of end-organ failure.
What medication was the child most likely started on?
🅐 Lamotrigine
🅑 Levitiracetam
🅒 Sodium valproate
🅓 Topiramate
🅔 Valpromide

Option1
[66]
A 60-year-old woman presents with a tremor.
Which one of the following features would suggest a diagnosis of essential tremor rather than Parkinson's disease?
🅐 Difficulty in initiating movement
🅑 Tremor is worse following alcohol
🅒 Postural instability
🅓 Unilateral symptoms
🅔 Tremor is worse when the arms are outstretched

Option5

[67]
A 14-year-old boy is seen in the clinic for assessment following new-onset seizures. On three separate occasions, the boy has
woken up in the morning, suffered jerking of his right arm, progressing to generalised tonic-clonic seizures. He notes that
before each episode, he has stayed up late the night before. He is otherwise fit and well.
What is the most likely diagnosis?
🅐 Benign rolandic epilepsy
🅑 Hypnogogic myoclonus
🅒 Juvenile myoclonic epilepsy
🅓 Lennox-Gastaut syndrome
🅔 Rett syndrome

Option3

[68]
A 55-year-old woman with a background of metastatic breast cancer presents with a short history of nausea and vomiting.
She has vomited five times today, despite taking regular metoclopramide. She received palliative chemotherapy three days
ago.
You decide to commence treatment with ondansetron.
What is the best description of the mechanism of action of this medication?
🅐 Dopamine antagonist
🅑 H1 (histamine) antagonist
🅒 5-HT1 (serotonin) agonist
🅓 5-HT2 (serotonin) antagonist
🅔 5-HT3 (serotonin) receptor antagonist

Option5

[69]
A 65-year-old man is referred to the neurology outpatient department with 3 weeks of weakness. He smokes 20
cigarettes/day.
On examination, power is 4/5 at hip flexion and knee extension, which improves to 5/5 on repeated testing. Knee reflexes
are absent. Cranial nerve examination is normal.
A chest x-ray demonstrates a coin-shaped lesion in the left mid-zone.
Creatinine kinase 289 U/L (40-320)
EMG testing demonstrates an increment in muscle action potentials after exercise.
What blood test is likely to be positive?
🅐 Anti-Hu antibody
🅑 Anti-Jo1 antibody
🅒 Anti-acetylcholine receptor (AChR) antibody
🅓 Anti-muscle-specific tyrosine kinase antibody
🅔 Voltage-gated Ca² channel antibody

Option5 ⁺
[70]
A 24-year-old man presents with lower limb weakness. He states that he had an episode of food poisoning two weeks ago
which consisted of severe abdominal pain and profuse non-bloody diarrhoea. Following this, he noticed progressive
difficulties with balance.
His past medical history is largely unremarkable, but he admits to intravenous drug use with heroin. He last injected heroin
three weeks ago.
On examination, marked ataxia and ophthalmoplegia are noted.
What is the most likely diagnosis?
🅐 Acute inflammatory demyelinating polyradiculoneuropathy
🅑 Acute motor axonal neuropathy
🅒 Botulism
🅓 Miller-Fisher syndrome
🅔 Tetanus

Option4

[71]
Which one of the following is least associated with the development of chorea?
🅐 Haemochromatosis
🅑 Ataxic telangiectaisa
🅒 Carbon monoxide poisoning
🅓 SLE
🅔 Huntington's disease

Option1

[72]
A 45-year-old man is investigated for spastic leg weakness which has come on over the previous 5 days.
He undergoes a whole spine MRI which shows: Hyperintense T2 signal extending across the spinal cord, between the levels
of T9 and T12.
Which of the following is most likely to cause this clinical and radiological presentation?
🅐 Conus medullaris syndrome
🅑 Human immunodeficiency virus
🅒 Cauda equina syndrome
🅓 Diabetes mellitus
🅔 Brown-Sequard syndrome

Option2

[73]
A 36-year-old man presents to the emergency department with a severe left-sided headache with pain around the left eye.
He has had several similar episodes over the last 2 weeks, lasting 40-60 minutes each. The headaches are associated with a
runny nose. On examination, there is redness and tearing of his left eye.
What is the most appropriate acute management?
🅐 Acetazolamide
🅑 High flow oxygen
🅒 Paracetamol and naproxen
🅓 Urgent CT head
🅔 Verapamil

Option2
[74]
You are asked to see a 21-year-old woman on the medical take who presents with a 2-day history of blurred vision and pain
in her right eye which worsened after her morning bath. She had similar symptoms 6 months ago affecting her right eye but
these resolved spontaneously and she did not seek medical attention.
Examination reveals a central scotoma and impaired colour vision on Ishihara plates in the right eye. Examination is
otherwise unremarkable. A contrast MRI brains hows T2 enhancement of the right optic nerve but no other abnormalities.
Lumbar puncture demonstrates oligoclonal bands in the cerebrospinal fluid (CSF). Her mother has multiple sclerosis (MS)
and she is understandably anxious to discuss her diagnosis and treatment options.
Which of the following is the most appropriate action?
🅐 Start beta-interferon
🅑 Reassure her she does not have MS and discharge
🅒 Give steroids and arrange MRI whole spine
🅓 Perform visual evoked potentials
🅔 Give steroids and arrange repeat MRI brain as a neurology outpatient

Option3

[75]
You see a 54-year-old man with a 6-year history of back pain following a road traffic accident. He has presented to the
hospital due to poor control of his pain. He is experiencing shooting pains down his legs with an associated burning
sensation.
Despite intensive physiotherapy, his pain is still poorly controlled and requiring full dose codeine and paracetamol which is
failing to control the pain.
His MRI was normal and he has been discharged by the spinal team due to no operable cause identified for his pain.
You're asked to review him due to ongoing pain. The nursing team has asked if he can have morphine sulfate prescribed as
required to help with this. He was started by his general practitioner 3 months previously on amitriptyline which he hasn't
found of any benefit. You decide rather than just prescribing morphine that you can try to adjust his chronic pain
management aswell.
Which of the following options would be best for the management of his chronic pain?
🅐 Continue amitriptyline and start duloxetine
🅑 Continue amitriptyline and start gabapentin
🅒 Stop amitriptyline and start modified release morphine sulfate
🅓 Stope amitriptyline and start duloxetine
🅔 Continue amitriptyline and start tramadol

Option4

[76]
You are examining a patient who complains of double vision. Whilst looking forward the patient's right eye turns downwards
and outwards. On attempting to look to the patient's left the patient is unable to adduct the right eye and double vision
worsens. On looking right the angle of the squint is less.
What is the most likely underlying problem?
🅐 Left 3rd nerve palsy
🅑 Left 6th nerve palsy
🅒 Right 3rd nerve palsy
🅓 Right 4th nerve palsy
🅔 Right 6th nerve palsy

Option3
[77]
Which one of the following antibodies is associated with ocular opsoclonus-myoclonus in patients with breast cancer?
🅐 Anti-Hu
🅑 Anti-La
🅒 Anti-GAD
🅓 Anti-Yo
🅔 Anti-Ri

Option5

[78]
A 51-year-old man with a history of schizophrenia is reviewed. He has developed parkinsonism secondary to his
antipsychotic medication.
Which one of the following drugs is most useful in the management of tremor?
🅐 Apomorphine
🅑 Cabergoline
🅒 Selegiline
🅓 Amantadine
🅔 Benzhexol

Option5

[79]
A 55-year-old female is admitted with a seizure. The seizure fails to respond to repeated dose of intravenous lorazepam. You
decide to try a second line agent. Shortly after administration of the drug the patients blood pressure drops to 75/45 mmHg.
What drug is most likely implicated?
🅐 Sodium valproate
🅑 Carbamazepine
🅒 Lamotrigine
🅓 Phenytoin
🅔 Levetiracetam

Option4

[80]
Which of the following drugs is least likely to cause peripheral neuropathy?
🅐 Amiodarone
🅑 Vincristine
🅒 Trimethoprim
🅓 Isoniazid
🅔 Nitrofurantoin

Option3
[81]
A 28-year-old woman presents with worsening headaches over the last 2 months. She describes intermittent visual loss that
lasts for several seconds at a time. She also describes a 'whooshing' sound in the ears that seems to be in time with her
pulse. Her past medical history includes asthma, obesity and acne vulgaris. Her regular medications include a Fostair inhaler
and isotretinoin. Fundoscopy is performed and bilateral optic disc swelling is noted.
Given the likely diagnosis, what is the most appropriate treatment option?
🅐 Acetazolamide
🅑 Amitriptyline
🅒 Propranolol
🅓 Sumatriptan
🅔 Topiramate

Option1

[82]
What is the mechanism of action of sumatriptan?
🅐 5-HT3 agonist
🅑 5-HT2 antagonist
🅒 5-HT2 agonist
🅓 5-HT1 antagonist
🅔 5-HT1 agonist

Option5

[83]
A 55-year-old male presented following a collapse. In his initial workup, he is found to have postural hypotension despite
being on no anti-hypertensives. He was recently diagnosed with idiopathic Parkinson's disease and was started on
Levodopa three months ago, but has noticed no substantial change from using it. He also confides that he has been having
problems with impotence and has started using a catheter due to frequent episodes of urinary retention.
What isthe most likely diagnosis?
🅐 Multi-system atrophy
🅑 Normal progression of idiopathic Parkinson's disease
🅒 Normal pressure hydrocephalus
🅓 Progressive supranuclear palsy
🅔 Corticobasal syndrome

Option1

[84]
A 47-year-old man with a known history of schizophrenia is admitted to the Emergency Department due to confusion. A
bottle of procyclidine tablets are found in his pocket. On examination the temperature is 38.1ºC with a blood pressure of
155/100 mmHg. Neurological examination reveals a GCS of 13/15 but assessment of his peripheral nervous system is
difficult due to generalised increased muscle tone.
What is the most likely diagnosis?
🅐 Neuroleptic malignant syndrome
🅑 Procyclidine overdose
🅒 Catatonic schizophrenia
🅓 Clozapine induced agranulocytosis
🅔 Quetiapine induced rhabdomyolysis

Option1
[85]
Which one of the following is least recognised as an adverse effect of phenytoin use?
🅐 Megaloblastic anaemia
🅑 Peripheral neuropathy
🅒 Alopecia
🅓 Osteomalacia
🅔 Coarsening of facial features

Option3

[86]
A 35-year-old man presents with progressive weakness of his hands. On examination you notice wasting of the small
muscles of the hand. A diagnosis of syringomyelia is suspected.
Which one of the following features would most support this diagnosis?
🅐 Hyper-reflexia in the upper limbs
🅑 Loss of vibration sensation in the hands
🅒 Loss of temperature sensation in the hands
🅓 Loss of light touch sensation in the hands
🅔 Fasciculation of the small muscles of the hand

Option3

[87]
A 61-year-old woman presents with bilateral tinnitus. She reports no change in her hearing or other ear-related symptoms.
Ear and cranial nerve examination is unremarkable.
Which medication is she most likely to have recently started?
🅐 Ciprofloxacin
🅑 Nifedipine
🅒 Repaglinide
🅓 Quinine
🅔 Bendroflumethiazide

Option4

[88]
A 61-year-old female with a long-standing history of type 2 diabetes mellitus, hypertension and hypercholesterolemia
developed a sudden onset weakness of her right lower leg while preparing breakfast. Her son brought her to the
emergency department where she told the attending doctor that she had a similar episode two days ago but resolved after
1 hour. She can speak well and fully understand what the doctor tells her.
On examination, the doctor finds that the patient also had decreased touch sensation on her right leg. A non-contrast
computed tomography (CT) scan is unremarkable but a repeat CT scan after 12 hours showed an area of hypo-attenuation
in a region of the brain.
Which of the following arteries of the cerebral circulation is the most likely to beoccluded in this patient?
🅐 Posterior cerebral artery
🅑 Anterior cerebral artery
🅒 Middle cerebral artery
🅓 Basilar artery
🅔 Posterior inferior cerebellar artery

Option2
[89]
A 23-year-old man with difficult to control epilepsy is reviewed in clinic, four months after a change in his antiepileptic
medication. He has remained seizure free but has gained 5 kg in weight since last reviewed.
Which one of the following antiepileptic drugs is most associated with weightgain?
🅐 Ethosuximide
🅑 Sodium valproate
🅒 Levetiracetam
🅓 Carbamazepine
🅔 Lamotrigine

Option2

[90]
A 66-year-old woman presents with a six-month history of urinary frequency and incontinence. She has a background in
relapsing-remitting multiple sclerosis (MS).
A urine dip is normal.
What is the most appropriate next step in her management?
🅐 Botulinum toxin type A injections
🅑 Intermittent self-catheterisation
🅒 Micturating cystogram
🅓 Oxybutynin
🅔 Ultrasound kidneys, ureters and urinary bladder (KUB)

Option5

[91]
A 57-year-old Polish patient, who just moved to the UK 6 months ago, presents to the emergency department following a
fall. You notice that he has a wide-based gait, bilateral hand tremor and a nystagmus.
Which medication is he likely to be taking?
🅐 Ethosuximide
🅑 Levetiracetam
🅒 Lamotrigine
🅓 Phenytoin
🅔 Sodium valproate

Option4

[92]
A 32-year-old woman is brought into the emergency department by ambulance with following an episode in of vertigo,
diplopia and dysarthria, after which she became drowsy and responsive to pain only. Her symptoms came on over ten
minutes and lasted for approximately one hour. Her initial symptoms have now fully resolved, but she feels nauseous.
Neurological examination is unremarkable. She has a past medical history of migraine. She takes the progesterone-only
contraceptive pill, but no other medications.
What is the most likely diagnosis?
🅐 Focal aware seizure
🅑 Focal impaired awareness seizure
🅒 Migraine with brainstem aura
🅓 Transient ischaemic attack
🅔 Multiple sclerosis

Option3
[93]
A 54-year-old female with known breast cancer develops progressive motor disorder with impairment of right arm function.
She is clumsy with all movements and exaggerated 'flinging' actions are noted on examination. There is a similar pattern in
the right leg with to a much lesser extent. There are no other abnormalities on neurological examination.
Where is the lesion most likely to be?
🅐 Caudate nucleus
🅑 Lateral cerebellum
🅒 Pre-motor cortex
🅓 Sub-thalamic nucleus
🅔 Temporal lobe

Option4

[94]
A 19-year-old female university student presents to the first seizure clinic after her flatmate became worried she had a fit first
thing in the morning after a night out.
This seizure is the first she has had in her entire life. She does not drink or take recreational drugs but did feel sleep-
deprived and dizzy prior to the seizure. The flatmate shows a video that reveals the seizure appearing to be tonic-clonic in
nature, involving arms and legs, no incontinence, no tongue-biting and no post-ictal drowsiness.
Blood tests, lumbar puncture, CT head, MRI brain and electroencephalogram (EEG) are all unremarkable.
How long should this patient be advised they cannot drive for?
🅐 1 month
🅑 2 years
🅒 3 months
🅓 6 months
🅔 12 months

Option4

[95]
Each of the following features are seen in myotonic dystrophy, except:
🅐 Mild mental impairment
🅑 Round face
🅒 Frontal balding
🅓 Myotonia
🅔 Cataracts

Option2

[96]
A 76-year-old man is reviewed in the Elderly Medicine clinic. He is concerned about his increasing forgetfulness over the
past six months. His daughter notes he has generally 'slowed down' and struggles to follow conversations. Over the past
month he has noted increasingly frequent episodes of urinary incontinence. He has also had one episode of faecal
incontinence in the past week. On examination he is noted to have brisk reflexes and a short, shuffling gait. No cerebellar
signs are noted.
What is the most likely diagnosis?
🅐 Multiple system atrophy
🅑 Parkinson's disease
🅒 Normal pressure hydrocephalus
🅓 Urinary tract infection
🅔 Pick's disease

Option3
[97]
A 72-year-old woman with a past history of treated hypertension presents to the Emergency Department. Yesterday she had
a 2 hour episode where she couldn't find the right word when speaking. This has never happened before and there were no
associated features. Neurological examination is unremarkable and blood pressure was 150/100 mmHg. Her only current
medication is amlodipine.
What is the most appropriate management?
🅐 Aspirin 300 mg immediately + specialist review within 2 weeks
🅑 Specialist review within 2 weeks
🅒 Aspirin 300 mg immediately + specialist review within 24 hours
🅓 Aspirin 75 mg + outpatient CT brain
🅔 Specialist review within 24 hours

Option3

[98]
A 28-year-old man is seen in the neurology clinic for a follow-up appointment having been diagnosed with focal seizures 12
months prior.
The patient was originally started on levetiracetam as a first-line treatment however, despite good compliance, the patient
reports he is still having regular episodes of acute right limb jerking associated with other focal seizure features. He has been
on this first-line treatment for over 6 months with minimal improvement.
What replacement medication should the patient be trialled on?
🅐 Carbamazepine
🅑 Ethosuximide
🅒 Lamotrigine
🅓 Phenytoin
🅔 Sodium valproate

Option3

[99]
A 63-year-old man is prescribed selegiline for Parkinson's disease. What is the mechanism of action?
🅐 Dopamine receptor antagonist
🅑 Dopamine receptor agonist
🅒 Monoamine oxidase-B inhibitor
🅓 Antimuscarinic
🅔 Catechol-O-methyl transferase inhibitor

Option3

[100]
A 25-year-old female with a history of depression presents to her GP with a two day history of numbness affecting the C6
distribution in her right arm. There is no history of neck pain or injury. Neurological examination confirms reduced sensation
in that dermatome but is otherwise unremarkable. She reports no similar episodes previously although does describe an
episode three months ago of reduced vision and painful movements in her right eye. This resolved spontaneously and she
did not seek medical attention.
What is the most likely diagnosis?
🅐 Somatisation disorder
🅑 Huntington's disease
🅒 Multiple sclerosis
🅓 Syringomyelia
🅔 Conversion disorder

Option3
[101]
A 31-year-old female with progressive leg weakness has nerve conduction studies for suspected Guillain-Barre syndrome.
Which one of the following findings would be most consistent with thisdiagnosis?
🅐 Reduced conduction velocity
🅑 Extended series of repetitive discharges lasting up to 30 seconds
🅒 Increased conduction velocity
🅓 Diminished response to repetitive stimulation
🅔 Reduced wave amplitude

Option1

[102]
Which one of the following is most associated with downbeat nystagmus?
🅐 Arnold-Chiari malformation
🅑 Pseudobulbar palsy
🅒 Jugular foramen syndrome
🅓 Acoustic neuroma
🅔 Cerebellar vermis lesions

Option1

[103]
Which one of the following features is most associated with frontal lobe lesions?
🅐 Wernicke's aphasia
🅑 Gerstmann's syndrome
🅒 Perseveration
🅓 Cortical blindness
🅔 Superior homonymous quadrantanopia

Option3

[104]
A 31-year-old man attends his appointment with an oncologist after being referred by his family physician. The man
reported feeling nauseous and having a persistent headache for the past month. The pain was not relieved by the regular
over-the-counter painkiller. The patient confirms that he has a significant family history of cancer and this is why he has been
referred to see the oncologist.
After chromosome analysis which revealed an abnormality related to the chromosome 3p, the patient is explained that his
condition will put him at higher risk of developing several tumors, which could be both benign and malignant. The
oncologist tells the patient that this condition can be passed on to future generations and that the patient's children have a
fifty-percent chance of developing the condition, given his spouse does not carry the mutation as well. The patient is happy
to know that he will not need a screening flexible sigmoidoscopy.
Which of the following diseases is the patient at the highest risk of developing due to his underlying condition?
🅐 Lung carcinoma
🅑 Clear-cell renal carcinoma
🅒 Prostate carcinoma
🅓 Breast carcinoma
🅔 Osteogenic sarcoma

Option2
[105]
A 65-year-old gentleman presents with progressive dementia and behavioural abnormalities. On closer questioning, he
seems to have deficits in concentration, memory and judgement difficulties. There is a family history of psychosis. During the
consultation, you notice the patient's gait is ataxic with a notable jerk of his left hand and general hypokinesia. There is also
nystagmus during horizontal gaze. Romberg's and Dix-Hallpike tests are negative.
What is the most likely diagnosis?
🅐 Vescular dementia
🅑 Lewy body dementia
🅒 Sporadic Creutzfeldt-Jakob disease
🅓 Alzheimer's disease
🅔 Frontotemporal dementia

Option3

[106]
A 59-year-old man with no significant past medical history is admitted to hospital following an ischaemic stroke. He
presented outside of the thrombolysis window and is treated with aspirin for the first few days. His blood pressure is 130/80
mmHg, fasting glucose is 5.6 mmol/l and fasting cholesterol is 3.9 mmol/l. He makes a good recovery and has regained
nearly all of his previous functions upon discharge.
Following recent NICE guidelines, which of the following medications should he be taking upon discharge (i.e. after 14
days)?
🅐 Aspirin + statin
🅑 Aspirin + dipyridamole + statin + ramipril
🅒 Clopidogrel + statin
🅓 Aspirin + dipyridamole
🅔 Aspirin + dipyridamole + statin

Option3

[107]
Which one of the following features is most associated with temporal lobe lesions?
🅐 Astereognosis
🅑 Auditory agnosia
🅒 Visual agnosia
🅓 Disinhibition
🅔 Expressive (Broca's) aphasia

Option2

[108]
A 45-year-old female is diagnosed with a glioma in the parietal lobe after being investigated for new onset seizures. Which
one of the following features is she most likely to develop?
🅐 Visual agnosia
🅑 Auditory agnosia
🅒 Acalculia
🅓 Inability to generate a list
🅔 Expressive (Broca's) aphasia

Option3
[109]
A 36-year-old man presents with 4 days of progressive limb weakness and difficulty mobilising. He describes the upper arms
and legs as having been affected first, followed by the lower limbs and now the digits. Alongside this, he is also
experiencing double vision. He has no significant past medical history but did experience suspected food poisoning with
diarrhoea 2 weeks ago.
On examination, power is 1/5 in all limbs. Tendon reflexes are reduced. Ophthalmoplegia is evident on cranial nerve
examination.
What antibody is classically associated with the underlying condition?
🅐 Anti-muscle-specific tyrosine kinase (anti-MuSK)
🅑 Anti-GM1
🅒 Anti-GQ1b
🅓 Anti-acetylcholine receptor (AChR)
🅔 Anti-voltage-gated calcium channel (anti-VGCC)

Option3

[110]
A 70-year-old man presents to the emergency department following an episode of right-sided body weakness. The
weakness developed gradually, and he was not able to use his right arm and leg for about 3 hours before resolved
spontaneously. The patient reports a similar episode 4 days ago.
He has a past medical history of diabetes mellitus and hypertension for the past 8 years. His BMI is 32 kg/m². Current
medications include metformin, sitagliptin, amlodipine, rosuvastatin, and aspirin.
A plain CT brain performed was unremarkable, while a CT angiogram revealed 40% stenosis of the left internal carotid artery.
Which of the following is the most appropriate treatment strategy to help prevent furthercomplications?
🅐 Continue antiplatelet therapy with aspirin only
🅑 Carotid angioplasty
🅒 Addition of clopidogrel
🅓 Fibrinolytic therapy with alteplase
🅔 Carotid endarterectomy

Option3

[111]
In the treatment of migraine, sumatriptan is an example of a:
🅐 Beta-blocker
🅑 Alpha-blocker and a partial 5-HT2 agonist
🅒 Specific 5-HT1 agonist
🅓 5-HT2 antagonist
🅔 Tricyclic antidepressant

Option3

[112]
Lateral medullary syndrome is caused by occlusion of which one of the following blood vessels?
🅐 Anterior inferior cerebellar artery
🅑 Posterior cerebral artery
🅒 Lateral sinus thrombosis
🅓 Middle cerebral artery
🅔 Posterior inferior cerebellar artery

Option5
[113]
A 19-year-old man is admitted following a generalised seizure. No past history is available as the man is currently in a
postictal state. On examination it is noted that he has three patches of hypopigmented skin and fibromata under two of his
finger nails.
What is the most likely diagnosis?
🅐 Neurofibromatosis
🅑 Lennox-Gastaut syndrome
🅒 Multiple endocrine neoplasia type 1
🅓 Birt-Hogg-Dube syndrome
🅔 Tuberous sclerosis

Option5

[114]
A 24-year-old patient is referred to the neurology clinic. She has been experiencing intermittent jerking of her right upper
limb. There is no loss of awareness during these episodes. They occur frequently and she finds them distressing. Her past
medical history is unremarkable other than an episode of hypoxia at birth.
The examination at the time of review is unremarkable.
MRI brain: Encephalomalacia affecting the left motor cortex
Despite the use of levetiracetam, her symptoms persist.
What is the most appropriate medication to choose at this point?
🅐 Carbamazepine
🅑 Clobazam
🅒 Lamotrigine
🅓 Lorazepam
🅔 Sodium valproate

Option3

[115]
A 33-year-old woman is reviewed in the epilepsy clinic. Her epilepsy has been well controlled on carbamazepine 40 0mg
twice daily. She is about to give birth to a baby girl and is very keen to breastfeed.
What should the patient be advised?
🅐 Breastfeeding should be avoided whilst taking anti-epileptic medication
🅑 Increase the dose of carbamazepine by 50%
🅒 No changes required
🅓 Switch carbamazepine to lamotrigine
🅔 Switch carbamazepine to levetiracetam

Option3

[116]
A 65-year-old man is referred to the care of the elderly clinic with a three-month history of recurrent falls.
On examination, there is bradykinesia, rigidity and a shuffling gait. He has hypomimia.
Which of the following listed clinical features is most characteristic of the idiopathic form of this condition?
🅐 Asymmetric tremor
🅑 Postural hypotension
🅒 Symmetrical tremor
🅓 Vertical gaze palsy
🅔 Visual hallucinations

Option1
[117]
A 21-year-old musician is brought to hospital after collapsing on stage. There was no loss of consciousness and he sustained
only minor bruising to his knees. Another musician onstage at the time notes he had been laughing at the time before his
legs suddenly 'gave way' and his mouth twitched for a few seconds.
The patient reports a similar episode occurring a few days previously. He has been extremely fatigued over the last several
months, which he had attributed to touring.
ECG and blood tests are normal.
What is the most likely underlying diagnosis?
🅐 Gelastic seizures
🅑 Narcolepsy
🅒 Functional neurological disorder
🅓 Tourette's syndrome
🅔 Lennox-Gastaut syndrome

Option2

[118]
A patient is given ondansetron for chemotherapy related nausea. What is the most likely side-effect?
🅐 Constipation
🅑 Dry mouth
🅒 Insomnia
🅓 Visual disturbance
🅔 Pruritus

Option1

[119]
You are called to review a 55-year-old man on the wards who started fitting around 5 minutes ago. He was admitted three
days ago following an acute coronary syndrome. His past medical history includes tonic-clonic epilepsy which is generally
well controlled on sodium valproate. On your arrival he is still fitting. Oxygen saturations are 99% on 100% oxygen and his
pulse rate is 96/min. Intravenous access is already in-situ.
What is the most appropriate next step?
🅐 Observe for a further 5 minutes to see if the seizure stops spontaneously
🅑 Give IV lorazepam
🅒 Give buccal midazolam
🅓 Give IV phenytoin
🅔 Fast-bleep an anaethetic doctor

Option2

[120]
A 55-year-old woman was found wandering in the street wearing inappropriate clothing for the current climate. She
admitted having memory issues for a while but denied any past medical history. She was discharged home after a normal CT
head scan to the memory clinic. One month later she was readmitted due to acute deterioration of mental status, failure to
thrive, and mutism.
Upon assessment, she was bed-bound and mute. A neurological exam demonstrated spastic contractions of her limbs with
diffuse myoclonic twitching.
What is the most likely diagnosis here?
🅐 Frontotemporal dementia
🅑 Huntington's disease
🅒 Sporadic Creutzfeldt-Jakob disease
🅓 Transient global amnesia
🅔 Vascular dementia

Option3
[121]
A 33-year-old man is seen in the Emergency Department with a fall. He is a known alcoholic and drank a bottle of vodka and
2 cans of cider in the last 24 hours. He is started on a chlordiazepoxide reducing regimen by the junior doctor. He is post-
taken by the medical consultant who examines him requests that he is urgently started on Pabrinex as he has features
consistent with Wernicke's encephalopathy.
Which of the following is a feature of Wernicke's encephalopathy?
🅐 Dysarthria
🅑 Retrograde amnesia
🅒 Ophthalmoplegia
🅓 Confabulation
🅔 Tremor

Option3

[122]
A 48-year-old man is referred to an ear, nose and throat specialist with slowly progressive hearing loss and tinnitus. He has a
past medical history of heart failure with preserved ejection fraction and takes regular furosemide. He has a family history of
early-onset hearing loss.
On examination, otoscopy is unremarkable bilaterally. There is evidence of hearing loss bilaterally.
Blood tests:
• Urea: 5.2 mmol/L (2.0 - 7.0)
• Creatinine: 89 μmol/L (55 - 120)
Pure tone audiometry demonstrates bilateral conductive hearing loss.
What is the most likely diagnosis?
🅐 Alport's syndrome
🅑 Furosemide treatment
🅒 Glue ear
🅓 Meniere's disease
🅔 Otosclerosis

Option5

[123]
A 64-year-old man with a history of Parkinson's disease is reviewed in clinic and a decision has been made to start him on
cabergoline.
Which one of the following adverse effects is most strongly associated with this drug?
🅐 Optic neuritis
🅑 Transient rise in liver function tests
🅒 Pulmonary fibrosis
🅓 Renal failure
🅔 Thrombocytopenia

Option3
[124]
A 58 year old gentleman presents with left sided paraesthesias affecting his thumb and first finger. He complains of grip
weakness and dropping objects unintentionally. On examination, there is wasting over the thenar eminence.
Which of the following signs would suggest a diagnosis otherthan carpal tunnel syndrome?
🅐 Positive Hoffmans sign
🅑 Thenar muscle wasting
🅒 Unilateral weakness of pincer grip
🅓 Positive Phalens test
🅔 Positive Tinnels test

Option1

[125]
A 32-year-old man is brought in following a seizure. His family reports he has been severally unwell the past few days with a
fever and vomiting. He was started on antibiotics by his GP last week for suspected bacterial sinusitis following a prolonged
period of symptoms including thick nasal mucus and facial discomfort. However, his family confirms he did not complete the
antibiotic course.
A CT head with contrast is performed and demonstrates a ring-enhancing lesion with evidence of inflammation of the
sinuses.
What antibiotic regime should be commenced?
🅐 IV ceftriaxone and ampicillin
🅑 IV ceftriaxone and co-trimoxazole
🅒 IV ceftriaxone and gentamicin
🅓 IV ceftriaxone and metronidazole
🅔 IV ceftriaxone and vancomycin

Option4

[126]
A 36-year-old man with difficult to control epilepsy is reviewed in clinic. He is currently taking phenytoin but presents due to
fatigue. A full blood count is performed:
• Hb: 10.1 g/dl
• MCV: 121 fl
• Plt: 234 × 10⁹/l
• WCC: 4.6 × 10⁹/l
What is the most likely cause for his tiredness?
🅐 Iron deficiency
🅑 Vitamin B12 deficiency
🅒 Liver dysfunction secondary to phenytoin
🅓 Haemolytic anemia secondary to phenytoin
🅔 Folate deficiency

Option5
[127]
An 84-year-old woman presents with confusion. She is alert with a GCS of 15/15. When taking a history, she speaks quickly
and fluently but her sentences do not make sense. She does not follow your instructions. When you ask her to repeat words
after you, she fails to do so and responds with a long sentence that has no meaning. You phone her daughter who tells you
that these changes happened all of a sudden a few hours previously.
What form of speech problem does this patient have?
🅐 Broca's aphasia
🅑 Conductive aphasia
🅒 Dysarthria
🅓 Global aphasia
🅔 Wernicke's aphasia

Option5

[128]
A 27-year-old man presents with a sudden onset occipital headache earlier in the day. He describes it as the worst headache
of his life and has vomited twice. He has no past medical history.
Blood pressure is 121/72 mmHg, heart rate 102 bpm, temperature 36.1ºC, and respiratory rate 20/min.
There is nuchal rigidity and photophobia but no rash. Cranial nerve examination is normal.
• Hb: 155 g/L Male: (135-180)
• Platelets: 166 × 10⁹/L (150 - 400)
• WBC: 4.9 × 10⁹/L (4.0 - 11.0)
• CRP: 4 mg/L (< 5)
Eight hours after the onset of headache a CT head without contrast is performed and has just been reported as normal.
What is the most appropriate next step?
🅐 CT angiography of the head
🅑 Perform a lumbar puncture immediately
🅒 Perform a lumbar puncture in 16 hours
🅓 Perform a lumber puncture in 4 hours
🅔 Start antibiotics for meningitis

Option4

[129]
A 78-year-old right-handed female is admitted with an acute onset stroke of 2 hours duration. The decision was taken by the
stroke team for thrombolysis which cures her symptoms. She is subsequently transferred to the high dependency unit for
closer monitoring. Overnight, she has three bouts of vomiting and is seen by an FY2 who detects a new onset right-left
disorientation and acalculia.
An urgent CT scan is requested which reveals an intracerebral haemorrhage.
Which area of the brain is most likely to have been affected?
🅐 Left temporal lobe
🅑 Left parietal lobe
🅒 Right frontal lobe
🅓 Right temporal lobe
🅔 Right parietal lobe

Option2
[130]
A 50-year-old man develops chronic, severe pain after sustaining a brachial plexus injury as a result of motorbike accident.
He has had no benefit from paracetamol or ibuprofen. He has had a trial of amitriptyline which was not successful.
Following recent NICE guidelines, what is the most appropriate medication to consider?
🅐 Sertraline
🅑 Topical lidocaine
🅒 Carbamazepine
🅓 Pregabalin
🅔 Buprenorphine

Option4

[131]
You are asked to perform a neurological exam of the lower limbs on a patient with multiple sclerosis.
Which one of the following findings is least typical?
🅐 Decreased tone
🅑 Patellar clonus
🅒 Upgoing plantars
🅓 Weakness
🅔 Brisk reflexes

Option1

[132]
Which one of the following features is most associated with parietal lobe lesions?
🅐 Grestmann's syndrome
🅑 Perseveration
🅒 Cortical blindness
🅓 Superior homonymous quadrantanopia
🅔 Wernicke's aphasia

Option1

[133]
A 70-year-old male patient presented to the neurology clinic with weakness in both lower limbs, progressively worsening
tingling with difficulty getting up from a chair. He reports laxity in his ankles for a while now with a tendency for his feet to
turn inwards. As a result, he has been using orthotics for one to two years.
Physical exam was remarkable for bilateral high arched feet with a visible pes cavus deformity, wasting of distal legs and feet,
and weakness in bilateral toe flexors.
What is the most likely diagnosis in this case?
🅐 Charcot-Marie-Tooth disease
🅑 Diabetic amyotrophy
🅒 Duchenne muscular dystrophy
🅓 Inclusion body myositis
🅔 Myotonic dystrophy

Option1
[134]
A 45-year-old male presents to the acute medical take with a three day history of progressive bilateral leg weakness.
Which of the following constellation of signs on examination is the most consistent with a diagnosis of Guillain-Barre
Syndrome?
🅐 Bradycardia with lower limb hyporeflexia and flaccid paralysis
🅑 Tachycardia with lower limb hyporeflexia and spastic paralysis
🅒 Bradycardia with lower limb hypereflexia and spastic paralysis
🅓 Tachycardia with lower limb hyporeflexia and flaccid paralysis
🅔 Tachycardia with lower limb hypereflexia and flaccid paralysis

Option4

[135]
A 33-year-old man presents to the emergency department with recurrent headaches and sweating. He has no past medical
history.
At the time of examination, the headache has resolved but the blood pressure is 210/94 mmHg. Examination demonstrates
no skin lesions and a non-tender thyroid with no nodules.
A CT head is normal but a CT chest, abdomen, and pelvis demonstrates a lesion on the leftadrenal gland and an exophytic
lesion in the right kidney.
What is the most likely diagnosis?
🅐 Birt-Hogg-Dube syndrome
🅑 Multiple endocrine neoplasia type 1 (MEN1)
🅒 Multiple endocrine neoplasia type 2 (MEN2)
🅓 Polycystic kidney disease
🅔 Von Hippel-Lindau syndrome

Option5

[136]
A 45-year-old female with a past medical history of asthma is diagnosed as having essential tremor.
What is the most suitable management?
🅐 Amitriptyline
🅑 Propranolol
🅒 Sodium valproate
🅓 Carbamazepine
🅔 Primidone

Option5

[137]
A 28-year-old woman with suspected multiple sclerosis has been reviewed in the outpatient neurology clinic with new
diplopia when looking the right.
On examination of the patient's eye movements, the left eye does not adduct on rightward gaze and the right eye exhibits
nystagmus. All other eye movements are normal. The pupils are equal and reactive to light.
The neurologist arranges an MRI scan of the brain.
Where is the lesion likely to be seen on MRI imaging?
🅐 Right abducens nerve
🅑 Left paramedian area of the midbrain and pons
🅒 Left ventriomedial periaqueductal grey matter at the level of the superior colliculus
🅓 Right paramedian area of the midbrain and pons
🅔 Right ventromedial periaqueductal grey matter at the level of the superior colliculus

Option2
[138]
A 66-year-old woman is taken to the emergency department by her son, who suspects that she may have had a stroke.
On examination, she has right-sided facial pain and temperature sensation loss. She has left-sided arm and leg pain, with
associated temperature sensation loss of the left arm and leg. Ataxia is also demonstrated, as well as nystagmus.
Given the findings, which artery is most likely to have been affected?
🅐 Anterior cerebral artery
🅑 Anterior inferior cerebellar artery
🅒 Middle cerebral artery
🅓 Posterior cerebral artery
🅔 Posterior inferior cerebellar artery

Option5

[139]
A 54 year-old woman presents with severe headache after a collapse while at a wedding reception. She had been dancing
when she abruptly fell to the ground. On examination she is neurologically intact, but clearly finds it difficult to concentrate
and is in some discomfort due to her headache. She is afebrile and denies chest pain. She reports that she had about 3
glasses of wine tonight and she normally drinks 10-12 units weekly. Her husband reports that she was unconscious for about
one minute and was drowsy and confused on waking.
ECG: normal sinus rhythm
What is the most appropriate management from the options below?
🅐 Urgent CT brain
🅑 Referral to first-fit clinic
🅒 Intravenous ceftriaxone and aciclovir
🅓 Referral to outpatient ambulatory ECG monitoring
🅔 Discharge with alcohol cessation advice

Option1

[140]
A 37-year-old female patient is brought into the emergency department with a 5-day history of altered personality, visual
and auditory hallucinations. On palpation of the abdomen, a mass is felt in the left iliac fossa. Ultrasound abdomen suggests
a left ovarian tumour. Her basic observations are as follows:
• Oxygen saturation: 99% on air
• Heart rate: 98 beats/minute
• Respiratory rate: 28 breaths/minute
• Temperature: 37.9 °C
What is the most likely diagnosis?
🅐 Meningitis
🅑 Anti-NMDA receptor encephalitis
🅒 Rabies
🅓 Japanese encephalitis
🅔 Mania

Option2
[141]
A 35-year-old man with a history of migraines has been admitted to the medical take with a right-sided hemiparesis. On
closer questioning, there is a family history of stroke and migraines. A CT brain reveals multiple hypodensities within the
basal ganglia and temporal lobes, out of keeping for his age. A diagnosis of CADASIL is suspected.
What is the pathophysiology of this condition?
🅐 NOTCH3 mutation
🅑 NOD/CARD15 mutation
🅒 FXN mutation
🅓 GLA mutation
🅔 SCN5a mutation

Option1

[142]
A 53-year-old woman presents to the emergency department with a sudden-onset, severe headache, describing it as the
worst of her life. She had been sitting at her desk when the headache came on. She has associated nausea and vomiting.
On examination, she has some neck stiffness and photophobia and appears drowsy. A CT scan shows hyperdense across
the basal cisterns and sulci.
Which of the following is indicated in managing the complications of this condition?
🅐 Amlodipine
🅑 Diltiazem
🅒 Felodipine
🅓 Nifedipine
🅔 Nimodipine

Option5

[143]
A 70 year old man has decompressive surgery for degenerative cervical myelopathy. Three years later he presents with neck
pain and hand paraesthesias.
Which one of the following management strategies is recommended?
🅐 Trial of neuropathic analgesia and cervical nerve root injections
🅑 Investigate with nerve conduction studies and EMG in the first instance
🅒 Urgent AP/lateral cervical spine radiographs as an MRI scan is contradicated
🅓 Urgent referral to spinal surgery or neurosurgery
🅔 Refer to physiotherapy services

Option4

[144]
Which one of the following is not a recognised adverse effect of phenytoin use?
🅐 Slurred speech
🅑 Nystagmus
🅒 Gynaecomastia
🅓 Diplopia
🅔 Ataxia

Option3
[145]
A 61-year-old woman with a history of metastatic breast cancer complains of nausea following a chemotherapy infusion. You
elect to prescribe ondansetron.
What is the most important site of action of this drug?
🅐 Efferent branch of the vagal nerve
🅑 Substantia nigra
🅒 Gastric mucosa
🅓 Hypothalamus
🅔 Medulla oblongata

Option5

[146]
A 14-year-old male is noted to have optic atrophy on fundoscopy. Neurological exam reveals dysarthric speech and
nystagmus. Knee and ankle jerks are absent but there is an extensor plantar response.
What is the likely diagnosis?
🅐 Leber's optic atrophy
🅑 Ataxic telangiectaisa
🅒 Friedreich's ataxia
🅓 Subacute combined degeneration of spinal cord
🅔 Multiple sclerosis

Option3

[147]
Which one of the following statements regarding the stopping of anti-epileptic drugs (AED) is most correct?
🅐 Can be considered if seizure free for > 5 years, with AEDs being stopped over 2-3 months
🅑 Can be considered if seizure free for > 2 years, with AEDs being stopped over 2-3 months
🅒 Can be considered if seizure free for > 1 years, with AEDs being stopped over 2-3 months
🅓 Can be considered if seizure free for > 5 years, with AEDs being stopped over 8-12 months
🅔 Can be considered if seizure free for > 1 years, with AEDs being stopped over 8-12 months

Option2

[148]
Which one of the following side-effects is not recognised in patients taking sodium valproate?
🅐 Alopecia
🅑 Weight gain
🅒 Hepatitis
🅓 Induction P450 system
🅔 Teratogenicity

Option4
[149]
A 67-year-old man presents to his general practitioner with his wife. She explains that, for several days, he has had difficulty
talking. His speech is often broken and halting. She feels that he is understanding whatever is said to him. His past medical
history includes hypertension and ischaemic heart disease.
On examination, he is unable to verbalise clearly - his speech is non-fluent and halting, managing 3 or 4 words at a time. He
is unable to repeat phrases when asked. However, he is able to follow commands.
Which area of the brain is most likely to be affected?
🅐 Arcuate fasciculus
🅑 Inferior frontal gyrus
🅒 Precentral gyrus
🅓 Superior temporal gyrus
🅔 Ventromedial prefrontal cortex

Option2

[150]
A 62-year-old man is referred to the neurology clinic with worsening symptoms over the past few months. The neurologist
suspects the patient has progressive supranuclear palsy.
Which one of the following features is least likely to be seen in this patient?
🅐 Poor response to L-dopa
🅑 Impairment of horizontal gaze
🅒 Falls
🅓 Cognitive impairment
🅔 Slurring of speech

Option2

[151]
Which one of the following is a contraindication to the use of a triptan in the management of migraine?
🅐 Concurrent pizotifen use
🅑 Patients older than 55 years
🅒 A history of epilepsy
🅓 Previous intracranial tumour
🅔 A history of ischaemic heart disease

Option5

[152]
Each one of the following is associated with Friedreich's ataxia, except:
🅐 Increased risk of deep vein thrombosis
🅑 Optic atrophy
🅒 Cardiomyopathy
🅓 Nystagmus
🅔 High-arched palate

Option1
[153]
A 21-year-old man presents with a short history of severe headache, fever and neck stiffness.
An initial CT brain is normal and you proceed to perform a lumbar puncture.
His lumbar puncture results are as follows.
• Appearance: Cloudy
• Opening pressure: 31 cmCSF (5 - 20)
• WBC count: 357 cells/μL (99% polymorphs) (<1)
• Glucose: 0.4 mmol/L (2.5 - 3.5)
• Protein: 1.3 g/L (0.18 - 0.45)
The patient is started on IV antibiotics and steroids and after a few days begins to recover.
He asks you about his prognosis and asks if he is likely to be left with any long-lasting effects of this illness.
What is the most common complication of his condition?
🅐 Adrenal insufficiency
🅑 Cognitive impairment
🅒 Conductive hearing loss
🅓 Epilepsy
🅔 Sensorineural hearing loss

Option5

[154]
A 42-year-old woman is referred to the clinic having been diagnosed with a phaeochromocytoma after originally presenting
with episodes of headaches, tachycardia and sweating. Given its relatively rare frequency, the patient is screened for an
underlying cause.
Following investigations an autosomal dominant condition that predisposes patients to neoplasia is confirmed. The
syndrome is due to an abnormal gene located on the short arm of chromosome 3 and is associated with visceral cysts and
tumours.
What cancer is most commonly associated with this patient's syndrome?
🅐 Adrenal cortex
🅑 Ovarian
🅒 Pancreatic
🅓 Parafollicular cells
🅔 Renal cell

Option5

[155]
A 67-year old male recently attended A&E, with a 3 month history of bilateral paraesthesias and twitching affecting the
thumb, first finger and lateral forearm. He denied any trauma. A MRI scan of his spine was performed and revealed cervical
canal stenosis with mild cord compression. He was discharged and advised to see his GP for follow-up.
Which of the following is the most appropriate initial step in management?
🅐 Refer to spinal surgery services
🅑 Refer for locally commissioned cervical root injections and review after 6 weeks
🅒 Enlist on the weekly minor ops clinic for carpal tunnel decompression
🅓 Commence neuropathic analgesia in the first instance and consider surgical evaluation if this does not work
🅔 Refer to physiology services and review in 6 weeks

Option1
[156]
A 20-year-old right-handed male presents to your neurology clinic with recurrent shock-like jerking of his right arm. He works
as a painter and finds he ends up throwing the paintbrush across the room when these movements occur. On further
questioning you note that these episodes have been ongoing since childhood but they were infrequent, they now present
four times a week and are significantly impacting his life. Theses episodes are always the same, last for approximately ten
seconds before self-resolving. There is no loss of consciousness during these episodes. He denies any headache during
these episodes.
There are no neurological findings on examination.
He is admitted for a three-day electroencephalogram (EEG), the results summary is as follows:
Brief bursts of poly-spike and wave discharge during ictal episodes.
Given the most likely diagnosis, what is the optimal first-line treatment for this patient?
🅐 Carbamazepine
🅑 Clobazam
🅒 Lamotrigine
🅓 Oxcarbazepine
🅔 Sodium valproate

Option5

[157]
A 24-year-old woman with Charcot-Marie-Tooth disease (type 1) asks how likely it is that any future children will have the
disease.
What is the most accurate answer?
🅐 Three times as likely as background population
🅑 0.25
🅒 Between 5-10%
🅓 Same as background population
🅔 0.5

Option5

[158]
A 23-year-old woman is admitted after a first seizure. It was witnessed as tonic-clonic, lasting 3 minutes before self-
terminating. Brain CT and MRI and an electroencephalogram (EEG) are all normal. Her main concern is whether or not she
can drive.
According to the DVLA, for how long must she abstain from driving?
🅐 1 month
🅑 3 months
🅒 6 months
🅓 12 months
🅔 5 years

Option3
[159]
A 75-year old gentleman presents with a short history of neck pain, paraesthesia in his finger tips and progressive leg
weakness. Following a MRI scan of his spine, he is diagnosed with degenerative cervical myelopathy due to a C4/5 disc
prolapse.
Which of the following is the most appropriate management?
🅐 Cervical decompressive surgery
🅑 Cervical nerve root injection
🅒 Analgesia and referral to physiotherapy
🅓 Analgesia and review in 4 weeks time
🅔 Analgesia and a hard cervical collar and review in 4 weeks

Option1

[160]
Each of the following are causes of peripheral neuropathy. Which one is associated with predominately sensory loss?
🅐 Diphtheria
🅑 Hereditary sensorimotor neuropathies
🅒 Porphyria
🅓 Lead poisoning
🅔 Uraemia

Option5

[161]
A 19-year-old female presents complaining of visual disturbance. Examination reveals a bitemporal hemianopia with
predominately the lower quadrants being affected.
What is the most likely lesion?
🅐 Brainstem lesion
🅑 Craniopharyngioma
🅒 Frontal lobe lesion
🅓 Pituitary macroadenoma
🅔 Right occipital lesion

Option2

[162]
An 80-year-old man presents to the Emergency Department reporting that he awoke this morning with dizziness and
vomiting. On further questioning, you establish that he has a sensation of the room spinning around him, which is worse
when he moves his head quickly, although persists with his head still, and that he vomits when these symptoms are at their
most severe. He is unsteady on his feet and staggers when you try to walk him around his bed. He has no other symptoms or
signs. He has a past medical history of hypertension, high cholesterol, type II diabetes which is diet controlled, and
osteoarthritis.
Which of the following diagnoses is it most important to exclude?
🅐 Posterior circulation stroke
🅑 Postural hypotension
🅒 Benign paroxysmal positional vertigo
🅓 Meningo-encephalitis
🅔 Viral labyrinthitis

Option1
[163]
A 45-year-old woman is found unconscious. Her neighbour tells you that she was taking multiple medications for depression
and schizophrenia. She is brought in to the emergency department where she is unconscious, febrile at 40ºC, systolic blood
pressure of 180 mmHg, heart rate140/min, and has muscle rigidity and no myoclonus.
Her CK is 68,000 iu/L.
What is the most likely diagnosis?
🅐 Serotonin syndrome
🅑 Neuroleptic malignant syndrome
🅒 Catatonia
🅓 Malignant hyperthermia
🅔 Acute dystonia

Option2

[164]
A 76-year-old man presents to the Emergency Department, this is his 5th attendance in 2 months. His past medical history
includes hypertension, hypercholesterolaemia and chronic alcohol use. You have never seen him and prior to entering the
cubicle the registrar tells you to give his usual treatment of fluids, Pabrinex and discharge back home. You manage to take a
minimal history from the patient but he denies falling or a history of head trauma. You examine him as best you can. He has
an ataxic gait, horizontal nystagmus and dysmetria.
What is an important cause of his symptoms not to be missed?
🅐 Alcohol withdrawal
🅑 Extradural haemorrhage
🅒 Subdural haemorrhage
🅓 Vestibular neuronitis
🅔 Cerebellar stroke

Option5

[165]
A 26-year-old Asian woman with no medical background presented with altered mental state, headaches, and seizures. She
had a dermal filler injection of her lip for aesthetic purposes performed 1-month prior to her acute presentation. Upon
clinical review, she underwent imagingof her brain with an initial CT scan. The scan revealed the presence of tenuous low-
densitychanges within regions of the anterior and medial temporal lobe as well as the island of Reil.
What is the most likely diagnosis in this case?
🅐 Cytomegalovirus encephalitis
🅑 Herpes simplex virus encephalitis
🅒 Japanese encephalitis
🅓 Limbic encephalitis
🅔 Varicella zoster virus encephalitis

Option2

[166]
A 43-year-old man is suspected of having a common peroneal nerve palsy following a fracture of his fibula. Each one of the
following features may be seen in such lesions, except:
🅐 Wasting of the anterior tibial and peroneal muscles
🅑 Weakness of foot dorsiflexion
🅒 Weakness of extensor hallucis longus
🅓 Weakness of foot eversion
🅔 Sensory loss over the medial aspect of the lower limb

Option5
[167]
A 29-year-old man with a history of schizophrenia is taken to the local Emergency Department as he is generally unwell. He is
currently taking olanzapine and citalopram. On examination he is noted to have a temperature of 37.0ºC and his blood
pressure is 170/100 mmHg.
Which other examination finding would best support a diagnosis of neuroleptic malignant syndrome?
🅐 Ataxia
🅑 Hyperreflexia
🅒 Muscle rigidity
🅓 Tremor
🅔 Papilloedema

Option3

[168]
A 42-year-old gentleman is seen in General Practice with double vision, weakness and a dry mouth.
He has a known history of type 1 diabetes, hypothyroidism and small cell lung cancer.
On examination, you note a bilateral ptosis and proximal muscle weakness in the upper limbs with absent reflexes. With
sustained isometric contraction of the upper limbs, you note an improvement in the weakness. He also has weakness of neck
flexion and eyelid elevation after sustained upgaze.
What is the likely autoantibody associated with this condition?
🅐 Anti-Musk antibody
🅑 Anti-mitochondrial antibody
🅒 Anti-voltage-gated calcium channel antibody
🅓 Anti-Jo1 antibody
🅔 Anti-Mi2 antibody

Option3

[169]
A 40-year-old woman presents with recurrent episode of vertigo associated with a feeling or 'fullness' and 'pressure' in her
ears. She thinks her hearing is worse during the attacks. Clinical examination is unremarkable.
What is the most likely diagnosis?
🅐 Meniere's disease
🅑 Benign paroxysmal positional vertigo
🅒 Acoustic neuroma
🅓 Cholesteatoma
🅔 Somatisation

Option1

[170]
A 19-year-old woman presented with a gradual development of bilateral hearing loss which was accompanied by nausea
and ringing in the ears but denied otalgia or otorrhoea. She reported a family history of hearing problems but was unsure
what the exact condition was.
Otoscopic examination demonstrated a reddish blush visible on the cochlear promontory beyond an intact tympanic
membrane.
What is the diagnosis in this case?
🅐 Adhesive otitis media
🅑 Cerumen impaction
🅒 Meniere's disease
🅓 Otosclerosis
🅔 Tympanosclerosis

Option4
[171]
A 72-year-old man becomes confused at the end of a charity walk for Alzheimer's. He is brought to the hospital by
paramedics because he repeatedly asked other attendees on the walk why he was there and how he had got there. There is
no past medical history of note and he takes no regular medication. He is orientated in time and person and knows he has
been brought to the hospital. His blood pressure is 123/82 mmHg, his pulse is 70 beats per minute and regular. The
neurological exam is unremarkable, routine blood tests and CT head are normal. He gradually recovers over the course of
3hrs.
What is the most appropriate intervention?
🅐 Aspirin
🅑 Clopidogrel
🅒 Reassurance
🅓 Rivaroxaban
🅔 Warfarin

Option3

[172]
A 40-year-old woman presents with increasing muscle weakness. She reports the weakness mainly affects her shoulders, hips
and upper arms. On questioning, she feels her symptoms are worse after a period of inactivity such as sitting and on repeat
movement/exercise her weakness actually improves. She confirms she has associated symptoms of a dry mouth and
sometimes has difficulty passing urine.
On examination, she has reduced power of the shoulders and upper arms with reduced reflexes throughout. The rest of her
examination including peripheral nerve and cranial nerve exam is normal.
What is directly affected in this patient's most likely condition?
🅐 Chloride channels
🅑 Myelin sheaths
🅒 Nicotinic acetylcholine receptors
🅓 Voltage-gated calcium channels
🅔 Voltage-gated sodium channels

Option4

[173]
A 62-year-old man is seen in the rapid access transient ischaemic attack clinic following three episodes over the past two
weeks of transient left sided weakness.
What is the most appropriate advice to give with regards to driving?
🅐 Cannot drive for 12 months
🅑 Cannot drive until investigations complete
🅒 Inform DVLA but can continue driving
🅓 Cannot drive for 3 months
🅔 Cannot drive for 1 month

Option4
[174]
A 45-year-old woman with known multiple sclerosis for several years has been experiencing worsening muscle stiffness and
difficulty in producing smooth movements. She has been taking baclofen for a year now but has had no effect.
What is the next best medication to help with these symptoms?
🅐 Botox
🅑 Dantrolene
🅒 Diazepam
🅓 Gabapentin
🅔 Tizanidine

Option4

[175]
You review a 62-year-old man three days after he had been admitted with an acute coronary syndrome. His past medical
history includes myasthenia gravis for which he takes long-term prednisolone. Since admission his symptoms of myasthenia
have become markedly worse. In particular you notice bilateral ptosis and slurring of speech.
Which of the following recently started medications is most likely to be responsible?
🅐 Clopidogrel
🅑 Atorvastatin
🅒 Ramipril
🅓 Aspirin
🅔 Bisoprolol

Option5

[176]
A 27-year-old female suffered a crush injury at work where an air vent fell and trapped her arm. This led to fixed focal
dystonia resulting in flexion contracture of the right wrist and digits.
On examination, there is evidence of intrinsic hand muscle wasting. The right forearm is supinated, the wrist hyperextended
and the fingers flexed with a decrease in sensation along the medial aspect of the hand and arm. A reduction in handgrip
strength was noted.
Which nerve roots are affected in this case?
🅐 C5
🅑 C5/C6
🅒 C7
🅓 C8
🅔 C8/T1

Option5
[177]
A 44-year-old man presents to his GP complaining of weakness in his hands and legs and numbness in his feet. He first
noticed some problems with walking in his late teens and reports that he's always been 'clumsy' and will often trip over. He is
otherwise well and takes no regular medications. On examination, he has a high-stepping gait with wasting of the lower legs
and high arches. Power is reduced in all limbs and reflexes are difficult to elicit. There is a reduction in sensation which is
more pronounced distally. Coordination is intact.
What is the most likely diagnosis?
🅐 Duchenne muscular dystrophy
🅑 Cervical spondylotic myelopathy
🅒 Guillain-Barre syndrome
🅓 Subacute combined degeneration of the cord
🅔 Charcot-Marie-Tooth disease

Option5

[178]
A 72-year-old man with a past medical history of hypertension and a previous non-alcoholic fatty liver disease presented to
the emergency department of a tertiary centre hospital at 09:00 am with a dense right-sided hemiparesis.
The patient's wife reports that she had difficulty waking him this morning at 08:00 am and noted a right-sided facial droop
with associated weakness of the right arm. She called an ambulance immediately. Before going to bed last night at 21:00
pm, the patient was well and walking around as normal.
On examination, the patient was found to have a dense hemiparesis affecting the right arm and lower part of the face with
associated increased tone and sensory loss. Plantars were downgoing. The patient was dysarthric and communication was
difficult. Pupils are equal and reactive.
The patient's blood pressure is 154/80 mmHg.
A CT scan was performed.
CT angiogram head: Area of low attenuation in the area of the left middle cerebral artery with mild evidence of swelling in
the left parietal area. Reduced perfusion from the proximal left middle cerebral artery suggestive of a subacute infarct of the
proximal left middle cerebral artery. No evidence of bleed or haemorrhagic transformation
ECG: Atrial fibrillation, no acute ST changes
What is the most appropriate next management?
🅐 Refer to interventional radiology for thrombectomy
🅑 Referral to neurosurgical team for decompressive craniotomy
🅒 Thrombolysis with alteplase
🅓 Start apixaban 5 mg twice daily and manage conservatively
🅔 Aspirin 300 mg once daily and manage conservatively

Option1

[179]
A 29-year-old female presents complaining of double vision and unsteadiness. She has no past medical history of note. On
examination she has limited movement of her eyes in all directions. Pupils are 3 mm, equal and reactive to light. Examination
of the peripheral nervous system is normal other than reduced reflexes and the plantars are down going. Some past-
pointing is also noted.
What is the most likely diagnosis?
🅐 Multiple sclerosis
🅑 Conversion disorder
🅒 Miller Fisher syndrome
🅓 Ataxic telangiectaisa
🅔 Friedreich's ataxia

Option3
[180]
A 27-year-old female presents with lethargy throughout the day where she is able to do her job effectively in the morning
but is extremely tired towards the end of the workday. Serum antibody receptor testing leads to a diagnosis of myasthenia
gravis. She is prescribed pyridostigmine amongst other agents.
What is the mechanism of the aforementioned drug?
🅐 Short acting acetylcholinesterase inhibitor
🅑 Long acting acetylcholinesterase inhibitor
🅒 Muscarinic agonist
🅓 Calcineurin inhibitor
🅔 Dihydrofolic acid reductase inhibitor

Option2

[181]
A 23-year-old woman is diagnosed with absence seizures. She is otherwise normally fit and well. A decision is made to
initiate treatment. She has recently given birth to her first child and is breastfeeding.
What should be recommended with regards to the management of her seizures?
🅐 Prescribe carbamazepine and advise her to continue breastfeeding
🅑 Prescribe carbamazepine and advise her to stop breastfeeding
🅒 Prescribe ethosuximide and advise her to stop breastfeeding
🅓 Prescribe lamotrigine and advise her to continue breastfeeding
🅔 Prescribe lamotrigine and advise her to stop breastfeeding

Option4

[182]
A 25-year-old man presents with a tonic-clonic seizure which self-terminated after 3 minutes. This is the first time he has had
a seizure. There were no obvious precipitants. He does not have a family history of seizures. A neurological examination was
unremarkable.
An MRI brain is requested:
MRI: No structural or vascular abnormalities identified
An EEG is also performed:
EEG: No seizure activity; Unremarkable examination
How long should the patient be advised to not drive?
🅐 12 months
🅑 4 weeks
🅒 6 months
🅓 Indefinitely
🅔 No restriction

Option3
[183]
A 64-year-old man is referred to the neurological clinic with rapidly progressive dementia. Up until two months ago he was
able to carry out most activities of daily life and only had very mild memory issues but now, he has developed features of
advanced dementia.
A diagnosis of Creutzfeldt-Jakob disease (CJD) is suspected after an MRI displays hyperintense signals in the basal ganglia
and thalamus. An EEG performed also shows biphasic, high amplitude sharp waves in keeping with the sporadic form of the
condition.
What is the other most common symptom which characterises this condition?
🅐 Hemi-spatial neglect
🅑 Myoclonus
🅒 Resting tremor
🅓 Urinary incontinence
🅔 Visual hallucinations

Option2

[184]
Which of the following is least associated with Parkinsonism?
🅐 Chlopromazine
🅑 Progressive supranuclear palsy
🅒 Dementia pugilistica
🅓 Lead poisoning
🅔 Wilson's disease

Option4

[185]
A 62-year-old woman is taken to the emergency department by her son, after she reported experiencing sudden-onset
weakness of the legs.
On examination, she has spastic paresis bilaterally of the lower limbs. A full neurological examination is conducted and she
is also found to have loss of pain and temperature sensation of the lower limbs.
Which of the following is the most likely diagnosis?
🅐 Anterior spinal artery syndrome
🅑 Brown-Sequard syndrome
🅒 Multiple sclerosis
🅓 Subacute combined degeneration of the cord
🅔 Syringomyelia

Option1

[186]
A 63-year-old man who is known to have small cell lung carcinoma presents with gradually worsening muscle weakness. This
initially affected his legs but is now spreading to the arms. He also complains of a dry mouth and erectile dysfunction.
Neurological examination show bilateral leg and arm weakness associated with hyporeflexia.
Antibodies to which one of the following are most likely to be responsible for these findings?
🅐 RNA-binding protein Nova-1
🅑 NMDA-receptors
🅒 Muscarinic acetylcholine receptors
🅓 Nicotinic acetylcholine receptors
🅔 Voltage gated calcium channels

Option5
[187]
A 40-year-old man presents with a progressive deterioration in vision over the past 2 weeks. On examination, there is
ophthalmoplegia, his gait is noticeably ataxic and there is a generalised loss of the deep tendon reflexes. He returned from
Turkey two weeks ago where he describes having a simple viral illness involving a sore throat and fever that lasted for
around 1 week and resolved shortly before his return home. He drank more alcohol than normal during the holiday, having
around 3 glasses of wine each night.
What is the cause of his poor vision?
🅐 Guillain-Barre syndrome
🅑 Acute motor axonal neuropathy
🅒 Chronic inflammatory demyelinating polyneuropathy
🅓 Alcoholic polyneuropathy
🅔 Miller-Fisher variant

Option5

[188]
You are reviewing a 22-year-old man who has developed headaches.
Which one of the following features is most typical of migraines?
🅐 Pain on neck flexion
🅑 Phonophobia
🅒 Epiphora
🅓 Recent viral illness
🅔 Bilateral tight-band like pain

Option2

[189]
You are working on the stroke ward. A 69-year-old lady has come in following a sudden onset of dizziness and visual
disturbances which started yesterday morning. She initially thought she was just dehydrated however later realised she was
unable to read her own shopping list. On the ward round the consultant examines her and finds she is indeed unable to
read. She is however, able to write. When she writes a sentence it makes perfect sense, although she is again unable to read
it out. She has no problems with her speech, and is able to converse completely normally. She has no motor focal
neurological deficit. The consultant asks you where the lesion is likely to be.
🅐 Corpus callosum
🅑 Wernicke's area
🅒 Right (non-dominant) parietal lobe
🅓 Left frontal lobe
🅔 Broca's area

Option1

[190]
A 29-year-old woman with a past history of hypothyroidism presents to the surgery complaining of weakness, particularly of
her arms, for the past four months. She has also developed double vision towards the end of the day, despite having a
recent normal examination at the opticians.
What is the most likely diagnosis?
🅐 Lambert Eaton myasthenic syndrome
🅑 Polymyositis
🅒 Polymyalgia rheumatica
🅓 Multiple sclerosis
🅔 Myasthenia gravis

Option5
[191]
A 63-year-old man is prescribed ropinirole for Parkinson's disease.
What is the mechanism of action?
🅐 MAO-B inhibitor
🅑 Antimuscarinic
🅒 Dopamine receptor agonist
🅓 Dopamine receptor antagonist
🅔 Decarboxylase inhibitor

Option3

[192]
Neuropathic pain characteristically responds poorly to opioids.
However, if standard treatment options have failed which opioid is it most appropriate to consider starting?
🅐 Tramadol
🅑 Morphine
🅒 Codeine
🅓 Oxycodone
🅔 Buprenorphine

Option1

[193]
A 5-year-old girl has been referred to a paediatric neurologist with frequent episodes which her parents describe as
'daydreaming'. They report that she would suddenly stare blankly into space during a conversation or activity and would be
unresponsive for approximately 10-15 seconds. These episodes occur several times a day and she has no recollection of the
incidents.
An electroencephalogram (EEG) has been arranged which shows generalised spike-wave activity. Results of blood tests and
neuroimaging are unremarkable.
Her parents are concerned about the long-term outlook of her condition.
Based on the seizure type, what is the most likely prognosis?
🅐 <10% probability of being seizure free in adolescence
🅑 25-30% probability of being seizure free in adolescence
🅒 45-50% probability of being seizure free in adolescence
🅓 70-75% probability of being seizure free in adolescence
🅔 90-95% probability of being seizure free in adolescence

Option5

[194]
A 65-year-old man with a history of Parkinson's disease is referred to the respiratory clinic with shortness of breath. He has
never smoked. Spirometry is performed:
• FEV1: 71%
• FVC: 74%
Which one of the following drugs is most likely to be responsible?
🅐 Levodopa
🅑 Entacapone
🅒 Ropinirole
🅓 Selegiline
🅔 Pergolide

Option5
[195]
A 54-year-old man presents with a persistent tremor. On examination there is 6-8 Hz tremor of the arms which is worse when
his arms are outstretched. His father suffered from a similar complaint.
What is the most suitable first-line treatment?
🅐 Amitriptyline
🅑 Propranolol
🅒 D-penicillamine
🅓 Levodopa
🅔 Diazepam

Option2

[196]
A 67-year-old male undergoes investigations for bilateral paraesthesia in the radial aspects of both hands, over the thumbs
and first fingers. He also has paraesthesia in the lateral aspects of both forearms and lower limb spasticity. Blood tests reveal
a HBA1c of 46 mmol/mol. He undergoes nerve conduction studies and EMG with evidence of denervation.
Which ONE of thefollowing diagnoses is most likely?
🅐 Bilateral carpal tunnel syndrome
🅑 Degenerative cervical myelopathy
🅒 Multiple sclerosis
🅓 Syringomyelia
🅔 Diabetic neuropathy

Option2

[197]
You are a foundation doctor working in general practice. You review a forty-year-old woman complaining of dizziness.
Symptoms are worse when rolling over in bed and are associated with nausea. She has never had any previous episodes,
does not complain of aural fullness and does not have nystagmus.
What could be done to confirm the diagnosis?
🅐 Audiogram
🅑 Rinne and Weber tests
🅒 Otoscopy
🅓 Dix-Hallpike manoeuvre
🅔 Epley manoeuvre

Option4

[198]
A 22-year-old man complains of hearing problems. You perform an examination of his auditorysystem including Rinne's and
Weber's test:
• Rinne's test
Left ear: bone conduction > air conduction
Right ear: air conduction > bone conduction
• Weber's test
Lateralises to the left side
What do these tests imply?
🅐 Normal hearing
🅑 Left conductive deafness
🅒 Right conductive deafness
🅓 Left sensorineural deafness
🅔 Right sensorineural deafness

Option2
[199]
A 25-year-old man is referred to neurology by his GP. Over the last six months he has been experiencing neck pain and
stiffness and arm weakness. He is normally fit and well and denies any history of trauma, bowel or bladder problems or
erectile dysfunction. He drinks five pints of beer per week.
On examination, there is a small area of erythema with central blistering on his right elbow which he had not noticed
previously. Pin-prick sensation is reduced in the C5-6 dermatomes bilaterally. Fine touch, proprioception and vibration
sensation are intact throughout. Forearm flexion is weak bilaterally with reduced biceps tendon reflexes. Forearm extension
is weak on the right with a brisk triceps reflex. He is alert and orientated. His cranial nerves are intact. His coordination and
gait are normal.
What is the most likely diagnosis?
🅐 Friedreich's ataxia
🅑 Multiple sclerosis
🅒 Subacute combined degeneration of the spinal cord
🅓 Syringobulbia
🅔 Syringomyelia

Option5

[200]
A 55-year-old man is diagnosed with amyotrophic lateral sclerosis.
Which one of the following drugs has been shown to confer a survival benefit?
🅐 Rituximab
🅑 Riluzole
🅒 Interferon-beta
🅓 Cyclophosphamide
🅔 Interferon-alpha

Option2

[201]
A 65-year-old woman undergoes an elective laparoscopic cholecystectomy. She reports suffering from post-operative
nausea and vomiting following previous anaesthetics. Her anaesthetist plans to use a combination of antiemetics during the
procedure. They elect to avoid a commonly used antiemetic because the patient's preoperative ECG showed a prolonged
QTc.
Based on the above information, what is the mechanism of action of the antiemetic drug they have chosen to avoid?
🅐 5-HT1 agonist
🅑 5-HT1 antagonist
🅒 5-HT2 antagonist
🅓 5-HT3 agonist
🅔 5-HT3 antagonist

Option5
[202]
A 53-year-old man is referred to the neurology outpatient clinic by his general practitioner with progressive weakness. He
has no past medical history and is not on any medications. He has a family history of myasthenia gravis.
On examination, power is 4+/5 in the upper limbs and 4/5 in the lower limbs. Testing of sensation is normal. Testing of
reflexes is variable, with some limbs demonstrating hyperreflexia and hyporeflexia in different nerve roots in the same limb.
The cranial nerve examination is normal and the plantars are downgoing. Tone is also variable, with some limbs
demonstrating hypotonia and some hypertonia.
Given the likely diagnosis, what treatment if used is likely to confer the greatest prognostic benefit?
🅐 Intravenous immunoglobulin
🅑 Natalizumab
🅒 Non-invasive ventilation (NIV)
🅓 Prednisolone
🅔 Riluzole

Option3

[203]
A 56-year-old man presents to the Diabetes clinic for review. He has a history of poorly-controlled type 2 diabetes mellitus,
which was first diagnosed fifteen years ago, and primary open angle glaucoma. He is taking latanoprost, metformin,
sitagliptin and once-daily insulin glargine. His most recent HbA1c reading was 98 mmol/mol.
At his last clinic review three months previously, he had complained of a burning, painful sensation in both his feet. He had
been started on gabapentin, which he had uptitrated gradually to the maximum licensed dose. In spite of this, he reports no
discernible improvement in the burning sensation in his feet.
What is the most appropriate management option with respect to his neuropathy?
🅐 Continue gabapentin, start duloxetine
🅑 Continue gabapentin, start codeine
🅒 Stop gabapentin, start amitriptyline
🅓 Stop gabapentin, start duloxetine
🅔 Refer to pain management clinic

Option4

[204]
Which one of the following haematological disorders is most associated with gingival hyperplasia?
🅐 Chronic lymphocytic leukaemia
🅑 Myelofibrosis
🅒 Polycythaemia rubra vera
🅓 Haemophilia A
🅔 Acute myeloid leukaemia

Option5
[205]
A worried mother brings in her 13-year-old girl who has been dropping things in the morning due to sudden jerks in her
arms. Her teacher has also reported that in the first hours of class she sometimes seems to 'zone out' and not listen at all. Her
mother is worried that she may have epilepsy, as the child's aunt does. The child has been otherwise developing normally
and is clinically well today.
What is the most likely diagnosis?
🅐 Atonic seizures
🅑 Focal seizures
🅒 Infantile spasms
🅓 Juvenile myoclonic epilepsy
🅔 Lennox-Gastaut syndrome

Option4

[206]
A 45-year-old lady presents with a 2-month history of left-hand weakness. She has no past medical history. On examination,
there is a mild weakness of the left upper and lower limbs with a right sided facial weakness which spares the forehead.
Where is the lesion?
🅐 Right cerebrum
🅑 Left cerebrum
🅒 Right pons
🅓 Left pons
🅔 Cervical spinal cord

Option3

[207]
A 19-year-old presents as she would like to start a combined oral contraceptive pill. During the history she states that in the
past she has had migraine with aura. She asks why the combined oral contraceptive pill is contraindicated.
What is the most appropriate response?
🅐 Theoretical risk of ischaemic stroke
🅑 Significantly increased risk of ischaemic stroke
🅒 Increased frequency of migraines
🅓 Migraine is independent risk factor for venous thromboembolism
🅔 Increased severity of migraines

Option2

[208]
A 52-year-old man with a history of epilepsy is reviewed. Since having his medication change he has experienced a
'numbness' of his hands and feet. On examination he has reduced sensation in a glove-and-stocking distribution associated
with a reduced ankle reflex. He is also noted to have lymphadenopathy in the cervical and inguinal region and some
bleeding around the gums.
Which one of the following medications is he most likely to have been taking?
🅐 Carbamazepine
🅑 Phenytoin
🅒 Topiramate
🅓 Sodium valproate
🅔 Lamotrigine

Option2
[209]
A 43-year-old woman attends the emergency department with acute confusion, left sided leg weakness and behavioural
disturbance. She has a past medical history of multiple sclerosis for which she is taking natalizumab.
She is haemodynamically stable. On examination she is not orientated to time or place and is being aggressive, which her
husband says is completely out of character. She has 3/5 weakness in her left leg. The rest of the examination is
unremarkable.
Her blood tests are:
• Hb: 136 g/L (Female: 115 - 160)
• Platelets: 336 × 10⁹/L (150 - 400)
• WBC: 14.2 × 10⁹/L (4.0 - 11.0)
• Na : 140 mmol/L (135 - 145)
• K : 4.6 mmol/L (3.5 - 5.0)
⁺ 5.2 mmol/L (2.0 - 7.0)
• Urea:

• Creatinine: 106 μmol/L (55 - 120)
• CRP: 78 mg/L (< 5)
Which is the most likely diagnosis?
🅐 Bacterial meningitis
🅑 Autoimmune encephalitis
🅒 Intracranial haemorrhage
🅓 Progressive multifocal leukoencephalopathy
🅔 Toxoplasmosis

Option4

[210]
A 59-year-old man presents following a loss of consciousness while at work. He reports new-onset horizontal diplopia and a
2-week history of severe headache. The headache is worst in the morning and when lying down. It is mild in intensity at the
time of interview and had been gradually worsening over the past few months. Examination reveals left-sided ptosis and the
left eye is abducted, depressed and intorted when the right eye is in the primary position of gaze. There is anisocoria
present, with the left pupil fixed and mydriatic compared to the right. The patient is sent for neuroimaging.
What is the most likely cause of this presentation?
🅐 Demyelination secondary to multiple sclerosis
🅑 Ischaemic neuropathy
🅒 Ophthalmoplegia secondary to migraine
🅓 Subarachnoid haemorrhage
🅔 Uncal herniation

Option5
[211]
A 58-year-old is brought in by ambulance with acute left-sided weakness affecting his face, arm and lower limb. He reports
these symptoms came on suddenly 2 hours ago whilst cleaning his garage.
He has a past medical history of type two diabetes for which he takes metformin. He works as a solicitor and is usually fit and
well.
On examination, heart rate is 80 beats/min, blood pressure 160/80mmHg, oxygen saturations 98% on air, respiratory rate
18/min, temperature 36.5ºC, and blood glucose level 6.5. His pulse is regular, heart sounds normal, chest clear, and
abdomen soft non tender.
Examination of the neurological system reveals reduced power 2/5 in the left upper limb, and power 3/5 in the left lower
limb. Power is 5/5 in the right upper and lower limbs. Sensation is intact throughout.
There is a left-sided facial droop with forehead sparing. Examination of the visual fields reveals a homonymous hemianopia.
The patient is taken urgently for a CT head, which reveals no intracranial haemorrhage.
CT angiography is performed which demonstrates an occlusion of the right proximal middle cerebral artery.
What is the correct definitive management of this patient?
🅐 Anti-hypertensives
🅑 Thrombolysis
🅒 Thrombolysis and thrombectomy
🅓 Aspirin 300 mg and fondaparinux
🅔 Aspirin 300 mg only

Option3

[212]
A 28-year-old woman undergoes a spontaneous vaginal delivery at 37 weeks gestation. Unfortunately, her newborn
develops widespread scalp and skin bruising 12 hours post-delivery. Her past medical history includes gastro-oesophageal
reflux disease, depression, pre-eclampsia, and epilepsy. Her drug history includes aspirin, folic acid, omeprazole, phenytoin,
and sertraline. She has also been on low molecular weight heparin (LMWH) throughout the antenatal period due to a
previous unprovoked deep vein thrombosis.
Which drug was most likely implicated?
🅐 Aspirin
🅑 Low molecular weight heparin (LMWH)
🅒 Omeprazole
🅓 Phenytoin
🅔 Sertraline

Option4

[213]
A 32-year-old nursery teacher complains of progressive hearing loss and tinnitus over the past three months. She denies
aural fullness and recent infections. The patient recalls that her mother was diagnosed with a 'hearing problem' in her thirties.
External examination of the ear is normal. Rinne test: bone conduction is louder than air conduction in both ears. Weber
test: sound localised to the centre of the forehead.
What is the most likely diagnosis?
🅐 Acute labyrinthitis
🅑 Cholesteatoma
🅒 Meniere's disease
🅓 Otosclerosis
🅔 Vestibular schwannoma

Option4
[214]
A 42-year-old woman with relapse-remitting multiple sclerosis (MS) has been suffering from increasingly painful spasms and
cramping in her lower limbs for the past 18 months.
After discussion with her neurologist, it is agreed that this symptom is due to her chronic spasticity in her legs.
She has previously not tried any medication besides simple analgesia to manage this symptom but has now been offered an
oral medication to specifically manage the spasticity.
What is the mechanism of action of this drug?
🅐 Acetylcholinesterase inhibitor
🅑 Dopamine agonist
🅒 Gamma-aminobutyric acid (GABA) agonist
🅓 Glutamate receptor antagonist
🅔 Non-selective monoamine reuptake inhibitor

Option3

[215]
A 24-year-old woman who is 14 weeks pregnant presents with a severe migraine. She has a long history of migraine and
stopped propranolol prophylaxis when she found out she was pregnant. Unfortunately the headache has not responded to
paracetamol 1 g.
What is the most appropriatenext step?
🅐 Ergotamine
🅑 Nasal zolmitriptan
🅒 Ibuprofen 400 mg
🅓 Almotriptan 12.5 mg
🅔 Codeine 30 mg

Option3

[216]
Which one of the following is least associated with Miller-Fisher syndrome?
🅐 Anti-GQ1b antibodies
🅑 Areflexia
🅒 Ataxia
🅓 Postural hypotension
🅔 Ophthalmoplegia

Option4

[217]
A 70-year-old man who presented with repetitive large involuntary movements of his right lower limb and right upper limb
has been diagnosed with hemiballismus.
This affects which part of the brain?
🅐 Medial thalamus and mammilary bodies of the hypothalamus
🅑 Subthalamic nucleus of the basal ganglia
🅒 Amygdala
🅓 Substantia nigra of the basal ganglia
🅔 Striatum (caudate nucleus) of the basal ganglia

Option2
[218]
A 64-year-old man presents to his general practitioner with uncontrolled leg movements associated with paraesthesia in
both lower limbs. He does not experience any pains with this. The symptoms are worse at night but persist up to midday on
occasion. The symptoms are meaning that he is sleeping a lot during the day and has had some days off work as a result. He
has a past medical history of type two diabetes mellitus and depression.
He has a body mass index of 31 kg/m².
Given the most likely diagnosis, which of the following treatment options would be most suitable first line?
🅐 Amitriptyline
🅑 Gabapentin
🅒 Physiotherapy
🅓 Ropinirole
🅔 Nortriptyline

Option4

[219]
Six months ago, a 33-year-old female patient attended her GP with recurrent episodes of unilateral headache with aura. The
GP diagnosed her with migraine and advised the use of paracetamol as needed, and provided a repeat prescription of
sumatriptan.
She has attended her GP surgery again today to discuss the headaches. She explains that sumatriptan and paracetamol
adequately suppressed the migraine symptoms, and so she started taking them both daily, or every other day, in the hope of
preventing the migraines altogether. Over the past couple of months, however, the headaches have been recurring almost-
daily, and sometimes more intensely than prior to starting medication. There are no red flags in the history, nor any visual
disturbances. She continues to take the medications, although they seem to provide no benefit anymore.
Given the likely diagnosis, what is the most appropriate management?
🅐 Add ibuprofen
🅑 Change to subcutaneous sumatriptan
🅒 Stop her medications abruptlly
🅓 Withdraw her medications gradually
🅔 Give high-flow oxygen therapy for 15-20 minutes

Option3

[220]
A 45-year-old woman presents complaining of visual disturbance. Examination reveals a left congruous homonymous
hemianopia. Where is the lesion most likely to be?
🅐 Optic chiasm
🅑 Left occipital cortex
🅒 Right optic tract
🅓 Right occipital cortex
🅔 Left optic tract

Option4

[221]
Which of the following investigations is the most important for diagnosing degenerative cervical myelopathy?
🅐 Nerve conduction studies and EMG
🅑 MRI cervical spine
🅒 CT myelogram
🅓 CT C-spine
🅔 AP and lateral C-spine radiographs

Option2
[222]
A 42-year-old woman with a history of myasthenia gravis is admitted to the Emergency Department. She is currently taking
pyridostigmine but there has been a significant worsening of her symptoms following antibiotic treatment for a chest
infection. On examination she is dyspnoeic and cyanotic with quiet breath sounds in both lungs.
Other than respiratory support, what are the two treatments of choice?
🅐 IV methylprednisolone or plasmapheresis
🅑 IV methylprednisolone or intravenous immunoglobulins
🅒 Plasmapheresis or atropine
🅓 IV methylprednisolone or atropine
🅔 Plasmapheresis or intravenous immunoglobulins

Option5

[223]
A 19-year-old man is diagnosed as having myoclonic seizures. What is the most appropriate first line
antiepileptic?
🅐 Sodium valproate
🅑 Carbamazepine
🅒 Topiramate
🅓 Clonazepam
🅔 Ethosuximide

Option1

[224]
An 18-year-old man presents to the emergency department with fever, headache, photophobia and neck stiffness. He is
started on ceftriaxone and the sepsis 6 bundle is completed within 30 minutes.
The following day the microbiologist calls with blood culture results:
• Peripheral blood culture: Gram-negative diplococcus
What is the most common complication of this condition?
🅐 Focal neurological deficit
🅑 Hydrocephalus
🅒 Seizures
🅓 Sensorineural hearing loss
🅔 Waterhouse-Friderichsen syndrome

Option4

[225]
A 35-year-old man presents to the Emergency Department (ED) with confusion and feeling hot. He has a history of
schizophrenia and was recently started on risperidone. He does not smoke or drink alcohol. On examination, he had
widespread muscle rigidity and was disoriented to time. His vital signs were heart rate 120/min, blood pressure 150/92
mmHg, respiratory rate 20/min and temperature 37.9ºC.
Investigations show myoglobinuria and raised creatine kinase.
In addition to supportive treatments and symptom control, which of the following medications is indicated?
🅐 Sodium bicarbonate
🅑 Flumazenil
🅒 Cyproheptadine
🅓 Propranolol
🅔 Dantrolene

Option5
[226]
A 56-year-old gentleman presents with lower limb stiffness and imbalance. His only past medical history of note is carpal
tunnel syndrome that was diagnosed a year ago on clinical grounds and has been refractory to treatment with splints and
steroid injections.
Which of the following is mostlikely?
🅐 Cauda equina syndrome
🅑 Subacute combined degeneration of the cord
🅒 Degenerative cervical myelopathy
🅓 Parkinsons disease
🅔 Multiple sclerosis

Option3

[227]
A 72-year-old woman presents to the emergency department after suddenly finding it difficult to steady herself when
walking. On examination, her blood pressure is 189/102 mmHg. You observe that her gait is ataxic, and her speech is
suggestive of dysarthria. She has paraesthesia over the left upper and lower limbs along with right-sided ptosis and miosis.
Given her symptoms, which artery has been occluded?
🅐 Left anterior inferior cerebellar artery
🅑 Left posterior cerebral artery
🅒 Left posterior inferior cerebellar artery
🅓 Right anterior inferior cerebellar aretery
🅔 Right posterior inferior cerebellar artery

Option5

[228]
A 34-year-old male rugby player presents to the GP surgery with a 5-day history of sudden onset severe pain in the right
shoulder and upper arm, which has now subsided but followed by a 1-day history of weakness in the shoulder. On
examination, there is reduced power in the deltoid muscle. There is a full range of passive movement of the shoulder joint.
The rest of the neurological examination is normal.
What is the most likely diagnosis?
🅐 Shoulder joint dislocation
🅑 Osteoarthritis
🅒 Adhesive capsulitis
🅓 Brachial neuritis
🅔 Cervical artery dissection

Option4

[229]
A 67-year-old man is reviewed in the neurology clinic due to concerns about increasing clumsiness. Examination reveals an
ataxic gait and increased upper limb tone with cog-wheel rigidity. Blood pressure is 135/80 lying and 95/70 standing.
What is the most likely diagnosis?
🅐 Motor neuron disease
🅑 Progressive supranuclear palsy
🅒 Parkinsons disease
🅓 Multiple sclerosis
🅔 Multiple system atrophy

Option5
[230]
A 65-year-old female is returned to the surgical ward after an uneventful vaginal hysterectomy. She has no significant past
medical history and is not taking any regular medications. She is provided with regular anti-emetics and pain relief as per
routine surgical and anaesthetic postoperative instructions.
Three hours after returning to the surgical ward, she complains of palpitations and worsening dyspnoea.
On examination, she appeared agitated with some accessory muscle use.
An updated set of observations are taken:
• Heart rate: 156 beats per minute
• Blood pressure: 102/65 mmHg
• Oxygen saturations: 98% on 3L/min nasal specs
• Respiratory rate: 24/min
A bedside ECG is performed which reveals polymorphic ventricular tachycardia.
Which of the following postoperative medications is the likely cause of her deterioration?
🅐 Morphine
🅑 Ondansetron
🅒 Metoclopramide
🅓 Diclofenac
🅔 Cyclizine

Option2

[231]
A 35-year-old man is brought into the emergency department after being assaulted and robbed 1 hour ago. He states he
was struck on his head and upper body with a crowbar before being knocked unconscious. He has nausea, right-sided
headache, and severe right forearm pain with deformity but denies chest and abdominal pain. He has a history of diabetes
mellitus and dyslipidemia.
His blood pressure is 168/88 mmHg, heart rate 55 beats/min and respiratory rate 14 breaths/min. During the examination,
the patient becomes obtunded.
What finding is most likely to be seen in this patient?
🅐 Increased jugular venous pressure (JVP)
🅑 Lower limb fasciculations
🅒 Nuchal rigidity
🅓 Petechial rash overlying anterior chest wall
🅔 Unilateral pupillary dilatation

Option5

[232]
A 31-year-old female with a history of epilepsy consults you following an uneventful pregnancy.
Which one of the following drugs would it be safe to continue during breast feeding?
🅐 Phenytoin
🅑 Carbamazepine
🅒 Lamotrigine
🅓 Sodium valproate
🅔 All of the above

Option5
[233]
A 23-year-old man is admitted following the sudden onset of an occipital headache. He initially thought it was a migraine
and delayed presenting to the Emergency Department. On examination GCS is 15/15, neurological examination is
unremarkable but neck stiffness is noted. He is apyrexial and no rash is noted. A CT scan done 7 hours after symptom onset
is normal.
At what time should a lumbar puncture be done?
🅐 Immediately
🅑 2 hours post-onset headache
🅒 4 hours post-onset headache
🅓 12 hours post-onset headache
🅔 24 hours post-onset headache

Option4

[234]
A 32-year-old woman is brought in to the emergency department having a seizure. She has a history of epilepsy and takes
carbamazepine. However, her friend tells you she was at a party drinking alcohol last night and it is unclear if she has taken
her medication today.
She has been having a tonic-clonic seizure despite receiving buccal midazolam at home and 10 mg rectal diazepam in the
ambulance. Once IV access is established IV lorazepam is started in the department. Unfortunately, she continues to seize at
35 minutes and is in status epilepticus.
What would be the most appropriate next step in management?
🅐 IV Carbamazepine
🅑 IV Phenytoin
🅒 IV Propofol
🅓 IV Lamotrigine
🅔 IV Thiamine

Option2

[235]
A 61 year-old man presents to the respiratory clinic with a 2-month history of progressive weakness and shortness of breath.
He finds it difficult to stand from sitting, and struggles climbing stairs. He is an ex-smoker with chronic obstructive pulmonary
disease (COPD). He had a recent exacerbation one month ago for which he was treated by the GP with a course of oral
prednisolone, during which time his weakness transiently improved. On examination you note a left-sided monophonic
wheeze and reduced breath sounds at the left lung base. Blood tests and a chest x-ray are requested.
• Hb:145 g/L
• WCC: 10.9 × 10⁹/l
• Na : 138 mmol/L
• K : 4.3 mmol/L
⁺ 6.8 mmol/L
• Urea:

• Creatinine: 93 mmol/L
• Calcium: 2.62 mmol/L
• Phosphate: 1.44 mmol/L
• Chest x-ray: Hyperexpanded lungs, left lower lobe collapse, bulky left hilum
What is the most likely cause of this patient's weakness?
🅐 Myasthenia gravis
🅑 Steroid induced myopathy
🅒 Lambert-Eaton myasthenic syndrome
🅓 Hypercalcaemia
🅔 Motor neuron disease

Option3
[236]
A 67-year-old male attends the Emergency department with sudden onset dizziness and vomiting, which has been present
for the past 2 hours. He has a background of hypertension and hypercholesterolaemia for which he takes ramipril and
simvastatin. Examination reveals vertical nystagmus and difficulty standing without support.
What is the most appropriate next step in this patient's management?
🅐 Prochlorperazine 12.5 mg IM
🅑 Arrange immediate admission for thrombolysis
🅒 Urgent CT head
🅓 Sumatriptan 50 mg PO
🅔 Perform the Epley manoeuvre

Option3

[237]
A 47-year-old man presents to the Emergency Department with a three day history of severeheadache associated with
vomiting. There is no past medical history of note. On examinationblood pressure is 98/62 mmHg, pulse is 108 bpm and
temperature is 37.0ºC. There is mild neckstiffness and a partial third nerve palsy of the left eye. Blood rests reveal:
• Hb: 14.8 g/dl
• Plt: 373 × 10⁹/l
• WBC: 13.6 × 10⁹/l
• Na : 132 mmol/l
• K : 5.2 mmol/l
⁺ 4.2 mmol/l
• Urea:

• Creatinine: 99 μmol/l
• Free T4: 9 pmol/l (range 10-22)
What is the most likely diagnosis?
🅐 Subarachnoid haemorrhage
🅑 Cavernous sinus thrombosis
🅒 Meningitis
🅓 Pituitary apoplexy
🅔 Lateral sinus thrombosis

Option4

[238]
A 72-year-old man presents with left-sided hemiparesis, aphasia, and a homonymous hemianopia.
Subsequent imaging confirms a large right-sided middle cranial artery territory infarct.
What scoring system can be used to quantify the disability in activities of daily living?
🅐 ABCD2 score
🅑 Barthel index
🅒 Guys hospital score
🅓 NIH Stroke Scale/Score (NIHSS)
🅔 Siriraj stroke score

Option2
[239]
A 13-year-old child with a diagnosis of tuberous sclerosis presents to the clinic with his mother. She explains how her son has
had two episodes of haematuria and bouts of flank pain with nausea and vomiting. Multiple investigations are performed
including an MRI scan which reveals a kidney abnormality.
Which of the following is the most likely cause of the symptoms?
🅐 Horseshoe kidney
🅑 Oncocytoma
🅒 Renal angiomyolipomata
🅓 Renal cell carcinoma
🅔 Renal medullary carcinoma

Option3

[240]
A 23-year-old patient is about to be discharged following an admission to hospital following a seizure. The patient had a
witnessed tonic-clonic seizure lasting four minutes. There were no clear provoking factors. Significantly, the patient denied
proceeding trauma and the use of alcohol or recreational drugs.
All blood tests were unremarkable, including FBC, U+Es, LFTs, glucose, clotting and TFTs. A CT head and MRI head were
both unremarkable, with no structural abnormalities. An EEG was performed which showed no seizure activity.
How long must the patient abstain from driving?
🅐 1 month
🅑 12 months
🅒 3 months
🅓 6 months
🅔 The patient is not restricted from driving

Option4

[241]
A 68-year-old man has been brought to the emergency department after his family noticed a progressive deterioration in his
breathing. He has a past medical history of COPD and is managed with inhalers.
On examination, he has no wheeze but is making minimal respiratory effort. He has wasting of the small muscles of his
hands, fasciculations of the left bicep and calf muscles, increased tone of the upper limbs, hyporeflexia of the biceps and
reduced power in upper and lower limbs.
An arterial blood gas (ABG) on room air demonstrates a mild type 2 respiratory failure. A chest x-ray demonstrates bilateral
diaphragmatic palsy.
What is the best treatment for prolongation of life in this patient?
🅐 Long-term oxygen therapy (LTOT)
🅑 Non-invasive ventilation (NIV)
🅒 Pulmonary rehabilitation
🅓 Pyridostigmine
🅔 Riluzole

Option2
[242]
A 25-year-old female presents 5 days after discharge from hospital following an admission for suspected meningitis. A
lumbar puncture was performed which showed no evidence of infection. Unfortunately she developed a headache 48 hours
after discharge. This has now lasted 3 days and has failed to settle with analgesia.
Which one of the following treatment options should beconsidered?
🅐 Intrathecal steroids
🅑 Repeat lumbar puncture
🅒 Course of oral prednisolone
🅓 Blood patch
🅔 Intravenous fluids on top of oral fluid intake

Option4

[243]
A 67-year-old man is reviewed in the falls clinic. Over the past few months he has sustained a number of falls. His daughter
reports that he is starting to 'shuffle around the house' and has particular problems going up and down stairs. She also notes
that he appears to be confused at times and often forgets his grandchildren's names.
On examination he appears to move and follow commands slowly. There is a resting tremor in the left hand more so than
the right. Some rigidity is also noted when examining his arms. Examination of the cranial nerves is unremarkable other than
a problem following movement in the vertical plane.
What is the most likely diagnosis?
🅐 Parkinsons disease
🅑 Motor neuron disease
🅒 Multiple system atrophy
🅓 Dementia with Lewy bodies
🅔 Progressive supranuclear palsy

Option5

[244]
A 13-year-old boy is brought to the clinic by his concerned parents, who report that over the last month, he has been
experiencing multiple episodes of sudden, brief lapses in awareness. These episodes were typically very brief, lasting only a
few seconds, during which the boy would stare blankly into space and become unresponsive to his surroundings. He is
otherwise well and after the episodes ended, the boy would quickly resume his previous activities, often without any
recollection of the lapse in awareness.
Based on the likely diagnosis, what medication is contraindicated in the patient?
🅐 Carbamazepine
🅑 Ethosuximide
🅒 Lamotrigine
🅓 Levetiracetam
🅔 Sodium valproate

Option1

[245]
A 69-year-old man is diagnosed as having Parkinson's disease.
Which one of the following psychiatric problems is most likely to occur in this patient?
🅐 Tics
🅑 Psychosis
🅒 Mania
🅓 Dementia
🅔 Depression

Option5
[246]
A 68-year-old female is reviewed in the neurology clinic. She sustained an ischaemic stroke three months previously and has
been left with residual disability. She has a past medical history of hypertension and hypercholesterolemia. She takes
amlodipine and clopidogrel and atorvastatin.
On examination, her speech is mostly fluent. Her comprehension is normal but her repetition of phrases is poor. When
asked to repeat the phrases 'bottles and rocks', she is unable to do so. There is no other residual neurological deficit.
What is the correct description of her speech disturbance?
🅐 Conduction dysphasia
🅑 Dysarthria
🅒 Expressive dysphasia
🅓 Global dysphasia
🅔 Receptive dysphasia

Option1

[247]
A 55-year-old man is seen in the neurology clinic with sensory loss.
On examination he has loss of fine touch, joint position sense and temperature sensation bilaterally in the lower limbs,
extending from his knees down, as well as in his fingertips.
His motor function is 5/5 in all four limbs.
His medications include: calcichew, alfacalcidol, sodium bicarbonate and ramipril
Which of the following is most likely to be the cause of these clinical findings?
🅐 Guillain-Barre syndrome
🅑 Lead poisoning
🅒 Polio
🅓 Chronic kidney disease
🅔 Amyotrophic lateral sclerosis

Option4

[248]
You want to prescribe an antiemetic to a 19-year-old female who is having a migraine attack.
Which one of the following medications is most likely to precipitate extrapyramidal side-effects?
🅐 Meptazinol
🅑 Ondansetron
🅒 Domperidone
🅓 Cyclizine
🅔 Metoclopramide

Option5
[249]
A 64-year-old woman presents to her GP with a five-day history of feeling right earache and feeling generally unwell. In the
last day she has developed painful blistering around the ear and her husband has told her that her facial movements appear
different. Her past medical history includes hypertension, for which she takes amlodipine.
On examination, a vesicular rash is noted around the right ear and the patient has a visible right-sided facial palsy. Her
observations are normal and she is apyrexial.
Given the likely diagnosis, which of the following is the most appropriate treatment plan?
🅐 Corticosteroids
🅑 Intravenous aciclovir
🅒 Intravenous aciclovir and intravenous corticosteroids
🅓 Oral aciclovir
🅔 Oral aciclovir and corticosteroids

Option5

[250]
A 58-year-old man is referred to the urgent neurology clinic following an episode of unresponsiveness 2 days ago. He recalls
a sudden onset of disorientation and word-finding difficulty which resolved after 30 minutes. His past medical history
includes hypertension and type 2 diabetes for which he takes ramipril, amlodipine, indapamide, metformin, and gliclazide.
He is an ex-smoker.
On examination, his heart rate is 76bpm and regular with a blood pressure of 135/80 mmHg. Thereis reduced pin-prick
sensation over the hands and feet in a glove and stocking distribution, but the remainder of his neurological examination is
unremarkable.
What is the most appropriate treatment for the secondary prevention of his symptoms?
🅐 Aspirin
🅑 Aspirin and dipyridamole
🅒 Clopidogrel
🅓 Sitagliptin
🅔 Warfarin

Option3

[251]
Which one of the following is least recognised as causing idiopathic intracranial hypertension?
🅐 Oral contraceptive pill
🅑 Tetracycline
🅒 Ciclosporin
🅓 Prednisolone
🅔 Vitamin A

Option3
[252]
A 23-year-old man is brought in by ambulance to the emergency department after having a 5-minute tonic-clonic seizure. He
was previously fit and well with no previous seizures. He does not take any regular medication and drinks 5 units of alcohol
per week.
Blood tests are unremarkable and a plain CT head has been reported as normal. An EEG is performed and also reported as
normal.
A referral to the first seizure clinic has been arranged, but in the meantime, he is worried about the impact on his daily life as
he normally drives a car to his office every day, where he works as an accountant.
What is the appropriate advice to give regarding his driving as per the Driver and Vehicle Licensing Agency (DVLA)
guidelines?
🅐 No driving for 5 years and must notify DLVA
🅑 No driving for 6 months and must notify DLVA
🅒 No driving for 6 months and no need to notify DLVA
🅓 No driving for 12 months and must notify DLVA
🅔 No driving for 12 months and no need to notify DLVA

Option2

[253]
A 34-year-old female presents due to a number of 'funny-dos'. She describes a sensation that hersurroundings are unreal,
'like a dream'. Following this she has been told that she starts to smackher lips, although she has no recollection of doing
this. What is the most likely diagnosis?
🅐 Myoclonic seizure
🅑 Focal aware seizure
🅒 Focal impaired awareness seizure
🅓 Focal to bilateral seizure
🅔 Absence seizure

Option3

[254]
A 33-year-old Indian man presented with a 5-day history of fevers, vomiting and dizziness. He had recently come back from a
trip to Kerala and had been treated with a two-week course of clarithromycin for a bout of prolonged severe sinusitis by his
GP. This was due to the fact that he had been suffering from severe frontal headaches, difficulty sleeping and periorbital
swelling. He has a background of tuberculosis (TB) and is currently on treatment with rifampicin and isoniazid.
On examination, there was some slurring dysarthria and mild coarse nystagmus to the left. His observation shows a
temperate of 38.3ºC, pulse 93 beats per minute, blood pressure120/80 mmHg and oxygen saturation 93% on room air.
What is the most likely diagnosis?
🅐 TB meningitis
🅑 Cerebellar metastases
🅒 Cerebellar haemorrhage
🅓 Cerebellar abscess
🅔 Chronic sinusitis

Option4
[255]
The following drugs commonly exacerbate myasthenia gravis, except:
🅐 Methotrexate
🅑 Gentamicin
🅒 Beta-blockers
🅓 Quinidine
🅔 Penicillamine

Option1

[256]
You review a 70-year-old woman four days after she was admitted with a suspected stroke. Unfortunately she has been left
with right sided sensory loss affecting her arms more than the legs and a right sided homonymous hemianopia. Functionally
she has difficulty dressing her self. Examination of her cranial nerves is unremarkable.
What area is the stroke most likely to have affected?
🅐 Middle cerebral artery
🅑 Lacunar
🅒 Anterior cerebral artery
🅓 Posterior cerebral artery
🅔 Posterior inferior cerebellar artery

Option1

[257]
A 50-year-old man presents to the emergency department with a one-hour history of right-sided weakness and facial droop,
which has now entirely resolved. He has a past medical history of migraines with aura, which started around 8 years ago, and
low mood. He had a similarly brief episode of sudden dizziness and slurred speech six months ago but did not seek medical
attention. His father had similar experiences in his 50s but died in a road traffic accident aged 60.
The patient's CT head is normal. He has an MRI head which shows a number of hyperintense lesions.
What is the most likely cause for his symptoms?
🅐 Early onset Alzheimer's disease
🅑 CADASIL
🅒 Huntington's disease
🅓 Variant Creutzfeldt-Jacob disease
🅔 Atypical migraine

Option2

[258]
A 35-year-old man is referred to a neurology clinic. His partner has noticed that he is having repeated episodes of loss of
awareness during the day. These episodes last for a few seconds and are associated with fluttering of the eyelids. There is no
memory of the episode upon recovery.
The examination is unremarkable at the time of review.
Given the likely diagnosis, what medication should be avoided?
🅐 Carbamazepine
🅑 Ethosuximide
🅒 Lamotrigine
🅓 Levetiracetam
🅔 Sodium valproate

Option1
[259]
A 23-year-old woman undergoes a planned lumbar puncture (LP) as part of neurological investigations for possible multiple
sclerosis. During the consent process, she expresses concern about a post-LP headache.
What is the mechanism of post-LP headaches?
🅐 Vertebral body injury
🅑 Nerve injury
🅒 Bleeding into cerebrospinal fluid
🅓 Leaking cerebrospinal fluid from the dura
🅔 Too much cerebrospinal fluid removed

Option4

[260]
A 73-year-old female with a history of recurrent falls at home and alcohol excess is brought to the Emergency Department
due to episodes of confusion over the past 5 days. Between these episodes she is apparently her normal self. On
examination her GCS is 14/15 and she has nystagmus on left lateral gaze.
What is the most likely diagnosis?
🅐 Subdural haemorrhage
🅑 Subarachnoid haemorrhage
🅒 Meningitis
🅓 Herpes simplex encephalitis
🅔 Alzheimer's disease

Option1

[261]
A 27-year-old man presents with a history of fits consistent with tonic-clonic seizures. What is the most suitable first-line
treatment?
🅐 Gabapentin
🅑 Lamotrigine
🅒 Sodium valproate
🅓 Carbamazepine
🅔 Phenytoin

Option3

[262]
A 60-year-old woman with known peripheral vascular disease and atrial fibrillation presents to the emergency department
with acute onset dizziness and alterations in sensation.
On examination, the patient has ataxia and nystagmus. She has left-sided facial numbness and findings in keeping with
Horner's syndrome on the same side. The patient has a reduced sensation in both the arm and leg on the right side.
What blood vessel occlusion is the most likely cause of this patient's presentation?
🅐 Left anterior cerebral artery
🅑 Left posterior inferior cerebellar artery
🅒 Left superior cerebellar artery
🅓 Right posterior inferior cerebellar artery
🅔 Right superior cerebellar artery

Option2
[263]
An 18-year-old male gives a history of early morning jerking movements of his arm. After a night of heavy drinking and sleep
deprivation, he has a generalised tonic-clonic seizure at 5 am. An EEG reveals generalised spike and wave discharges.
What is the most appropriate choice of anti-epileptic?
🅐 Carbamazepine
🅑 Ethosuximide
🅒 Sodium valproate
🅓 Gabapentin
🅔 Phenytoin

Option3

[264]
A 46-year-old man presented with a sudden onset of lower extremity paraesthesias, with an urge to move his legs when
lying in bed or sitting for long periods. These symptoms worsened in the evening, particularly when he lies in bed trying to
sleep at night. He ended up walking for some time in order to relieve these complaints. He has no past medical history of
chronic kidney or chronic liver disease.
What is the most important blood test given the likely diagnosis?
🅐 Alanine transaminase
🅑 Creatine kinase
🅒 Serum ferritin
🅓 Serum potassium
🅔 Haemoglobin A1c

Option3

[265]
A 20-year-old male patient presents to the GP surgery with a 2-year history of upper back pain, shoulder and arm weakness.
On inspection, you notice that he has an asymmetric smile. On further examination, there is marked weakness and wasting of
the right facial muscles, right trapezius, deltoid and biceps muscles. There is also winging of the right scapula. There is some
weakness and wasting of the same muscle groups on the left side but to a much lesser extent. A type of muscular dystrophy
is suspected.
What is the inheritance pattern of this condition?
🅐 Autosomal dominant
🅑 Autosomal recessive
🅒 X-linked recessive
🅓 X-linked dominant
🅔 Idiopathic

Option1

[266]
A 29-year-old female has just given birth a baby boy who weighs 3.1kg. The baby unfortunately develops bleeding via the
umbilicus, mucous membranes, gastrointestinal tract. The mother has a past medical history of epilepsy.
What anti-epileptic is most likely implicated?
🅐 Sodium valproate
🅑 Carbamazepine
🅒 Lamotrigine
🅓 Levetiracetam
🅔 Phenytoin

Option5
[267]
A 63-year-old female presents with a four-month history of progressive weakness in the lower limbs associated with
numbness. She also complains of feeling tired and lightheaded lately. She has had recent investigation for this and showed
macrocytic anaemia with vitamin B12 deficiency. She is currently awaiting to commence on B12 replacement. Otherwise, she
is normally fit and well and is not on any regular medication.
Neurological examination of the lower limb shows the following:
• Power: 4/5 (both)
• Sensation to coarse touch, pain, temperature and pressure: normal (both)
• Sensation to fine touch and vibration: reduced (both)
• Proprioception: reduced (both)
• Ankle reflex: absent (both)
• Babinski response: upgoing (both)
Which of the following area of the spinal cord is most likely affected in this patient?
🅐 Anterior and dorsal columns
🅑 Anterior and lateral columns
🅒 Lateral horn
🅓 Ventral horn
🅔 Dorsal and lateral columns

Option5

[268]
An 18-year-old man is reviewed in the neurology clinic after a recent attendance to the emergency department with new-
onset seizures.
He described three episodes of sudden, forceful contraction of both arms and legs, each lasting one-to-two seconds with no
loss of consciousness. Two episodes occurred whilst walking and caused a fall; the third was whilst working at a cafe and
caused him to spill a drink on a customer.
He has no other medical history and no regular medication. Clinical examination, vital signs, and routine blood tests are all
within normal ranges.
Which is the appropriate first-line pharmacological treatment for this type of seizure?
🅐 Carbamazepine
🅑 Ethosuximide
🅒 Lamotrigine
🅓 Levetiracetam
🅔 Sodium valproate

Option5

[269]
A 34-year-old woman who presents with confusion, headache and fever is admitted to the Emergency Department. Shortly
after admission she has a seizure. A MRI scan is performed which shows patchy haemorrhagic changes in the temporal lobe.
Given the likely diagnosis, what is thetreatment of choice?
🅐 Supportive treatment + intravenous cefotaxime
🅑 Supportive treatment + intravenous aciclovir
🅒 Supportive treatment + intravenous intravenous amphotericin B
🅓 Supportive treatment alone
🅔 Supportive treatment + intravenous immunoglobulin

Option2
[270]
A 55-year-old man presents with a worsening early morning headache. He has now also developed persistent nausea. A
systematic enquiry was otherwise unremarkable. On examination, you note that he is unable to recognise simple objects
when his eyes are closed. A neurological examination was otherwise unremarkable.
What area of the brain is affected?
🅐 Inferior frontal gyrus
🅑 Lateral geniculate nucleus
🅒 Medial geniculate nucleus
🅓 Parietal lobe
🅔 Temporal lobe

Option4

[271]
A 42-year-old woman presents describing a sudden onset of visual disturbances, including blurred vision. She has also
noticed increasing numbness and tingling in her right arm and leg over the past few days.
On examination, neurological findings revealed weakness, hyperreflexia, and impaired coordination. A diagnosis of possible
multiple sclerosis is suspected and so an MRI is performed demonstrating hyperintense lesions perpendicular to the corpus
callosum (i.e. Dawson fingers).
What MRI sequence has most likely been performed?
🅐 Dynamic contrast-enhanced (DCE) MRI
🅑 MRI FLAIR (fluid-attenuated inversion recovery)
🅒 MRI STIR (short tau inversion recovery)
🅓 T1-weighted (T1W) sequence
🅔 T2-weighted (T2W) sequence

Option2

[272]
A 46-year-old male presented to the emergency department with a 3-week history of 'tightness' and 'stiffness' in his upper
and lower limbs. He reports that he feels as if he has 'pulled a muscle' in his left leg. He has a background medical history of
well-controlled type two diabetes mellitus, multiple sclerosis, hypertension and hypercholesterolaemia.
On examination, he has brisk deep tendon reflexes and hypertonia.
Given the likely diagnosis, what is the first-line treatment?
🅐 Baclofen
🅑 Botulinum toxin injection
🅒 Diazepam
🅓 Physiotherapy
🅔 Pregabalin

Option1
[273]
A 39-year-old lady admitted complaining of severe headache and floppy limbs. She says she has some blurry vision.
She is well known to the cardiology firm due to her post partum cardiomyopathy Previous investigations for for blurry vision
were inconclusive with no particular diagnosis reached.
On examination she has 5/5 power and sensation but tires easily. There is an upgoing left plantar. The patient has a body
mass index of 41 kg/m².
Her ECG shows normal sinus rhythm.
Results from a lumbar puncture are as follows:
• White Cells: 2 cells/L
• Red Cells: 3 cells/L
• Glucose: 75% of plasma level
• Xanthochromia: negative
• Opening pressure: 36 cm H₂O
The on call radiologist has authorised this report:
• CT Head: No acute intracranial bleed. Skull base intact. Chronic changes to right hemisphere
Which is the next most appropriate investigation?
🅐 Ultrasound doppler scan of the carotids
🅑 Autoimmune screen
🅒 CT arterial angiogram
🅓 Echocardiogram
🅔 MRI brain with venogram

Option5

[274]
A 38-year-old woman presents to neurology clinic after being referred by her GP. She has a past medical history of crohn's
disease which is currently well controlled on a vegan diet.
She describes a 2-month history of gradual onset numbness in both feet.
On examination, the patient's gait is ataxic. Romberg's test is positive. There is loss of proprioception and vibration sense to
the mid shin bilaterally. Plantars are upgoing bilaterally, with absent ankle jerks.
You suspect the patient has subacute combined degeneration of the spinal cord.
Which part of the nervous system is affected?
🅐 The posterior and anterior spinocerebellar tracts of the spinal cord
🅑 The cerebellum
🅒 The dorsal column and lateral corticospinal tracts of the spinal cord
🅓 The lateral corticospinal tract of the spinal cord
🅔 The dorsal column and ventral spinothalamic tracts of the spinal cord

Option3
[275]
A 17-year-old male presents to the emergency department (ED) following a head injury during a rugby match.
His parents who witnessed the injury tell you he lost consciousness immediately after the collision for a number of minutes.
He was then alert and himself for a couple of hours. They have now bought him to ED as he became drowsy and was
complaining of a headache.
On examination his Glasgow coma scale (GCS) is 12, the pupils are unequal and there is a clear swelling on the right side of
the head.
Given the likely diagnosis of an intracranial haemorrhage, which vessel has most likely beendamaged?
🅐 Bridging veins
🅑 Carotid artery
🅒 Circle of Willis
🅓 Dural artery
🅔 Middle meningeal artery

Option5

[276]
A 59-year-old man presents with a severe pain deep within his right ear. He feels dizzy and reports that the room 'is
spinning'. Clinical examination shows a partial facial nerve palsy on the right side and vesicular lesions on the anterior two-
thirds of his tongue.
What is the most likely diagnosis?
🅐 Meniere's disease
🅑 Herpes zoster ophthalmicus
🅒 Ramsay Hunt syndrome
🅓 Acoustic neuroma
🅔 Trigeminal neuralgia

Option3

[277]
A right handed 26-year-old health care assistant presents to the first seizure clinic after his girlfriend became worried he was
having fits first thing in the morning. There is no significant past medical history, is adopted, so no family history is available,
and is currently on no medications. He says to the doctor that he was not concerned as he has had jerking movements whilst
falling asleep for years and thought the morning episodes were similar.
They have been happening most mornings for the past year and he feels they become worse when he is tired, stressed or
has consumed alcohol the night before. He does not take any illegal substances and a urine drug screen confirms this. When
on the early shift he has noticed that he is dropping things. His girlfriend shows a video of him with his eyes open, jerking his
upper arms for 10 seconds. There is no associated aura, no incontinence, no tongue-biting, no loss of consciousness and no
post-ictal drowsiness or confusion.
Investigations: blood tests (including glucose, inflammatory markers, electrolytes and liver function), lumbar puncture, CT
and MRI brain are entirely unremarkable.
Which anti-epileptic medication is generally first-line for these type of seizures?
🅐 Carbamazepine
🅑 Ethosuximide
🅒 Lamotrigine
🅓 Lithium
🅔 Sodium valproate

Option5
[278]
A 72-year-old gentleman with a history of hypertension and stroke presents to the emergency department accompanied by
his wife. His wife reported him waking at 7am confused. Repetitive questioning was a prominent feature. He was observed to
be rubbing his hands together during the event. He was able to recognise his wife and correctly reported his address and
date of birth to paramedics. The event lasted approximately 10 minutes. On arrival to the emergency department he a
Glasgow Coma Scale score of 13 (E3 M6 V4). A full neurological examination wasotherwise normal.
What is the most likely diagnosis?
🅐 Epilepsy
🅑 Transient psychogenic amnesia
🅒 Transient global amnesia
🅓 Brain tumour
🅔 Transient ischaemic attack

Option1

[279]
Which one of the following is least associated with the development of chorea?
🅐 Ataxic telangiectaisa
🅑 SLE
🅒 Wilson's disease
🅓 Pregnancy
🅔 Infective endocarditis

Option5

[280]
A 28-year-old woman presents with a headache. For the four weeks, she has noticed an increased frequency of headaches,
occurring nearly every day. She has a history of migraines but explains these headaches feel different. She usually takes
paracetamol and sumatriptan, which she has taken every few days for the past couple of months.
Given the most likely diagnosis, what is the most appropriate management option?
🅐 Discontinue paracetamol and sumatriptan immediately
🅑 Slowly withdraw both paracetamol and sumatriptan
🅒 Withdraw paracetamol first, followed by sumatriptan
🅓 Withdraw sumatriptan first, followed by paracetamol
🅔 Withdraw sumatriptan only

OptionA

[281]
A 30-year-old woman is taken to the emergency department by her wife due to severe agitation and hallucinations. On
further probing, she appears to be having delusions and her thinking is very disordered. Whilst being examined, she
experiences a seizure. An MRI scan of the head demonstrates abnormalities in the frontal and temporal lobes.
The medical team are called to review and, suspecting a rare condition, perform a specialist blood test which later comes
back with the following result:
• Anti-NMDA receptor antibodies: Positive
Given the likely underlying cause, which of the following scans is most important to perform?
🅐 CT scan of the abdomen
🅑 Chest X-ray
🅒 High-resolution CT scan of the chest
🅓 Pelvic ultrasound
🅔 Positron emission tomography (PET) scan of the head

Option4
[282]
A 54-year-old man presents with sudden onset visual loss. A systematic enquiry was otherwise unremarkable. On
examination, you note that he has a loss of vision in the right upper quadrant of the visual field in both eyes.
Where is the lesion?
🅐 Inferior optic radiations in the left temporal lobe
🅑 Inferior optic radiations in the right temporal lobe
🅒 Left optic tract
🅓 Superior optic radiations in the left temporal lobe
🅔 Superior optic radiations in the right temporal lobe

Option1

[283]
Which one of the following is least likely to produce a lymphocytosis in the cerebrospinal fluid?
🅐 Systemic lupus erythematosus
🅑 Guillain-Barre syndrome
🅒 Viral encephalitis
🅓 Partially treated bacterial meningitis
🅔 Behcet's syndrome

Option2

[284]
A 62-year-old man with a history of hypertension and epilepsy is noted to have gingival hyperplasia on examination in the
cardiology clinic.
Which one of the following drugs is most likely to be responsible?
🅐 Sodium valproate
🅑 Lisinopril
🅒 Atorvastatin
🅓 Nifedipine
🅔 Carbamazepine

Option4

[285]
A man is recovering after having an operation to remove a meningioma in his left temporal lobe.
What sort of visual field defect is he at risk of having following the procedure?
🅐 Right inferior homonymous quadrantanopia
🅑 Right superior homonymous quadrantanopia
🅒 Left inferior homonymous quadrantanopia
🅓 Right homonymous hemianopia with macula sparing
🅔 Left superior homonymous quadrantanopia

Option2

[286]
Which one of the following causes of peripheral neuropathy is most associated with an axonal, rather than demyelinating,
pathology?
🅐 Paraprotein neuropathy
🅑 Guillain-Barre syndrome
🅒 Hereditary sensorimotor neuropathies (HSMN) type I
🅓 Amiodarone
🅔 Vasculitis

Option5
[287]
A 23-year-old woman attends a follow-up neurology appointment for focal seizures. Six months previously she was started
on levetiracetam which has been titrated to the maximum dose. Despite this, seizures are uncontrolled and she opts to try an
alternative medication. She has plans to start a family in the next few years but currently uses barrier contraception. She is
otherwise fit and well.
What is the most appropriate alternative medication?
🅐 Carbamazepine
🅑 Ethosuximide
🅒 Lamotrigine
🅓 Oxcarbazepine
🅔 Sodium valproate

Option3

[288]
A 65-year old gentleman with a background of osteoarthritis and previous cervical laminectomy for degenerative cervical
myelopathy presents with a 2-month history of worsening gait instability and urinary urgency.
Which of the following is the most likely explanation for his symptoms?
🅐 Transverse myelitis
🅑 Recurrent degenerative cervical myelopathy
🅒 Multiple sclerosis
🅓 Cauda equina syndrome
🅔 Spinal metastases

Option2

[289]
Which one of the following drugs is used in the management of multiple sclerosis?
🅐 Beta-interferon
🅑 Gamma-interferon
🅒 Infliximab
🅓 Rituximab
🅔 Alpha-interferon

Option1

[290]
A 72-year-old man with a background of diabetes and a previous myocardial infarction presents with abnormal vision.
Examination reveals an inferior homonymous quadrantanopia.
What is the most likely site of the lesion?
🅐 Inferior optic radiation in the parietal lobe
🅑 Inferior optic radiation in the temporal lobe
🅒 Inferior to the optic chiasm
🅓 Superior optic radiation in the parietal lobe
🅔 Superior to the optic chiasm

Option4
[291]
A 23-year-old man with a history of migraine presents for review. His headaches are now occurring about once a week. He
describes unilateral, throbbing headaches that may last over 24 hours. Neurological examination is unremarkable. Other
than a history of asthma he is fit and well.
What is the most suitable therapy to reduce the frequency of migraine attacks?
🅐 Propranolol
🅑 Zolmitriptan
🅒 Topiramate
🅓 Amitriptyline
🅔 Pizotifen

Option3

[292]
A 23-year-old man is rushed to the emergency department following a close-range gunshot wound to the head from a
handgun. The bullet entered the side of the patient's skull, resulting in a small entrance wound with no exit wound.
A CT scan of the head is conducted, confirming isolated penetration and consequent damage to the parietal lobe. He
undergoes neurosurgical intervention to remove the bullet.
What condition may be associated with this lesion?
🅐 Anosmia
🅑 Astereognosis
🅒 Expressive (Broca's) aphasia
🅓 Perseveration
🅔 Wernicke's aphasia

Option2

[293]
Which one of the following is least associated with myotonic dystrophy?
🅐 Dysphagia
🅑 Aortic regurgitation
🅒 Diabetes mellitus
🅓 Testicular atrophy
🅔 Learning difficulties

Option2

[294]
A 19-year-old woman presents to her General Practitioner with a bilateral sensorineural hearing loss. An MRI brain is
requested and it reveals what are likely to be bilateral vestibula rschwannomas.
What neurocutaneous syndrome is the patient very likely to have?
🅐 Sturge Weber syndrome
🅑 Von Hippel Lindau syndrome
🅒 Tuberous sclerosis
🅓 Neurofibromatosis type 1
🅔 Neurofibromatosis type 2

Option5
[295]
A 33-year-old female with multiple sclerosis complains that her vision becomes blurred during a hot bath.
What is this an example of?
🅐 Uhthoff's phenomenon
🅑 Oppenheim's sign
🅒 Werdnig-Hoffman's sign
🅓 Lambert's sign
🅔 Lhermitte's sign

Option1

[296]
A 52-year-old man is reviewed in the neurology clinic. He has been referred due to the development of difficultly in finding
the right words whilst speaking. His comprehension of normal conversation has however remained normal.
Where is the likely lesion?
🅐 Anterior temporal lobe
🅑 Posterior temporal lobe
🅒 Parietal lobe
🅓 Lateral part of frontal lobe (inferior frontal gyrus)
🅔 Medial part of frontal lobe (cingulate gyrus)

Option4

[297]
You are the neurology doctor and have been asked to review a 35-year-old woman who has been involved in a road traffic
collision and sustained a significant head injury.
On your arrival she is asleep, she has some slight twitching movements in her right arm and leg. On waking these
movements become more violent, her right arm and leg repetitively flinging out with varying amplitude.
Given the probable diagnosis, where is the lesion most likely to be?
🅐 Left basal ganglia
🅑 Left motor cortex
🅒 Psychosomatic
🅓 Right basal ganglia
🅔 Right motor cortex

Option1

[298]
A 24-year-old man presents to the emergency department complaining of two weeks of tingling in his feet and difficulty
walking. He has no medical history. He frequently inhales nitrous oxide with his friends. His diet is normal.
On examination, there is impaired proprioception and vibration sense extending to the ankles. The plantar responses are
extensor. The ankle reflexes are absent and the knee reflexes are brisk.
Blood tests:
• Vitamin B12 level: 700 pg/ml (200-900)
What is the likely diagnosis?
🅐 Amyotrophic lateral sclerosis
🅑 Guillain-Barre syndrome
🅒 Pellegra
🅓 Subacute combined degeneration of the spinal cord
🅔 Syphilis infection

Option4
[299]
A 71-year-old woman is presecribed ondansetron to help treat nausea which has not responded to either metoclopramide
or cyclizine.
What is the mechanism of action of ondansetron?
🅐 5-HT3 antagonist
🅑 5-HT2 antagonist
🅒 Dopamine antagonist
🅓 NK1 receptor antagonist
🅔 Antihistamine

Option1

[300]
A 43-year-old woman with multiple sclerosis presents for review. She is having increasing problems with painful involuntary
contractions of the leg muscles.
What is the most appropriate first-line therapy?
🅐 Referral for relaxation therapy
🅑 Baclofen
🅒 Diazepam
🅓 Dantrolene
🅔 Natalizumab

Option2

[301]
A 15-year-old girl and her mother present to the clinic. They describe an episode last week to you. She had been at a
sleepover at her friend's house and studying hard for her school examinations. In the morning, when she returned home,
she was drinking a cup of tea with her mother and her arm suddenly jerked and the tea flew everywhere. She remained
conscious throughout. Her mother also described intermittent 'blank spells' where she notices her daughter staring blankly
for a few seconds before recovering. The girl is otherwise well.
What is the most likely diagnosis?
🅐 Benign rolandic epilepsy
🅑 Juvenile myoclonic epilepsy
🅒 Lennox-Gestaut syndrome
🅓 Typical absence seizures
🅔 West's syndrome

Option2
[302]
You review a 47-year-old man in clinic. He has been referred with difficult to control hypertension despite taking a
combination of lisinopril, indapamide and amlodipine. Whilst examining the patient you note over 15 large cafe-au-lait spots
on his trunk. Some axillary freckling is also present. His blood pressure (despite medication) is 170/94 mmHg and pulse
90/min.
Routine bloods are ordered:
• Hb: 15.2 g/dl
• Platelets: 201 x 109/l
• WBC: 5.2 x 109/l
• Na+: 141 mmol/l
• K+: 4.3 mmol/l
• Urea: 6.1 mmol/l
• Creatinine: 88 μmol/l
Urine dipstick is normal.
Which one of the following investigations is most likely to be diagnostic?
🅐 MRI brain
🅑 24 hr urinary cortisol
🅒 Glucose tolerance test with growth hormone measurements
🅓 Renal arteriography
🅔 24 hr urinary collection of cateholamines

Option5

[303]
A 52-year-old gentleman with no past medical history presents to the emergency department accompanied by his wife. His
wife reported finding him confused earlier in the day. Repetitive questioning was a prominent feature. He was able to
recognise his wife and correctly reported his address and date of birth to paramedics. On arrival to the emergency
department, he had recovered and was back to his normal self. The event lasted approximately 3 hours. A full neurological
examination was normal.
What is the most likely diagnosis?
🅐 Epilepsy
🅑 Transient psychogenic amnesia
🅒 Transient global amnesia
🅓 Brain tumour
🅔 Transient ischaemic attack

Option3

[304]
A 54-year-old male patient has been brought in by ambulance after falling down the stairs. He complains of immediate
onset of back pain and right leg weakness following the fall. On examination, there is weakness of all muscle groups of right
hip and leg. There is loss of proprioception and vibration sense in the distribution of L1-S5 of the right hip and lower limb.
There is loss of pain and temperature sensation in the distribution of L1-S5 of the left hip andlower limb.
What is the diagnosis?
🅐 Anterior cord syndrome
🅑 Central cord syndrome
🅒 Posterior cord syndrome
🅓 Brown-Sequard syndrome
🅔 Cauda equina syndrome

Option4
[305]
A 64-year-old man presents to the emergency department via ambulance. Four hours earlier, he noticed some speech
difficulties and his wife phoned the emergency services. He has a past medical history of hypertension, atrial fibrillation and
hypercholesterolaemia and is on regular atorvastatin and ramipril. Previously, he elected not to be anticoagulated for atrial
fibrillation. He smokes ten cigarettes per day, drinks alcohol occasionally and is the director of a language school.
On examination, he has an expressive dysphasia. There is evidence of right-sided hemiplegia, sensory loss and
homonymous hemianopia.
Urgent CT head and CT angiography were organized, which excluded intracranial haemorrhage and confirmed occlusion of
the proximal anterior circulation.
What is the most appropriate management?
🅐 Apixaban
🅑 Aspirin 300 mg
🅒 Intravenous thrombolysis
🅓 Intravenous thrombolysis and mechanical thrombectomy
🅔 Mechanical thrombectomy

Option4

[306]
A 76-year-old man presents with an 18-month history of neck stiffness. He complains of numbness in his fingers and difficulty
picking up small objects. Recently he has felt increasingly unsteady and had a fall. On examination, he has hyperreflexia in
his arms and legs.
Which of the following is the most appropriate management plan?
🅐 Infliximab
🅑 Cervical decompression surgery
🅒 Steroids
🅓 Carpal tunnel surgery
🅔 Riluzole

Option2

[307]
A 30-year-old female patient with a history of relapsing-remitting multiple sclerosis presents with new-onset double vision.
She reports that in the last week, she has noticed double vision when trying to focus on objects on the left side of her visual
field. She reports no double vision when looking to the right.
On examination, asking the patient to track the examiner's finger and look to the left (i.e. left horizontal conjugate gaze)
elicits double vision, with the patient reporting that images appear 'side by side.' In addition, there is a failure of the right eye
to adduct past the midline, and nystagmus is noted in the left eye. Asking the patient to look to the right elicits no symptoms
or abnormal findings. Asking the patient to converge her eyes on a nearby, midline object elicits no abnormalities and the
patient is able to abduct both eyes.
The lesion that is most likely responsible for this patient's symptoms is located in which part of the nervous system?
🅐 Calcarine sulcus of the occipital lobe
🅑 Lateral area of medulla
🅒 Nucleus of the oculomotor nerve
🅓 Optic nerve
🅔 Paramedian area of midbrain and pons

Option5
[308]
A 71-year-old man is reviewed following an ischaemic stroke. He is known to be intolerant of clopidogrel.
What is the most appropriate therapy to help reduce his chance of having a furtherstroke?
🅐 Aspirin + dipyridamole, stop dipyridamole after 2 years
🅑 Dipyridamole, stop dipyridamole after 2 years
🅒 Aspirin lifelong
🅓 Warfarin
🅔 Aspirin + dipyridamole lifelong

Option5

[309]
A 62-year-old man is admitted to the Emergency Department with a left hemiplegia. His symptoms started around 5 hours
but he initially thought he had slept in an awkward position. He has no past medical history of note but on examination is
found to have and irregular pulse of 150/min. The ECG confirms atrial fibrillation. A CT head is immediately arranged and
reported as normal.
What is the most appropriate initial management?
🅐 Aspirin
🅑 Aspirin + dipyridamole
🅒 Alteplase
🅓 Warfarin
🅔 Aspirin + warfarin

Option1

[310]
A 44-year-old woman presents to her neurologist with complaints of stiffness and involuntary spasms of her muscles. She has
a background of relapsing-remitting multiple sclerosis. She takes natalizumab. She does not smoke or drink alcohol. She
lives with her husband and two children.
On examination, there is evidence of increased tone in the upper limbs in a flexor distribution.
From the list below, what is the appropriate initial choice of medication to relieve her symptoms?
🅐 Dantrolene
🅑 Diazepam
🅒 Gabapentin
🅓 Ropinirole
🅔 Tizanidine

Option3

[311]
A 65-year-old man who is known to have metastatic colorectal cancer presents for review. Since last been seen he reports
being generally stiff and on examination is noted to have diffuse hypertonia.
Which antibodies are most likely to be responsible for this presentation?
🅐 Anti-GAD
🅑 Anti-Ri
🅒 Anti-Hu
🅓 Anti-La
🅔 Anti-Yo

Option1
[312]
A 55-year-old man presents to his GP with visual disturbance.
On examination, the patient is noted to have visual field abnormalities; he is unable to perceive stimuli in the superior
temporal field of his left eye and the superior nasal field of his right eye.
The GP requests an MRI urgently.
Where is a lesion most likely to be found?
🅐 Left parietal lobe
🅑 Left temporal lobe
🅒 Optic chiasm
🅓 Right parietal lobe
🅔 Right temporal lobe

Option5

[313]
Which one of the following dopamine receptor agonists used in the management of Parkinson's disease is least associated
with pulmonary, retroperitoneal and pericardial fibrosis?
🅐 Pergolide
🅑 Lisuride
🅒 Bromocriptine
🅓 Cabergoline
🅔 Ropinirole

Option5

[314]
A 36-year-old woman presents with a one-hour history of a severe headache. You review her within a few minutes of arrival in
the department.
She describes a sudden onset headache and feels it is the worst headache she has ever had in her life. She has felt
nauseated since it started and vomited once and also describes photophobia.
Her GCS is 15, she has no focal neurology and is afebrile.
You suspect a subarachnoid haemorrhage and a CT brain is performed one hour after you rassessment.
The CT shows no acute intracranial pathology.
What is the most appropriate next step in the management of this patient?
🅐 CT angiogram
🅑 Consider an alternative diagnosis
🅒 Digital subtraction angiography
🅓 Lumbar puncture
🅔 Magnetic resonance angiography

Option2
[315]
A 15-year-old boy is referred to neurology for several episodes of jerking motions. One has been captured on video by his
mother.
On review, this shows a single jerking motion of both arms whilst he sits at the kitchen table eating his breakfast. There is no
loss of consciousness.
The mother reports he has never had any other types of unusual movements.
What is the most appropriate management?
🅐 Carbamazepine
🅑 Ethosuximide
🅒 Levetiracetam
🅓 Phenytoin
🅔 Sodium valproate

Option5

[316]
A 44-year-old woman presents to her Neurology clinic due to ongoing headaches. She has been troubled by frontal
headaches for six months. These have proved refractory to analgesia, and she feels they are getting more frequent and
more intense. She is currently taking codeine and paracetamol four times a day; ibuprofen three times a day, and
sumatriptan for particularly severe episodes. Her neurologist suspects that her headaches may be caused by medication
overuse, and suggests that she should try coming off her medications.
What is the most appropriate way in which to withdraw her analgesia?
🅐 Gradually withdraw all analgesia
🅑 Stop all analgesia abruptly
🅒 Stop codeine; gradually wean paracetamol, ibuprofen and sumatriptan
🅓 Stop paracetamol and ibuprofen; gradually wean sumatriptan and codeine
🅔 Stop paracetamol, ibuprofen and sumatriptan; gradually decrease codeine

Option5

[317]
A 45-year-old woman presents with pain associated with muscle spasticity in the context of relapsing-remitting multiple
sclerosis. Baclofen is initiated with the aim of reducing spasticity.
What is the mechanism of action of this medication?
🅐 Gamma-aminobutyric acid (GABA) receptor agonist
🅑 Gamma-aminobutyric acid (GABA) receptor antagonist
🅒 N-methyl-D-aspartate (NMDA) receptor agonist
🅓 N-methyl-D-aspartate (NMDA) receptor antagonist
🅔 Muscarinic M3 receptor antagonist

Option1
[318]
A 32-year-old female presents with difficulty sleeping due to an urge to move her legs constantly when she goes to bed at
night.
She is well otherwise with no previous medical history other than asthma and denies any chance of pregnancy. She has had a
copper coil fitted in last year.
Neurological examination is unremarkable, with normal tone, power, reflexes and sensation bilaterally.
Which of the following is the single most important investigation?
🅐 Serrum ferritin
🅑 Serum folate
🅒 Clotting
🅓 Vitamin D
🅔 Serum B12

Option1

[319]
A 41-year-old female presented to the neurology clinic with a 3-month history of urinary frequency and incontinence. She
has a background medical history of multiple sclerosis, hypertension and hypercholesterolaemia.
What is the next step in her management?
🅐 Doxazosin
🅑 Intermittent self-catheterisation
🅒 Kidneys, ureters, and urinary bladder (KUB) ultrasound
🅓 Oxybutynin
🅔 Pelvic floor muscle exercises

Option3

[320]
A 24-year-old man is seen in the 'First Seizure' clinic. He has been referred by the local the Emergency Department following
a single episode of a witnessed seizure.
Which one of the following factors would be least relevant when deciding whether to start anti-epileptic drugs aftera single
seizure?
🅐 Brain imaging shows a structure abnormality
🅑 The patient has a neurological deficit
🅒 The EEG shows unequivocal epileptic activity
🅓 The patient is less than 45 years old
🅔 The patient considers the risk of having a further seizure unacceptable

Option4

[321]
A 52-year-old woman presents with a two week history of dizziness when she rolls over in bed. She says it feels like the room
is spinning around her. Examination of her ears and cranial nerves is unremarkable.
Given the likely diagnosis of benign paroxysmal positional vertigo what is the mostappropriate management?
🅐 Trial of prochlorperazine
🅑 Request MRI brain
🅒 Advise review by an optician
🅓 Perform Epley manoeuvre
🅔 Trial of cinnarizine

Option4
[322]
A 42-year-old man is seen in the clinic after with progressive spontaneous and continuous movement of his lower limbs. The
patient reports they were first noticed by his partner at night but now have progressed to the point where he has an
uncontrollable urge to move his legs continuously day or night. He has also now developed a crawling, numbing sensation,
again affecting his lower limbs.
Base level investigations are all normal and a programme of leg stretching before sleep is initially recommended.
What first-line medication should be trailed if this patient's condition does not improve?
🅐 Cabergoline
🅑 Pergolide
🅒 Propranolol
🅓 Quinine
🅔 Ropinirole

Option5

[323]
A 29-year-old man presents complaining of visual disturbance. Examination reveals a right superior homonymous
quadrantanopia.
Where is the lesion most likely to be?
🅐 Optic chiasm
🅑 Left temporal lobe
🅒 Right temporal lobe
🅓 Left optic nerve
🅔 Left parietal lobe

Option2

[324]
A 45-year-old woman who works in a children's nursery presents due to 'weakness' in her right foot. This has developed over
the past couple of days. She is otherwise well and reports no other symptoms.
On examination, you note a high-stepping gait on the right side. Weakness of the right foot dorsiflexors is noted. All reflexes
in the lower limb are normal and no fasciculations are noted. Hip abduction strength is normal.
What is the most appropriate management?
🅐 Admit for CT head and lumbar puncture
🅑 Urgent referral to neurology
🅒 Advice to avoid leg crossing, squatting or kneeling and review in 4 weeks
🅓 Advise the patient to increase their vitamin D consumption
🅔 Arrange nerve conduction studies

Option3
[325]
A 35-year-old male patient with a past medical history of adult polycystic kidney disease is admitted to hospital with a
sudden-onset, severe occipital headache associated with nausea and vomiting. He rates the severity of his headache as
10/10 and describes it as 'the worst headache ofhis life.'
Vital signs are notable for a heart rate of 96 beats per minute, blood pressure 135/70 mmHg, respiratory rate of 24 breaths
per minute, and temperature of 37.0ºC.
On examination, his Glasgow Coma Scale (GCS) score is 14/15, pupils are symmetric in size and reactive to light, and there
are no focal neurological deficits.
A non-contrast Computed Tomography (CT) scan of his head reveals the suspected diagnosis.
While awaiting further management in the Emergency Department a few hours after symptom onset, he develops an acute
deterioration in his neurological status with a GCS drop to 11/15. Haemodynamic parameters remain within normal range.
Given the suspected complication in question, what is the most appropriate next step inmanagement?
🅐 Repeat non-contrast CT head
🅑 Transcranial Dopplers
🅒 Lumbar puncture
🅓 MRI head
🅔 ECG

Option1

[326]
A 12-year-old boy is brought to the Emergency Department. He was hit on the side the head by a cricket ball during a
match. His teacher describes him initially collapsing to the ground and complaining of a sore head. After two minutes he got
up, said he felt OK and continued playing. After 30 minutes he suddenly collapsed to the ground and lost consciousness.
What type of injury is he most likely to have sustained?
🅐 Cerebral contusion
🅑 Subarachnoid haemorrhage
🅒 Intraventricular haemorrhage
🅓 Extradural haematoma
🅔 Subdural haematoma

Option4

[327]
A 34-year-old man presents with progressive loss of pain and temperature sensations over his arms and shoulders. He also
complains of weakness in his hands and legs. His wife notes he had burnt his hands on several occasions without realising.
He denies incontinence or peri-analnumbness.
On examination, there is a spastic weakness of the lower limbs and bilateral upgoing plantars. Proprioception and vibration
sensations are preserved. He is usually fit and well.
What is the most likely cause of his symptoms?
🅐 Cervical radiculopathy
🅑 Chronic inflammatory demyelinating polyradiculoneuropathy
🅒 Multiple sclerosis
🅓 Subacute degeneration of the cord
🅔 Syringomyelia

Option5
[328]
A 44-year-old man was admitted to the medical unit with vomiting, drowsiness and headache. On examination he was
febrile, squinting to bright lights and had severe pain when extending his knee when his hip was lifted off the bed. He was
started on IV cefotaxime and IV dexamethasone and underwent a lumbar puncture.
What is the most common long-term complication of this condition?
🅐 Encephalitis
🅑 Hydrocephalus
🅒 Sensorineural hearing loss
🅓 Seizures
🅔 Cerebral abscess

Option3

[329]
A 33-year-old man presents complaining of visual disturbance. Examination reveals a bitemporal hemianopia with
predominately the upper quadrants being affected.
What is the most likely lesion?
🅐 Craniopharyngioma
🅑 Brainstem lesion
🅒 Pituitary macroadenoma
🅓 Frontal lobe lesion
🅔 Right occipital lesion

Option3

[330]
A 34-year-old accountant presents with a one week history of pain around his right eye occurring once or twice a day. They
are described as being very severe and lasting between 10-30 minutes each. He also describes a feeling of a blocked nose.
What is the treatment of choice to treat this current episode?
🅐 Ibuprofen
🅑 Acetazolamide + topical pilocarpine
🅒 Prednisolone
🅓 Subcutaneous sumatriptan
🅔 Ergotamine

Option4

[331]
A 35-year-old woman is seen in the emergency department with right-sided facial weakness. She first noticed the weakness
yesterday morning but did not think it was anything serious and so did not seek medical attention. She has been brought in
today by her friend who is concerned that the weakness has become worse overnight . Her observations are unremarkable.
Cranial nerve exam reveals unilateral weakness of the facial muscles in the distribution of the VII nerve affecting the entire
right side of the face. She has no other focal neurology in the face, arms or legs and the skin is intact with no lesions. Bloods
done on arrival are normal.
Given the underlying diagnosis what is the most appropriate management of this patient?
🅐 CT head
🅑 Aspirin
🅒 Prednisolone
🅓 Aciclovir
🅔 Prednisolone + Aciclovir

Option3
[332]
A 35-year-old woman presents to her GP with three weeks of tingling in her right hand and left foot. She has no significant
past medical history and is on no regular medications. One year previously, she had an episode of visual blurring in her left
eye that lasted for a week before resolving. She did not seek medical attention at the time.
On clinical examination, there was reduced sensation in her right hand and left foot. Flexing her neck elicited paraesthesia in
the upper limbs. Her vision was normal.
Which of the following clinical features is more common early in the course of this disease?
🅐 Cognitive decline
🅑 Difficulty swallowing
🅒 Lethargy
🅓 Muscle contractions
🅔 Speech disturbance

Option3

[333]
A 68-year-old man presents to the emergency department with left-sided arm weakness and a facial droop. He has a
background of hypertension and high cholesterol. His symptoms commenced at 9:30 in the morning. He hoped they would
resolve but they have been getting worse. A CT head shows no acute bleed and an area of infarction, and CT angiogram
confirms occlusion of the proximal anterior circulation. By the time his imaging is complete, it is now 14:30.
What is the most appropriate management at this stage?
🅐 Aspirin
🅑 Clopidogrel
🅒 Edoxaban
🅓 Thrombectomy
🅔 Thrombolysis

OptionD

[334]
A 35-year-old man is reviewed in the neurology outpatient clinic. He was referred by his general practitioner with a four
month history of an insidious onset of abnormal limb movements. He has not significant past medical history and is on no
regular medications.
On examination, there are sudden, brief (< 100 ms) and almost shock-like involuntary single jerks of his face and right upper
limb. The movements are stimulus-sensitive and provoked by touching. The jerks are not continuous and there is some
interruption. Neurological assessment is otherwise unremarkable.
A CT head is organised, which demonstrated a lesion at the left sensori-motor cortex.
This is discussed at the neuroradiology MDT and the appearances are felt to be benign and not requiring surgical
intervention.
Given the likely diagnosis, what is the most appropriate medication choice to control his symptoms?
🅐 Clonazepam
🅑 Lamotrigine
🅒 Levetiracetam
🅓 Topiramate
🅔 Sodium valproate

Option5
[335]
Which one of the following is not a recognised causes of miosis?
🅐 Old age
🅑 Pontine haemorrhage
🅒 Holmes-Ardle pupil
🅓 Argyll-Robertson pupil
🅔 Horner's syndrome

Option3

[336]
A 35-year-old female was referred by her GP for recurrent urinary tract infections. She has taken 3 courses of antibiotics in
the last two months. She is sexually active with her husband and has no vaginal discharge, itch, or genital rash. She has a past
medical history of migraines and multiple sclerosis. On examination, her abdomen was soft and non-tender. Her bladder
was palpable but there was no suprapubic tenderness.
• Temperature 36.5ºC.
• Heart rate 89 bpm.
• Respiratory rate of 17/min.
• Blood pressure 125/85 mmHg.
• Saturations 98% on air.
The urine dipstick was normal on admission.
What is the first-line investigation to assess the cause of her recurrent urinary tract infections?
🅐 CT kidney, ureters and bladder
🅑 Flow cytometry
🅒 MRI brain
🅓 US bladder
🅔 Vaginal swab for microscipy, culture, and sensitivities

Option4

[337]
A 66-year-old woman is investigated for ascites and found to have ovarian cancer. She presents due to 'unsteadiness'. On
examination there is evidence of nystagmus and past-pointing.
Which one of the following antibodies is most likely to be present?
🅐 Anti-Hu
🅑 Anti-Yo
🅒 Anti-Ri
🅓 Anti-Ro
🅔 Anti-La

Option2

[338]
A 39-year-old female who presents with a 3-month history of acute attacks of loss of vision and weakness in the lower limbs is
diagnosed with neuromyelitis optica.
When making a diagnosis of neuromyelitis optica, which of the following is a possible distinguishing feature from
multiplesclerosis?
🅐 Memory loss
🅑 Optic neuritis
🅒 Acute myelitis
🅓 NMO-IgG seropositive
🅔 Brain CT not meeting criteria for multiple sclerosis at disease onset

Option4
[339]
A 63-year-old male patient who has lung cancer with spinal metastases attends the palliative care clinic. He reports persistent
burning pain and numbness in his left leg associated with significant sleep disturbance. His general practitioner started him
on gabapentin 3 months ago and this has had no effect. Neurological examination reveals loss of fine touch, vibration sense,
and proprioception in the left lower limb. His current analgesic medications are regular paracetamol and gabapentin with as
required ibuprofen.
Which of the following is the most appropriate change to his current medications?
🅐 Add amitriptyline
🅑 Add buprenorphine transdermal patch
🅒 Add morphine sulphate
🅓 Switch gabapentin to amitriptyline
🅔 Switch gabapentin to carbamazepine

Option4

[340]
A 33-year-old man presents with a gradual onset loss of feeling in his hands. His past medicalhistory includes alcohol misuse.
Upper and lower limb motor examinations identify bilateral extensor plantar reflexes with absentankle jerks. Sensory
examination identifies reduced sensation to vibration and proprioceptiondistal to the elbows and knees, and reduced light
touch sensation in a glove stocking distribution.
Blood tests:
• Hb: 120 g/L (Male: 135-180)
• Platelets: 160 x 109/L (150 - 400)
• MCV: 110 fL (80 - 100)
• Fasting blood glucose: 4.3 mmol/L (3.9-5.6)
• Serum vitamin B12: 130 ng/L (190-950)
• Serum folate: 2.5 ng/ml (2.7-17.0)
Which affected areas of the nervous system are causing his symptoms?
🅐 Anterior corticospinal tracts and dorsal columns
🅑 Anterior spinocerebellar tracts and dorsal columns
🅒 Anterior spinothalamic tracts and dorsal columns
🅓 Lateral corticospinal tracts and dorsal columns
🅔 Lateral spinothalamic tracts and dorsal columns

Option4

[341]
A 44-year-old male is seen in Neurology clinic after presenting with a subacute history of a headache. His headache seems
to be worse on recumbency and coughing/sneezing.
On examination, upper and lower limbs are normal with intact cranial nerves. On closer examination of the oropharynx,
there is evidence of palatal myoclonus. A CT scan confirms a space occupying lesion.
Where is the lesion likely located?
🅐 Temporal lobe
🅑 Olivary nucleus
🅒 Parietal lobe
🅓 Occipital lobe
🅔 Frontal lobe

Option2
[342]
A 65-year-old lady presents to clinic with a 6-month history of walking difficulty. She offers a more recent history of urinary
urgency. Examination reveals a short, shuffling gait, mild bilateral, postural upper limb tremor, and extensor plantar
response. She scored 20/30 on an MMSE.
What is the most likely diagnosis?
🅐 Idiopathic Parkinson's disease
🅑 Dementia with Lewy bodies
🅒 Parkinson's disease with dementia
🅓 Normal pressure hydrocephalus
🅔 Vascular dementia

Option4

[343]
A 38-year-old woman presented with blurring of her vision on lateral gaze. She has previously suffered an episode pain on
moving her eyes with difficulty in seeing red colours 6 months ago which resolved spontaneously after a week.
She was seen in a neurology clinic and examined by a neurologist. On rightward gaze the left eye failed to adduct and the
right eye exhibited nystagmus. Leftward, upward and downward gazes were unremarkable. The pupils are equal and both
reactive to light.
The peripheral examination was unremarkable.
An MRI brain scan is subsequently performed and the report is awaited.
Where is the lesion most likely located from this presentation?
🅐 Medial longitudinal fasciculus
🅑 Optic chiasm
🅒 Oculomotor nerve
🅓 Optic radiation
🅔 Periventricular area

Option1

[344]
A 42-year-old man attends the clinic with a one-month history of worsening weakness in his right arm and leg, along with
difficulties in maintaining balance. A neurological examination revealed right-sided hemiparesis with increased tone, brisk
deep tendon reflexes, and an up-going plantar reflex. He has a past medical history of multiple sclerosis which is treated
with natalizumab.
An MRI scan is performed:
• MRI head: Multiple areas of demyelination predominantly involving the subcortical white matter and deep grey matter
What pathogen is likely implicated?
🅐 BK virus
🅑 Herpes simplex virus type 1
🅒 Herpes simplex virus type 2
🅓 JC virus
🅔 Varicella-zoster virus

Option4
[345]
A 25-year-old woman presents to the emergency department with a five-day history of fever, malaise, upper respiratory tract
symptoms and painful skin. She then developed a dark red target pattern rash which has spread from the torso and upper
limbs to involve her distal limbs, face, palms and soles.
She has recently been diagnosed with epilepsy and started on anti-convulsant medication.
On examination, there are widespread purpuric macules and confluent areas of erythema. There are scattered blisters and
skin erosions including on her mucous membranes and labia. Gentle lateral pressure causes detachment of the epidermis.
Which medication is the most likely to have caused the presentation?
🅐 Gabapentin
🅑 Lamotrigine
🅒 Levetiracetam
🅓 Sodium valproate
🅔 Topiramate

Option2

[346]
A 55-year-old presents to the emergency department with a rash. He has felt feverish for the preceding three days. His past
medical history includes epilepsy, hypertension, heart failure with preserved ejection fraction and rheumatoid arthritis. His
medications include aspirin, hydralazine, furosemide, lamotrigine and methotrexate.
His observations are heart rate 111 beats per minute, blood pressure 93/55 mmHg, respiratory rate 20/minute, oxygen
saturation 96% on room air and temperature 38.1ºC.
On examination there are widespread blisters on his skin, some of which have burst, leaving painful sores. The mucus
membranes are affected with ulceration of the mouth, throat and genital tract. Approximately 10% of his body surface area is
affected.
Given the likely diagnosis, which of his medications is the likely cause?
🅐 Aspirin
🅑 Furosemide
🅒 Hydralazine
🅓 Lamotrigine
🅔 Methotrexate

Option4

[347]
A 55-year-old man presents to his general practitioner with reports of recurrent episodes of the room spinning. He has no
past medical history. He denies hearing loss or aural fullness. The patient experiences this commonly when he is in bed and
each episode lasts 10-20 seconds. This has been ongoing for months and was preceded by an upper respiratory tract
infection. He is well in between episodes.
The examination is unremarkable.
What is the likely diagnosis?
🅐 Benign paroxysmal positional vertigo
🅑 Meinere's disease
🅒 Multiple sclerosis
🅓 Posterior circulation stroke
🅔 Viral labrinthitis

Option1
[348]
A 58-year-old man presents to the Emergency Department unable to move his right side with slurred speech and a right-
sided facial droop. He tells you that he woke up with the symptoms this morning, and you are able to understand him. He is
usually functionally independent but took early retirement due to bad osteoarthritis, and his daughter checks on him every
day.
On examination, he has complete paralysis of the right side of his face. Power is 3/5 in his right upper and lower limb and
sensation is intact globally. CT Head rules out an intracranial haemorrhage and CT angiography confirms occlusion of the
proximal anterior circulation, with CT perfusion showing limited infarct core volume.
What is the most definitive management?
🅐 Give aspirin 300 mg orally
🅑 Urgent neurosurgery referral
🅒 Intravenous alteplase
🅓 Mechanical thrombectomy
🅔 Give warfarin 5 mg orally

Option4

[349]
A 65-year-old male is admitted with a left sided facial droop. He states that her noticed it when he woke up this morning. He
has no past medical history and takes no regular medicines. On examination he has a left sided facial droop which spares
the forehead muscles.
Blood tests are as follows:
• Hb: 145 g/l
• Platelets: 489 x 109/l
• WBC: 9.2 x 109 /l
• Neuts: 6.4 x 109 /l
• Lymphs: 2.2 x 109 /l
• Glucose: 11.5 mmol/l
• Na+: 136 mmol/l
• K+ : 3.9 mmol/l
• Urea: 6.2 mmol/l
• Creatinine: 42 μmol/l
• CRP: 10 mg/l
What is the most likely cause?
🅐 Bell's palsy
🅑 Acoustic neuroma
🅒 Stroke
🅓 Sarcoidosis
🅔 Diabetes mellitus

Option3
[350]
A 24-year-old man presents to his GP with a tingling sensation in his feet and weakness.
On examination, he has reduced sensation in both feet. He has absent ankle jerks but brisk knee reflexes. Power is 4 out of 5
distally. His gait is ataxic and Romberg's test is positive.
His past medical history consists of anxiety for which he takes sertraline.
He is a smoker of 20 cigarettes per day and drinks 5 units of alcohol a day. Recreationally he uses nitrous oxide and
occasionally cocaine. He also reports regular unprotected sex with sex workers.
What is most likely to be involved with his presenting complaint?
🅐 Treponema pallidum
🅑 Alcohol
🅒 Copper
🅓 Serotonin
🅔 Vitamin B12

Option5

[351]
A 35-year-old man is referred to the neurology outpatient department with one month of progressive weakness. He had a
diarrhoeal illness preceding events. He is from Ghana.
On examination, power is 4/5 at hip flexion and knee extension and this improves to 5/5 after a brief period of exercise. The
knee reflexes are absent. The facial muscles and cranial nerves are normal.
• Creatinine kinase: 420 U/L (40-320)
EMG testing demonstrates an increment in muscle action potentials after exercise.
What is the likely diagnosis?
🅐 Guillain-Barre syndrome
🅑 Inclusion body myositis
🅒 Lamber-Eaton syndrome
🅓 Myasthenia gravis
🅔 Polymyositis

Option3

[352]
A 42-year-old woman presents as she has noticed a 'droop' in the right side of her face since she woke up this morning.
There is no associated limb weakness, dysphagia or visual disturbance. On examination you notice right-sided upper and
lower facial paralysis.
Which one of the following features would be most consistent with a diagnosis of Bell's palsy?
🅐 Vesicular rash around the ear
🅑 Hyperacusis
🅒 Sensory loss over the distribution of the facial nerve
🅓 Pins and needles in the right arm
🅔 Rhinorrhoea

Option2
[353]
A 32-year-old woman is brought to her GP by her husband who is worried about her behaviour. He is concerned that the
patient seems to be mentally 'slowing down' and seems increasingly disorientated and clumsy. The patient has been
diagnosed with multiple sclerosis and was startedon a new medication by her neurologist two months ago. On examination,
she has dysarthria andan ataxic gait. A neurology consultation was sought, and the reactivation of a certain polyomavirus was
found to be the cause of the new symptoms.
What medication had the patient taken that might lead to such a reactivation?
🅐 Beta-interferon
🅑 Glatiramer acetate
🅒 Natalizumab
🅓 Ocrelizumab
🅔 Riluzole

Option3

[354]
A 29-year-old female with progressive dementia and myoclonus is seen in the memory clinic. On examination, the patient
has marked myoclonus with impairment of the concentration and memory aspects of the Addenbrooke's test. An MRI
reveals a 'hockey stick sign'. A few weeks later the patient develops akinetic mutism and paresis of vertical upgaze.
What is the likely diagnosis given the clinical and radiological findings?
🅐 Progressive supranuclear palsy
🅑 Variant Creutzfeldt-Jacob disease
🅒 Lewy body dementia
🅓 Wilson's disease
🅔 Huntington's disease

Option2

[355]
A 41-year-old man presents with a two week history of headaches around the left side of his face associated with watery
eyes. He describes having about two episodes a day each lasting around 30 minutes. On examination he has a red left eye
and a partial left ptosis. There is no past medical history of note other than migraines as a child.
What is the likely diagnosis?
🅐 Atypical migraine
🅑 Cluster headache
🅒 Trigeminal neuralgia
🅓 Acute angle closure glaucoma
🅔 Cavernous sinus thrombosis

Option2

[356]
A 23-year-old woman gives birth to a baby boy. She has a complex past medical history of bipolar disorder, epilepsy and
anti-phospholipid syndrome. She is also taking codeine and naproxen frequently for chronic back pain following a car
accident. When preparing her discharge letter, she informs you that she intends to breastfeed her baby.
Out of her regular medications below, which is the only drug that can be continued whilst breastfeeding?
🅐 Aspirin
🅑 Codeine
🅒 Lamotrigine
🅓 Lithium
🅔 Naproxen

Option3
[357]
A 24-year-old woman is seen in a neurology clinic following an episode where her husband observed her suddenly fall to
the ground and become unresponsive. She reports that she suddenly became stiff and blue, before dramatically shaking for
approximately two minutes. After the shaking stopped, she was drowsy for a further 30 minutes. Prior to this episode she
had been well and had never suffered anything like this before. She has no significant past medical history of note. The
neurologist performed an MRI and EEG, both of which were normal.
What advice regarding driving should be given?
🅐 Stop driving for one year and inform the DVLA
🅑 Stop driving until established on anti-epileptic medication
🅒 Stop driving for 6 months and inform the DVLA in the event of any further seizures
🅓 Stop driving for 6 months and inform the DVLA
🅔 Stop driving for 9 months and inform the DVLA

Option4

[358]
Which one of the following antibodies are associated with painful sensory neuropathy in patients with small cell lung cancer?
🅐 Anti-Ri
🅑 Anti-GAD
🅒 Anti-Ro
🅓 Anti-Hu
🅔 Anti-Yo

Option4

[359]
You see a 23-year-old man who complains of sudden unavoidable urges to sleep. When these occur, he can collapse on the
spot and immediately lose consciousness. This can be brought on by strong emotions such as fear or laughter.
Which of the following neuropeptides is low in this condition?
🅐 GABA
🅑 Orexin
🅒 Cryptochrome
🅓 Melatonin
🅔 Leptin

Option2

[360]
An 80-year-old right-handed woman has recently been admitted with an acute ischaemic stroke affecting part of the left
hemisphere. She is seen in clinic a month later. She is able to understand questions and produce fluent connected speech,
however she frequently substitutes incorrect sounds. When she makes an error she notices but finds it difficult to correct the
error. She is unable to repeat phrases.
What type of aphasia does she have?
🅐 Anomic aphasia
🅑 Broca's aphasia
🅒 Conduction aphasia
🅓 Global aphasia
🅔 Wernicke's aphasia

Option3
[361]
You are a CT1 in Acute Medicine covering a general medical ward at night. You are asked to see a 60-year-old patient with
known myasthenia gravis who is complaining of increased breathlessness and fatigue.
Which of the following antibiotics is most likely to be responsible for a myasthenic crisis?
🅐 Metronidazole
🅑 Doxycycline
🅒 Gentamicin
🅓 Clarithromycin
🅔 Amoxicillin

Option3

[362]
A 39-year-old-male present with morning headaches associated with nausea for the last 2 months and an inability to look
upwards. On examination, while upgaze is diminished bilaterally, downgaze is preserve and it is not corrected by the doll's-
head-manoeuvre. On examination of hispupils, his pupils constrict on accommodation but did not constrict on exposure to
light.
One also notices some mild eyelid retraction bilaterally. Where is the lesion localised to?
🅐 Frontal lobe
🅑 Dorsal midbrain
🅒 Ventral midbrain
🅓 Occipital
🅔 Cerebellar vermis

Option2

[363]
A 21-year-old man attends the clinic with poor balance. He recently moved to the UK from India. He states that he was
diagnosed with a hereditary disorder in early childhood but is unable to recall the name of the condition. On examination,
he is grossly ataxic and has multiple telangiectasia over his body. A systematic examination was otherwise unremarkable.
What gene is likely implicated?
🅐 ACVRL1 gene
🅑 ATM gene
🅒 FXN gene
🅓 SCA1-SCA3 gene
🅔 SOX18 gene

Option2

[364]
A 75-year-old man is brought to the emergency department with sudden onset numbness affecting his face and limbs. He
has a past medical history of hypertension and hypercholesterolaemia. His medications are amlodipine and atorvastatin.
On examination, he is ataxic and there is nystagmus. Right-sided ptosis and miosis are noted. There is an absence of right-
sided facial sensation and left-sided arm and leg sensation. There is no weakness.
What is the likely diagnosis?
🅐 Benedikt syndrome
🅑 Lateral medullary syndrome
🅒 Millard-Gubler syndrome
🅓 Nothnagel syndrome
🅔 Weber syndrome

Option2
[365]
A 27-year-old female presents complaining of generalised weakness. Examination of her face reveals bilateral ptosis,
dysarthric speech and a slow-relaxing grip.
What is the most likely diagnosis?
🅐 Myotonic dystrophy
🅑 Myasthenia gravis
🅒 Multiple sclerosis
🅓 Ataxic telangiectaisa
🅔 Friedreich's ataxia

Option1

[367]
What is the most common clinical pattern seen in motor neuron disease?
🅐 Progressive muscular atrophy
🅑 Progressive bulbar palsy
🅒 Spinocerebellar ataxia
🅓 Relapsing remitting
🅔 Amyotrophic lateral sclerosis

Option5

[367]
A 28-year-old woman presents to the GP with a 3-month history of increased fatigue, which has been particularly worse over
the last 2 weeks. She feels increasingly lethargic throughout the day and is experiencing muscle cramps. Cardiovascular,
respiratory, gastrointestinal and neurological examinations were all within normal limits.
There is no past medical history of note. The patient only reports a previous episode of visual disturbance 6 months ago,
where they were unable to see clearly out of the left eye and red colours were not as pronounced. This symptom resolved
spontaneously within 2 weeks.
Blood were taken and the results showed:
• Hb: 105 g/L (115-160 g/L)
• WCC: 5.6 x109/L (4.0-11.0 x109/L)
• Na+: 140 mmol/L (135-145 mmoL/)
• K+: 3.4 mmol/L (3.5-5.0 mmol/L)
• Urea: 3.7 mmol/L (2.0-7 mmol/L)
• Creatinine: 80 umol/L (55-120 umol/L)
• Vitamin B12: 413 ng/L (200-900 ng/L)
• Folate: 3.1 nmol/L (3.0 nmol/L)
• ESR: 24 mm/hr (0-29 mm/hr)
What is the most likely diagnosis?
🅐 Anaemia
🅑 Fibromyalgia
🅒 Giant cell arteritis
🅓 Multiple sclerosis
🅔 Myasthenia gravis

Option4
[368]
Which type of motor neuron disease carries the worst prognosis?
🅐 Relapsing remitting
🅑 Progressive bulbar palsy
🅒 Progressive muscular atrophy
🅓 Spinocerebellar ataxia
🅔 Amyotrophic lateral sclerosis

Option2

[369]
A 78-year-old man is seen in the movement disorders outpatient clinic. He has a diagnosis of Parkinson's disease and is
currently taking co-beneldopa 650mg five times daily. He is most troubled by drooling of saliva which unfortunately has not
been helped by management strategies provided by speech and language therapy.
What would be the most appropriate initial management plan?
🅐 Start glycopyrronium bromide
🅑 Start atropine inhaler
🅒 Increase co-beneldopa to six times daily
🅓 Botulinum toxin A injection
🅔 Start scopolamine

Option1

[370]
A 75-year-old man has attended the falls clinic with a 1-month history of an increased frequency of falls. In the last month, he
reports falling 5 times. Prior to this, he was independently mobile but now requires the assistance of a frame. He lives with
his wife who says that he has brief episodes of confusion following by lucid periods. His past medical history includes
hypertension, alcoholic fatty liver disease, and gout.
On examination, he has normal power and sensation in his upper limbs. He has a shuffling gait but there is generally good
power in his lower limbs. His cranial nerve examination isu nremarkable aside from being unable to abduct his left eye on
the left lateral gaze.
What is the most likely diagnosis?
🅐 Hepatic encephalopathy
🅑 Lewy body dementia
🅒 Normal-pressure hydrocephalus
🅓 Subdural haematoma
🅔 Wernicke's encephalopathy

Option4

[371]
A 39-year-old man is diagnosed as having cluster headaches. He has received subcutaneous sumatriptan on two occasions
but would like to start medication to help prevent further attacks.
Of the following options, which one is the most suitable treatment?
🅐 Atenolol
🅑 Amitriptyline
🅒 Sodium valproate
🅓 Verapamil
🅔 Gabapentin

Option4
[372]
A 29-year-old female presents complaining of weakness in her arms, leading to increasing difficulties at work. On
examination she has a bilateral ptosis and loss of the red-reflex in both eyes. Urine testing also reveals glycosuria.
What is the most likely diagnosis?
🅐 Myotonic dystrophy
🅑 Homocystinuria
🅒 Multiple sclerosis
🅓 Myasthenia gravis
🅔 HIV

Option1

[373]
A 24-year-old woman presents for advice. Over the past few months she has been having increasing problems with migraine
around the time of menstruation. Her current migraine started around 6 hours ago and has not responded to a combination
of paracetamol and aspirin.
What is the most appropriate next step to relieve her headache?
🅐 Codeine
🅑 Ergotamine
🅒 Sumatriptan
🅓 Venlafexine
🅔 Norethisterone

Option3

[374]
A 68-year-old man presents to his GP with a one-year history of progressive 'slowing'. He describes a general loss of
dexterity, smaller handwriting and his wife has described his face as being 'less expressive'. He has noticed a subtle tremor
in the left hand over the past few months, which is more pronounced when sat listening to the radio.
On examination, he has mask-like facies, mild left-sided bradykinesia and cogwheel rigidity.
Given the likely diagnosis, which finding points towards idiopathic disease rather than a drug-induced disease?
🅐 Age of onset
🅑 Asymmetrical symptoms
🅒 Bradykinesia
🅓 Cogwheel rigidity
🅔 Micrographia

Option2

[375]
A 45-year-old male presents with recent aggressive behaviour, depression, chorea and athetosis. His father had similar
symptoms at the age of 65. A neurodegenerative disorder with trinucleotide repeat expansion is suspected.
Which of the following trinucleotide repeats is most likely present?
🅐 GAA
🅑 GCC
🅒 CGG
🅓 CTG
🅔 CAG

Option5
[376]
A 23-year-old man presents with recurrent headaches. These typically occur 2 or 3 times a month and are characterised by
severe, right-sided headaches which are throbbing in nature and last around 8-12 hours. When he gets the headaches he
finds it hard to carry on working and tends to go and lie down in a dark room. The headaches so far have responded poorly
to paracetamol.
Which one of the following medications should be prescribed to help reduce the frequency of these headaches?
🅐 Ibuprofen
🅑 Pizotifen
🅒 Propranolol
🅓 Zolmitriptan
🅔 Carbamazepine

Option3

[377]
A 44-year-old man presents with a 3-month history of progressive weakness in the upper and lower limbs. On examination,
you note brisk knee reflexes, absent ankle jerks, and extensor plantars. Power is reduced in all muscle groups of the upper
(MRC grade 3/5) and lower (MRC grade 3/5) limbs. A neurological examination was otherwise unremarkable.
What is the most likely diagnosis?
🅐 Amyotrophic lateral sclerosis
🅑 Friedreich's ataxia
🅒 Progressive muscular atrophy
🅓 Subacute combined degeneration of the spinal cord
🅔 Taboparesis

Option1

[378]
A 62-year-old patient presents to the emergency department with severe nausea and vomiting. She describes the onset as
fairly sudden, with no obvious cause. She has no significant past medical history except for hypertension and takes regular
amlodipine for this. On examination, there are no specific findings. As part of initial management, she is given an antiemetic.
Approximately 30 minutes later, she starts to complain of palpitations and light headedness. Anurgent ECG is performed,
which demonstrates polymorphic ventricular tachycardia.
What antiemetic was she most likely to have been given?
🅐 Aprepitant
🅑 Cyclizine
🅒 Hyoscine
🅓 Nabilone
🅔 Ondansetron

Option5
[379]
You are called to see a 62-year-old female inpatient, with a known history of epilepsy, who is having a seizure. The nurse who
witnessed the seizure says it began by affecting her right hand before involving her entire right arm and then progressing to
a loss of consciousness with her entire body shaking.
What is the most likely diagnosis?
🅐 Generalised tonic-clonic
🅑 Focal impaired awareness seizure
🅒 Jacksonian march with secondary generalisation
🅓 Focal aware seizure
🅔 Myoclonic

Option3

[380]
A 23-year-old man is referred to neurology clinic. He describes episodes of leg weakness following bouts of laughing whilst
out with friends. The following weekend his friends described a brief collapse following a similar episode.
What is the most likely diagnosis?
🅐 Strokes-Adams attack
🅑 Cataplexy
🅒 Hypokalaemic periodic paralysis
🅓 Absence seizure
🅔 Myasthenia gravis

Option2

[381]
Each one of the following is associated with ataxic telangiectasia, except:
🅐 Telangiectasia
🅑 Cerebellar ataxia
🅒 Autosominant dominant inheritance
🅓 Recurrent chest infections
🅔 Increased risk of malignancy

Option3

[382]
A 62-year-old man presents with an episode of collapse. He states that the episode of the collapse was preceded by the
worst headache of his life. He denies any recent head injuries. On examination, you note nuchal rigidity and photophobia.
Observations are within normal limits.
A CT brain is arranged (performed 4 hours after the onset of the headache):
• CT brain (non-contrast): No acute intra-cranial abnormality
Which of the following is the most appropriate action?
🅐 CT angiogram
🅑 Consider alternative diagnoses
🅒 Digital subtraction angiogram
🅓 Lumbar puncture
🅔 MRI angiogram

Option2
[383]
A 41-year-old female presents with double vision and difficult walking for the last 1 week. She says she has been falling a lot
the past week and has double visions when looking left or right. She had a cold 4 weeks ago. On examination, one notes a
wide based gait (ataxia), areflexia in the lower limbs and ophthalmoplegia in all directions. There were no other neurological
deficits or systemic symptoms such as weight loss noted. Blood tests which included an autoimmune screen were positive
for Anti-GQ1b antibodies.
What is the most likely diagnosis?
🅐 Meningoencephalitis
🅑 Cerebral venous sinus thrombosis
🅒 Miller-Fisher syndrome
🅓 Vasculitis
🅔 Paraneoplastic syndrome

Option3

[384]
A 58-year-old female presents to general practice complaining of weakness in her legs. She reports increasing difficulty
climbing stairs and getting up from sitting positions.
On taking a history, you note she has no significant past medical history but has smoked for most of her adult life.
On examination, you note a waddling gait, reduced strength in proximal muscles of all limbs and reduced tendon reflexes.
there is no wasting, fasciculations, sensory abnormalities or ocular symptoms. On repetitive movements, her weakness
appears to initially improve.
Given the likely diagnosis, the presence of which of the following antibodies in the blood would be diagnostic?
🅐 Ach receptor antibodies
🅑 Anti-Mi-2 antibodies
🅒 Antinuclear antibody
🅓 Rheumatoid factor
🅔 Voltage-gated calcium-channel antibodies

Option5

[385]
A 45-old-year homeless man is brought to the emergency department as he is found intoxicated on the street. There is no
sign of injury and basic investigations are all normal. On sobering up the patient reports he has been suffering from
progressive weakness mainly in the lower limbs with tingling and numbness in his hands. He also reports issues with his
memory and vision.
On examination, there is generalised weakness and reduced proprioception and vibration sensation in the distal limbs,
worse in the hands than the feet. Romberg's test is negative but Babinski is positive. There are brisk knee reflexes and ankle
jerks are absent.
Given the patient's most likely diagnosis, which spinal pathways are affected?
🅐 Dorsal column and anterior spinothalamic tract
🅑 Dorsal column and lateral spinothalamic tract
🅒 Dorsal column and anterior spinothalamic tract
🅓 Anterior corticospinal tract and lateral spinothalamic tract
🅔 Reticulospinal tract and vestibulospinal tract

Option2
[386]
A 58-year-old man is brought into the emergency department by ambulance. He reports that he has been feeling
progressively weak over the previous few days, initially being unable to climb stairs, which he put down to a recent
diarrhoeal illness. He was forced to call for an ambulance today as he was unable to lift himself from a chair.
Routine biochemistry and haematology investigations come back normal, as does a chest x-ray.An arterial blood gas is
shown below:
• pH: 7.29 (7.35 - 7.45)
• pO2: 10.9 kPa (10.6 - 13)
• pCO2: 7.2 kPa (4.6 - 6.0)
He begins to tire, and is intubated and taken to the intensive care unit. A lumbar puncture shows elevated protein only.
Which of the following is the most appropriate treatment?
🅐 Aciclovir
🅑 Cyclophosphamide
🅒 IV immunoglobulins
🅓 Methylprednisolone
🅔 Plasmapheresis

Option3

[387]
A 30-year-old lady presents with acute occipital headache associated with vomiting, photophobia and stiff neck. There is no
history of rash or fever. She has a past medical history of phaeochromocytoma for which she had surgery. She reports that
her father died of kidney cancer and her brother is blind in his right eye due to a bleed in the eye.
What is the unifying diagnosis?
🅐 Polycystic kidney disease
🅑 Von Hippel Lindau disease
🅒 Multiple endocrine neoplasia type 1
🅓 Tuberous sclerosis
🅔 Alport syndrome

Option2

[388]
A 61-year-old man with a chronic cough and gradual weight loss presents to the clinic with gradually worsening proximal
myopathy over the past few weeks. A hilar mass has been identified on chest x-ray and he is awaiting referral to the
respiratory oncology clinic. Neurological testing confirms weakness which is improved by reinforcement and movement
repetition.
Which of the following autoantibodies is most likely to be found?
🅐 Anti-AChR
🅑 Anti-Hu
🅒 Anti-Ri
🅓 Anti-Yo
🅔 Anti-VGCC

Option5
[389]
A 28-year-old woman is brought by Emergency ambulance to the hospital. She is accompanied by her husband who tells
you that his wife has suffered a progressive flu like illness over the past few days. Over the past 24hrs she has begun to
develop short term memory problems, been behaving oddly, and has now developed worsening drowsiness and
disorientation. On examination her Glasgow Coma Scale is 13. You confirm confused, rambling speech. She has neck
stiffness, fundoscopy is unremarkable.
Investigations
• Hb: 130 g/l
• Platelets: 182 x 109/l
• WBC: 9.2 x 109/l
• Neuts: 2.8 x 109/l
• Lymphs: 5.8 x 109/l
• Eosin: 0.4 x 109/l
• Na+: 134 mmol/l
• K+: 4.7 mmol/l
• Urea: 5.9 mmol/l
• Creatinine: 92 μmol/l
• CRP: 61 mg/l
Which of the following is the most likely underlying diagnosis?
🅐 Herpes encephalitis
🅑 Meningococcal meningitis
🅒 Severe influenza
🅓 Measles encephalitis
🅔 Tuberous sclerosis

Option1

[390]
A 56-year-old-male on haemodialysis via a right internal jugular tunnelled line presented with a 2-week history of fevers,
night sweats and weight loss.
Blood and line cultures grew Staphylococcus aureus and a transesophageal echocardiogram confirmed a vegetation. His
tunnelled line was removed and he dialysed via a temporary femoralline.
After 2 weeks of IV flucloxacillin, he began to get night sweats, fevers and a persistent headache. He became confused and
hypoactive, needing full nursing care now.
What is the most likely cause for his symptoms?
🅐 Myocardial abscess
🅑 Meningitis
🅒 Brain abscess
🅓 Stroke
🅔 Temporary line sepsis

Option3
[391]
A 76-year-old female has come to a transient ischaemic attack (TIA) clinic following a suspected TIA, 2 days ago where her
carer described her expressing distress as she felt she lost some of her vision. Moreover, at the time, she was crying and
being aggressive with her carers.
Her cranial nerve (CN) examination is as follows:
CN I: demonstrates normal smell with the scents used for this test (orange and coffee).
CN II: pupils are equal and reactive to light. Snellen chart examination is 6/12 bilaterally. Visual field deficits found on direct
confrontation are blind spots in the top right quarter in both eyes. No colour blindness is detected.
CN III, IV and VI: no diplopia, nystagmus or gaze palsy detected.
CN V: facial sensation is intact. Muscles of mastication are functioning and a jaw jerk reflex is present.
CN VII: facial movement is intact.
CN VIII: no conductive or sensorineural hearing loss detected.
CN IX and X: no deviation of uvula detected.
CN XI: neck rotation and shrugging movements are intact.
CN XII: no tongue muscle wasting or reduced power detected.
Upper and lower limb neurological examination are unremarkable.
Which lobe of the brain has been affected?
🅐 Left frontal
🅑 Left parietal
🅒 Left temporal
🅓 Right parietal
🅔 Right temporal

Option3

[392]
A 24-year-old female presents with a headache. She has a past history of epilepsy and is known to suffer from migraines, but
has previously managed attacks with a combination of paracetamol andmetoclopramide. This combination is however not
working for the current episode.
What second line medication is it most appropriate to use?
🅐 Codeine + paracetamol
🅑 Pizotifen
🅒 Zolmitriptan + paracetamol
🅓 Methylsergide
🅔 Ergotamine

Option3
[393]
A 54-year-old female presents to the GP with fatigue. Her GP takes some blood tests which are included below.
• Hb: 104 g/L (Female: 115 - 160)
• Platelets: 198 x 109/L (150 - 400)
• WBC: 7.0 x 109/L (4.0 - 11.0)
• Ferritin: 80 ng/mL (20 - 230)
• Folate: 2.2 nmol/L (> 3.0)
As a consequence, she is started on folic acid supplementation.
A few weeks later she presents to the emergency department with tingling and weakness in herlower limbs.
What pattern of deficits would you expect to find?
🅐 Loss of pain and temperature sensation, muscle weakness and hyperreflexia
🅑 Loss of propioception and vibration sensation, muscle weakness and hyperreflexia
🅒 Loss of propioception and vibration sensation, muscle weakness and hyporeflexia
🅓 Muscle weakness and hyperreflexia without sensory changes
🅔 Muscle weakness and hyporeflexia without sensory changes

Option2

[394]
A 65-year-old female is admitted with a right hemiparesis. Examination reveals she is in atrial fibrillation. CT confirms an
ischaemic stroke and aspirin 300mg is commenced.
If the patient is well and develops no new problems at what point should warfarin be started?
🅐 After 14 days
🅑 Immediately
🅒 After 7 days
🅓 Following a repeat CT at 24 days to exclude secondary haemorrhage
🅔 Following a repeat CT at 14 days to exclude secondary haemorrhage

Option1

[395]
A 49-year-old man is prescribed procyclidine for Parkinson's disease. What is the mechanism of action?
🅐 Antimuscarinic
🅑 Dopamine receptor agonist
🅒 Decarboxylase inhibitor
🅓 Dopamine receptor antagonist
🅔 Monoamine oxidase-B inhibitor

Option1

[396]
A 20-year-old man presents with recent episodes of severe, stabbing pain in the right eye. These episodes typically occur
once a day and last around 30 minutes. His wife reports him pacing around and shouting with the pain. She also reports that
his right eye appears red and that he has clear nasal discharge during the episodes.
Given the likely diagnosis, which one of the following should you advise the patient to avoid to help prevent further
episodes?
🅐 Opoid medications including codeine
🅑 Stress
🅒 Alcohol
🅓 Sunlight
🅔 Excessive exercise

Option3
[397]
A 71-year-old man is admitted to the Emergency Department. His family report that since yesterday he has been very
'clumsy' and unsteady on his feet. This morning he started to complain of numbness down his left side. On examination you
notice that he has a right-sided Horner's syndrome and horizontal nystagmus. Examination of the peripheral nervous system
confirms the sensory loss on the left side.
Where is the lesion most likely to be?
🅐 Lateral sinus thrombosis
🅑 Posterior cerebral artery
🅒 Posterior inferior cerebellar artery
🅓 Middle cerebral artery
🅔 Anterior inferior cerebellar artery

Option3

[398]
A 54-year-old man is admitted to the Emergency Department with a left hemiplegia. His symptoms started around 5 hours
ago and he has had no headache, visual disturbance or loss of consciousness. On examination a dense left hemiplegia is
noted. Blood pressure is 120/78 mmHg, GCS is 15/15 and pupils are equal and reactive to light. An urgent CT scan is
performed shortly after his arrival. This demonstrates no abnormality.
What is the most appropriate initialmanagement?
🅐 Enoxaparin
🅑 Alteplase
🅒 Dexamethasone
🅓 Warfarin
🅔 Aspirin

Option5

[399]
Your next patient is a 37-year-old man who has presented with a 12 hour history of gradual visual loss in the right eye.
He states his vision was normal when he woke this morning but over the course of the day noticed his vision becoming hazy.
For the last hour he has had complete loss of vision but is able to distinguish light and dark.
On examination, visual acuity is 6/6 in the left eye, but he is only able to distinguish gross hand movements in the right eye.
Ophthalmoscopy shows bleeding within the vitreous cavity.
Past medical history includes a small subarachnoid haemorrhage four years ago and left adrenalectomy for
phaeochromocytoma.
He takes doxazosin 4mg and simvastatin 40mg once daily. He has no allergies.
When asking about family history, he explains that he was adopted as a baby and so is unsure if there are any illnesses that
run in the family.
What is the most likely diagnosis that would explain the above findings?
🅐 Neurofibromatosis type I
🅑 Neurofibromatosis type II
🅒 Sturge-Weber syndrome
🅓 Tuberous sclerosis
🅔 Von Hippel Lindau disease

Option5
[400]
Which one of the following is least associated with normal pressure hydrocephalus?
🅐 Papilloedema
🅑 Dementia
🅒 Urinary incontinence
🅓 Gait abnormality
🅔 Enlarged fourth ventricle

Option1

[401]
A 45-year-old man presents with dizziness and right-sided hearing loss.
Which one of the following tests would most likely indicate an acoustic neuroma?
🅐 Jerky nystagmus
🅑 Left homonymous hemianopia
🅒 Tongue deviated to the left
🅓 Fasciculation of the tongue
🅔 Absent corneal reflex

Option5

[402]
A 22-year-old man was seen in the Emergency Department after his mother reported three separate seizure episodes. He
was unable to remember what happened and was unaware anything was happening at the time. His mother describes the
episodes lasting 30 seconds each and no obvious trigger. She reported that he lost consciousness and began chewing and
licking his top lip, but did not have any limb movements or incontinence. She also said that after each episode he struggled
to 'find his words' for at least a minute.
What type of seizure and localising lobe does the above description correspond most to?
🅐 Focal awareness impaired, parietal lobe
🅑 Focal aware, frontal lobe
🅒 Focal awareness impaired, frontal lobe
🅓 Focal awareness impaired, temporal lobe
🅔 Focal aware, temporal lobe

Option4

[403]
Which of the following statements is most accurate regarding the usefulness of cervical spine radiographs (X-rays) in the
assessment of degenerative cervical myelopathy (DCM)?
🅐 Cervical spine radiographs should be obtained in all patients suspected of having DCM
🅑 Where DCM is suspected, AP (anteroposterior), lateral and oblique cervical spine radiographs should be requested
🅒 Cervical spine radiographs are a useful first line investigation where a diagnosis of DCMis suspected
🅓 Cervical spine radiographs have a low sensitivity but high specificity for DCM
🅔 Cervical spine radiographs cannot diagnose DCM

Option5
[404]
A 5-year-old boy is diagnosed as having absence seizures. What is the chance he will be seizure free by the age of 16-years-
old?
🅐 5-10%
🅑 20-25%
🅒 40-45%
🅓 65-70%
🅔 90-95%

Option5

[405]
Which one of the following medications is most useful for helping to prevent attacks of Meniere's disease?
🅐 Promethazine
🅑 Prochlorperazine
🅒 Betahistine
🅓 Chlorphenamine
🅔 Cinnarizine

Option3

[406]
A 49-year-old man presents to the Emergency Department complaining of visual disturbance. Examination reveals a right
incongruous homonymous hemianopia.
Where is the lesion most likelyto be?
🅐 Left optic tract
🅑 Left optic radiation
🅒 Right optic tract
🅓 Right optic radiation
🅔 Optic chiasm

Option1

[407]
A 67-year-old female with stable rheumatoid arthritis attends a clinic for her annual review. She reports that she has noticed a
loss of sensation in both her hands over the last 4 months and that it has been getting worse. In addition to arthritis, she also
has hypertension and diabetes. She is currently taking infliximab, metformin, glipizide, and amlodipine. Her last HbA1c was
53 mmol/mol.
On examination, you note bilateral loss of temperature and pain sensation on the medial aspect of both hands and elbows.
Proprioception and vibration sensation are preserved.
What is the most likely diagnosis?
🅐 Amyotrophy
🅑 Diabetic neuropathy
🅒 Mononeuritis multiplex
🅓 Syringomyelia
🅔 B12 deficiency

Option4
[408]
A 26-year-old is referred to the neurology clinic having presented with progressive weakness mainly of the upper limbs,
shoulders and hip girdle. The weakness is more prominent on his left side. He has also developed some associated facial
weakness in particular affecting his ability to smile and close his eyes. He has no problems with eye movement, swallowing
or cognition.
Investigation reveals tortuosity of the retinal arterioles and a reduction in high-frequency hearing.
On review of the patient's family history, his father suffered from the same issues but his mother and sister have no similar
symptoms.
What condition has the patient most likely presented with?
🅐 Amyotrophic lateral sclerosis
🅑 Becker muscular dystrophy
🅒 Facioscapulohumeral muscular dystrophy
🅓 Myasthenia gravis
🅔 Myotonic dystrophy

Option3

[409]
A 55-year-old man presents complaining of visual disturbance. Examination reveals a right congruous homonymous
hemianopia with macula sparing.
Where is the lesion most likely to be?
🅐 Right optic nerve
🅑 Right optic radiation
🅒 Left optic tract
🅓 Left occipital cortex
🅔 Optic chiasm

Option4

[410]
Which one of the following anti-epileptic drugs is most likely to cause visual field defects?
🅐 Lamotrigine
🅑 Phenytoin
🅒 Ethosuximide
🅓 Vigabatrin
🅔 Pregabalin

Option4

[411]
A 35-year-old man with a history of schizophrenia is brought to the Emergency Department by worried friends due to
drowsiness. On examination he is generally rigid. A diagnosis of neuroleptic malignant syndrome is suspected.
Each one of the following is a feature of neuroleptic malignantsyndrome, except:
🅐 Renal failure
🅑 Pyrexia
🅒 Elevated creatine kinase
🅓 Usually occurs after prolonged treatment
🅔 Tachycardia

Option4
[412]
Whilst on the acute medicine unit ward round a 30-year-old woman with known relapsing-remitting multiple sclerosis, who is
currently suffering from profuse bloody diarrhoea secondary to isolated shigella toxin, states that she feels dizzy and has
blurry vision which is worse when looking to the right. She also reports a painful left eye.
Upon ocular examination: external eye examination is normal. Visual acuity is normal on right, but says her left eye can only
make out blurry shapes. Both pupils are normal size, however the right reacts quickly to your pen-torch and the left slowly
with an associated relative afferent pupillary defect. Visual field examination was normal. When testing the extraocular
muscles the left eye was slow at adducting and a couple of beats of horizontal jerk nystagmus were noted on the right with
abduction. On fundoscopy the right was normal and on the left disc pallor was seen.
Where on an MRI head would you see evidence of demyelination in this young woman?
🅐 Left parietal lobe
🅑 Left paramedian area of the midbrain and pons
🅒 Optic chiasm
🅓 Right occipital cortex
🅔 Right paramedian area of the midbrain and pons

Option2

[413]
A 38-year-old man is referred to the neurology outpatient department by his general practitioner with reports of a recurrent
sense of dejà vu. He has no significant past medical history otherwise.
During the course of the examination, he appears to start smacking his lips. This occurs for approximately sixty seconds.
After the event, he has transient difficulty with fluently expressing words, which resolves without intervention.
Given the likely diagnosis, to which part of the brain can the pathology be localized?
🅐 Frontal lobe
🅑 Occipital lobe
🅒 Parietal lobe
🅓 Temporal lobe
🅔 Whole brain

Option4

[414]
A 72-year-old man who is being treated for Parkinson's disease is reviewed.
Which one of the following features should prompt you to consider an alternative diagnosis?
🅐 Micrographia
🅑 Impaired olfaction
🅒 REM sleep behaviour disorder
🅓 Diplopia
🅔 Psychosis

Option4

[415]
A 54-year-old man with small cell lung cancer complains of muscle weakness.
Each one of the following are features of Lambert-Eaton syndrome, except:
🅐 Proximal muscles more commonly affected
🅑 Hyporeflexia
🅒 Dry mouth
🅓 Repeated muscle contractions lead to decreased muscle strength
🅔 Impotence

Option4
[416]
A 74-year-old male patient is brought in to hospital by ambulance with right-sided weakness and difficulty speaking. His wife
reports that he was well the previous night and awoke with these symptoms one hour ago.
On initial assessment, he is noted to have an expressive aphasia, right-sided facial droop, and right-sided weakness most
prominent in the upper extremity.
A non-contrast CT head does not reveal any intracranial haemorrhage. A CT angiogram reveals a clot in the first branch (M1)
of the left middle cerebral artery (MCA). The patient does not have a history of previous intracranial haemorrhage,
intracranial neoplasm, prior stroke or traumatic brain injury, or recent bleeding from the gastrointestinal tract.
Which of the following interventions is most likely to be of benefit in this patient?
🅐 Thrombolysis
🅑 Mechanical clot retrieval
🅒 Treatment-dose heparin
🅓 Mechanical clot retrieval + thrombolysis
🅔 Dual antiplatelet therapy

Option2

[417]
An 18-year-old woman presents for a routine neurology review. She has a longstanding history of seizures, occurring several
times per day. During these episodes, she completely stops what she is doing and becomes unresponsive to voice or tactile
stimulation. Her eyelids rapidly flutter for approximately 20 seconds, before the episode finishes. She is able to quickly
resume activity after this.
She has trialled a number of different medications over the years, with varying degrees of success.
What medication is likely to have been avoided throughout?
🅐 Carbamazepine
🅑 Ethosuximide
🅒 Lamotrigine
🅓 Levetiracetam
🅔 Sodium valproate

Option1

[418]
A 34-year-old man is reviewed in the neurology clinic. He has been established on sodium valproate for primary generalised
epilepsy. Despite now taking a therapeutic dose he continues to have seizures and is troubled by weight gain since starting
sodium valproate. He asks to stop his current medication and try a different drug.
Which one of the following drugs would be the most appropriate second-line treatment?
🅐 Lamotrigine
🅑 Ethosuximide
🅒 Pregabalin
🅓 Gabapentin
🅔 Tiagabine

Option1
[419]
A 64-year-old woman presents with migraine and her daughter says that she noticed that when she asked her mum to
repeat a phrase she said she could not.
The patient has been having migraines for years and they have always been accompanied by a visual aura of flashing lights
and nil else. There are no focal neurological signs and CT brain is unremarkable.
Upon further questioning, the patient can follow simple and complex commands (open eyes and touch left hand to right ear)
and when asked can name 4 animals in a minute. She is also fluent and understandable when asking her general questions.
However, when the patient is asked to repeat 3 simple words and a sentence she cannot.
What is this type of dysphasia?
🅐 Anomic dysphasia
🅑 Broca's dysphasia
🅒 Conductive dysphasia
🅓 Transcortical motor dysphasia
🅔 Wernicke's dysphasia

Option3

[420]
Which of the following features is least likely to be found in a patient with tuberous sclerosis?
🅐 Adenoma sebaceum
🅑 Café-au-lait spots
🅒 Retinal hamartomas
🅓 Ash-leaf spots
🅔 Lisch nodules

Option5

[421]
A 51-year-old man presented to the Emergency Department with painless loss of vision in his right eye. This had been
preceded by episodes of floaters.
His past medical history included well-controlled type 2 diabetes mellitus, for which he was not on any medications, and a
period of poorly-controlled hypertension five years ago, which had culminated in him requiring surgery.
He recalls that his father had died from kidney cancer in his sixties, but is unable to provide any further past medical history.
What is the most likely unifying diagnosis that explains this presentation?
🅐 Hereditary haemorrhage telangiectasia
🅑 Multiple endocrine neoplasia type 2 (MEN2)
🅒 Neurofibromatosis type 1
🅓 Neuromyelitis optica
🅔 Von Hippel-Lindau disease

Option5
[422]
An 81-year-old woman was admitted eight days ago to the hospital with right-sided weakness. After a normal CT head, she
was managed with high-dose aspirin for a suspected stroke which was subsequently confirmed on MRI.
She is now medically optimised for discharge. The therapy team are involved in planning her discharge. She previously lived
alone, with a daughter who helped with the shopping.
What is the most appropriate tool for estimating her ability to return home?
🅐 Barthel index
🅑 Berg balance scale
🅒 Clinical frailty score
🅓 National Institutes of Health Stroke Scale (NIHSS)
🅔 Program of Research to Integrate the Services for the Maintenance of Autonomy 7 (PRISMA-7)

Option1

[423]
A 34-year-old woman has presented to the ambulatory general medical clinic with pain in her leftl eg. She was diagnosed
with lumbar radiculopathy affecting the left L5 lumbar nerve root following a disc prolapse. She has no other medical
problems and does not take any other medications.
She was started on amitriptyline 10 weeks previously by the GP and following a period of up-titration has now been taking
75mg OD for the last 3 weeks.
She reports that the pain is not under control.
What is the next most appropriate management strategy to manage her pain?
🅐 Stop amitriptyline and start duloxetine
🅑 Continue amitriptyline at the current dose and review in 2 weeks time
🅒 Continue amitriptyline at the current dose and add in duloxetine
🅓 Increase amitriptyline in gradual increments over the course of 6 weeks
🅔 Refer to the pain clinic to consider increasing the dose of amitriptyline

Option1

[424]
A 31-year-old woman presents with a 4 month history of headache. She has brought a headache diary which demonstrates
that her symptoms are present on around 20-25 days of each month. The headache is typically unilateral and she is currently
taking paracetamol 1g qds and ibuprofen 400 mg tds everyday to try and relieve her symptoms. A diagnosis of medication
overuse headache is suspected.
What is the most appropriate management?
🅐 Add metoclopramide + start propranolol
🅑 Gradually withdraw analgesics + start propranolol
🅒 Abruptly stop analgesics
🅓 Gradually withdraw analgesics
🅔 Continue analgesics + start propranolol

Option3
[425]
A 38-year-old man with a background of Von Hippel-Lindau syndrome has presented with sudden painless visual loss in his
right eye.
Fundoscopy reveals a right-sided vitreous haemorrhage.
Which underlying pathology is likely to have precipitated this event?
🅐 Central retinal vein occlusion
🅑 Retinoblastoma
🅒 Ocular glioma
🅓 Hypertension
🅔 Retinal haemangioma

Option5

[426]
Which one of the following side-effects is least associated with the use of levodopa?
🅐 Psychosis
🅑 On-off effect
🅒 Postural hypotension
🅓 Cardiac arrhythmias
🅔 Galactorrhea

Option5

[427]
A 45-year-old female with a history of epilepsy is reviewed in the neurology clinic.
Which one of the following features is most likely to be attributable to sodium valproate therapy?
🅐 Clubbing
🅑 Weight loss
🅒 Hirsutism
🅓 Renal impairment
🅔 Tremor

Option5

[428]
A 54-year-old woman presents with sudden onset of memory loss. Her husband states that for a period of 8 hours she was
disoriented in time and place, but could remember who she was and was able to recognise family members. She has no
recollection of the events. There is no history of any traumatic events. The mental state examination is otherwise normal.
What is the most likely diagnosis?
🅐 Bipolar affective disorder
🅑 Dissociation
🅒 Factitious amnesia
🅓 Fugue state
🅔 Transient global amnesia

Option5
[429]
A 47-year-old with polycythaemia is admitted to the stroke unit with right leg weakness and difficulty speaking. His speech is
halting and labored, although the words he is saying are making sense and he is not repeating himself. An MRI brain
confirms a partial anterior circulation stroke affecting the middle cerebral artery (MCA) territory.
Given his symptoms, where is the most likely location of the infarct?
🅐 Right superior temporal gyrus
🅑 Left arcuate fasciculus
🅒 Left inferior temporal gyrus
🅓 Right superior frontal gyrus
🅔 Left inferior frontal gyrus

Option5

[430]
A 64-year-old man presents with a eight-month history of generalised weakness. On examination he has fasciculation and
weakness in both arms with absent reflexes. Examination of the lower limbs reveal increased tone and exaggerated reflexes.
Sensation was normal and there were no cerebellar signs.
What is the most likely diagnosis?
🅐 Progressive muscular atrophy
🅑 Amyotrophic lateral sclerosis
🅒 Vitamin B12 deficiency
🅓 Syringomyelia
🅔 Multiple sclerosis

Option2

[431]
A 55-year-old man is referred to the neurology clinic due to a resting tremor and an abnormal gait characterised by short,
shuffling steps.
Which one of the following features would point towards a diagnosis of Parkinson's disease rather than parkinsonism of
another cause?
🅐 Asymmetrical tremor
🅑 Bradykinesia
🅒 Impairment of vertical gaze
🅓 Confusion
🅔 Poor response to levodopa therapy

Option1

[432]
A 72-year-old man presents to the Emergency Department. Whilst walking back from a friends house he slipped on some
ice and fell backwards, landing on his right arm and banging his head on the kerb in the process. His past medical history
includes atrial fibrillation for which he takes bisoprolol and warfarin. A routine INR taken four days ago was 2.2. There are no
signs of any external injury to his right arm or scalp.
What is the most appropriate course of action with relation to his head injury?
🅐 Arrange a CT scan to be performed within 8 hours
🅑 Discharge with standard head injury advice
🅒 Admit for 24 hours of observation
🅓 Admit for 8 hours of observation
🅔 Discharge with standard head injury advice + advise he stops warfarin for 5 days

Option1
[433]
A 23-year-old man presents with a 4-day history of an itchy and sore right ear. He has recently returned from holiday in
Spain. On examination the right ear canal is inflamed but no debris is seen. The tympanic membrane is clearly visible and is
unremarkable.
What is the most appropriate management?
🅐 Topical corticosteroid + aminoglycoside
🅑 Topical corticosteroid
🅒 Refer to ENT
🅓 Topical corticosteroid + clotrimazole
🅔 Oral flucloxacillin

Option1

[434]
You review a 65-year-old man who is currently taking antipsychotic medication. His carers have noticed that his movements
have been very slow over the past few weeks.
Which one of the following would suggest a diagnosis of Parkinson's disease rather than drug-induced parkinsonism?
🅐 Rigidity
🅑 Masked face
🅒 Bilateral sympotoms
🅓 Flexed posture
🅔 Restlessness of arms and legs

Option1

[435]
A 28-year-old lady presents with a two-day history of headaches, nausea and vomiting, and blurred vision.
She is long-sighted and has worn glasses since childhood. She has a background of severe migraines and was recently
started on a prophylactic medication by her GP.
On examination, both eyes are red with mid-dilated unreactive pupils bilaterally.
Which medication is the most likely cause of this presentation?
🅐 Aspirin
🅑 Propranolol
🅒 Sodium valproate
🅓 Sumatriptan
🅔 Topiramate

Option5

[436]
Which one of the following is least characteristic of Wernicke's encephalopathy?
🅐 Ataxia
🅑 Confusion
🅒 Ophthalmoplegia
🅓 Confabulation
🅔 Nystagmus

Option4
[437]
A 33-year-old woman presents to her GP with headaches.
She initially noted unilateral, throbbing headaches that occurred every few weeks and would be relieved by lying in a
darkened room.
The patient began to use regular paracetamol, ibuprofen and codeine for these headaches, taking them daily.
She now describes a constant headache that she feels all across her head and is not relieved by lying in a darkened room.
Her neurological examination is unremarkable.
What is the most appropriate management?
🅐 Stop all analgesia immediately
🅑 Taper off current analgesia whilst introducing propranolol
🅒 Taper off current analgesia whilst introducing topiramate
🅓 Wean down dose of codeine and stop paracetamol/ibuprofen immediately
🅔 Wean down dose of paracetamol/ibuprofen and stop codeine immediately

Option4

[438]
You are working on the neurology ward. A 32-year-old woman with two young children has just been diagnosed with a
chronic neurological disease after presenting with acute double vision. Two years ago she had an episode of acute vertigo
which she put down to labyrinthitis. She has had an MRI scan of her brain and spine. She wants to know what sort of disease
course she should expect.
Which of the following patterns is most likely for the first several years of the disease?
🅐 Symptoms affecting peripheral nervous system first, and central nervous system later
🅑 Occasional flares of new symptoms in different areas of the body with variable recovery in between
🅒 A slow and gradual accumulation of symptoms over the next 20 years
🅓 Symptoms gradually ascending up her body starting with foot drop
🅔 Rapid accumulation of new symptoms initially but slowing after around 10 years

Option2

[439]
Which of the following features is least likely to be found in a patient with tuberous sclerosis?
🅐 Shagreen patches
🅑 Café-au-lait spots
🅒 Retinal hamartomas
🅓 Axillary freckling
🅔 Renal angiomyolipomata

Option4

[440]
A 55-year-old man presents with word-finding difficulties which came on suddenly one hour ago. He is not able to give a
meaningful response to any of your questions. He is unable to name simple objects and the output of spontaneous speech
is markedly diminished. You ask him to touch his left ear lobe with his right index finger which he manages without issue.
What area of the brain is affected?
🅐 Inferior gyrus
🅑 Lateral geniculate nucleus
🅒 Medial geniculate nucleus
🅓 Striate cortex
🅔 Superior temporal gyrus

Option1
[441]
A 59-year-old man is diagnosed with Parkinson's disease after being referred with a tremor and bradykinesia. His symptoms
are now affecting his ability to work as an accountant and are having a general impact on the quality of his life.
What treatment is he most likely to be offered initially?
🅐 Levodopa
🅑 MAO-B inhibitor
🅒 COMT inhibitor
🅓 Dopamine antagonist
🅔 Dopamine agonist

Option1

[442]
A 65-year-old man presents to the hospital with a sudden acute change in his vision he noticed whilst watching the TV. The
patient is known to have hypertension and atrial fibrillation however reports poor adherence to his regular medication.
On eye examination, there are no clear sclera changes. Visual field examination reveals a homonymous quadrantanopia with
a loss of the right inferior aspect of the vision. All eye movements are normal, pupils are equal and reactive to light and
fundoscopy appears normal.
Where is this patient's lesion most likely located?
🅐 Left superior optic radiations in the parietal lobe
🅑 Left superior optic radiations in the temporal lobe
🅒 Right inferior optic radiations in the parietal lobe
🅓 Right inferior optic radiations in the temporal lobe
🅔 Right superior optic radiations in the parietal lobe

Option1

[443]
A 29-year-old man with myotonic dystrophy has an electrocardiogram.
Which one of the following findings is most likely to be present?
🅐 Wide QRS complex
🅑 Atrial fibrillation
🅒 Voltage criteria for left ventricular hypertrophy
🅓 Right axis deviation
🅔 Prolonged PR interval

Option5

[444]
A 34-year-old man who is known to suffer from focal impaired awareness seizures is reviewed in the neurology clinic. He has
not been able to tolerate either carbamazepine or sodium valproate.
What is the most appropriate next line drug?
🅐 Phenytoin
🅑 Lamotrigine
🅒 Ethosuximide
🅓 Topiramate
🅔 Clonazepam

Option2
[445]
A 52-year-old man is prescribed apomorphine. What type of receptors does apomorphine act on?
🅐 Opioid receptors
🅑 GABA receptors
🅒 Cholinergic receptors
🅓 Dopamine receptors
🅔 Muscurinic receptors

Option4

[446]
Roger is a 32-year-old man presenting with diplopia. He is known to have type 1 diabetes and multiple sclerosis. He had
been experiencing double vision over the last 3 days. This predominantly happens when he looks to the right.
He denies any associated double vision on vertical vision. He has not noticed any difficulty in moving his eyelids. He has not
noticed any increased sensitivity to light and his eye is not red.
On examination, both eyelids display normal strength. With the left eye closed, the right eye displays a full range of
movement. With the right eye closed, the left eye fails to adduct when looking towards the right. Nystagmus on the right eye
is noted when the patient is asked to look to the right with both eyes. On convergence, both eyes can adduct towards the
midline. The pupillary exam is normal with both pupils reacting appropriately to light.
What is the underlying pathology that has caused his diplopia?
🅐 Extrinsic compression of the left oculomotor nerve
🅑 Intrinsic vascular damage to the left oculomotor nerve
🅒 Lesion of the left abducens nerve
🅓 Lesion on the left paramedian area of pons and medulla
🅔 Lesion on the left paramedian area of the midbrain and pons

Option5

[447]
An 80-year-old man presents with sudden-onset difficulty walking and a droopy eyelid. He noticed slurred speech a few
weeks ago, but as this resolved after 10 minutes he did not seek medical attention.
On general examination, he has an ataxic gait and loss of sensation in his right arm and leg. On examination of his cranial
nerves, he has nystagmus to the left, ptosis, meiosis and loss of sensation on the left-hand side of the face.
Where is the likely arterial lesion?
🅐 Anterior inferior cerebellar
🅑 Pontine
🅒 Posterior cerebral
🅓 Posterior inferior cerebellar
🅔 Superior cerebellar

Option4
[448]
An obese 24-year-old female presents with headaches and blurred vision. Examination reveals bilateral blurring of the optic
discs but is otherwise unremarkable with no other neurological signs. Blood pressure is 130/74 and she is apyrexial.
What is the most likely underlying diagnosis?
🅐 Multiple sclerosis
🅑 Meningococcal meningitis
🅒 Brain abscess
🅓 Normal pressure hydrocephalus
🅔 Idiopathic intracranial hypertension

Option5

[449]
A 56-year-old man presented to the emergency department with a sudden-onset occipital headache and slurred speech. He
described a constant throbbing pressure like headache and associated visual disturbances. His past medical history consists
of type-2 diabetes mellitus and hypertension.
On examination, he had an irregularly irregular pulse. Assessment of his cranial nerves demonstrated left homonymous
hemianopia. Power was grossly intact across the limbs and his gait was normal.
Which of the following initial investigations would be most appropriate?
🅐 Carotid doppler
🅑 Echocardiogram
🅒 T2-weighted MRI head scan
🅓 Non-contrast CT head scan
🅔 24-hour holter monitoring

Option4

[450]
A 26-year-old man is reviewed in the Neurology clinic. Two weeks previously, he had presented to the Emergency
Department after experiencing a witnessed tonic-clonic seizure, which resolved spontaneously after two minutes. He has no
past medical history, and denies ever having experienced a seizure before.
Neither the CT scan of his brain performed during his Emergency Department visit, nor a subsequent MRI scan of the brain
showed any abnormalities, and a recent electroencephalogram (EEG) did not show any conclusive epileptiform activity.
Which of the following represents the correct advice he should be given concerning driving restrictions?
🅐 The patient is able to drive immediately
🅑 The patient is unable to drive for a minimum of 6 months, irrespective of whether anti-epileptic medication is initiated or
not
🅒 The patient is unable to drive for a minimum of 12 months, irrespective of whether anti-epileptic medication is initiated or
not
🅓 It is at the discretion of the Neurology consultant whether the patient is able to resume driving or not at this stage
🅔 The patient will be unable to drive until they have proved compliance with anti-epilepticmedication

Option2
[451]
A 64-year-old female with a history of rheumatoid arthritis presents with increased difficulty in walking. On examination there
is weakness of ankle dorsiflexion and of the extensor hallucis longus associated with loss of sensation on the lateral aspect of
the lower leg.
What is the most likely diagnosis?
🅐 Tibial nerve palsy
🅑 Obturator nerve palsy
🅒 Common peroneal nerve palsy
🅓 Lateral cutaneous nerve palsy
🅔 Pudendal nerve palsy

Option3

[452]
A 36-year-old woman presents with a progressively worse tingling sensation in the legs and poor balance. She has no
significant past medical history.
On examination, proprioception and vibration sense are both heavily reduced in the lower limbs. Distal paraesthesia is
evident. Knee reflexes are brisk.
A blood film is taken which demonstrates macrocytic anaemia and hypersegmented neutrophils.
Given the likely diagnosis, which parts of the spinal cord are affected?
🅐 Anterior inferior cerebellar and lateral corticospinal tract
🅑 Dorsal column and lateral corticospinal tract
🅒 Spinocerebellar tract and dorsal column
🅓 Spinothalamic tract and dorsal column
🅔 Spinothalamic tract and lateral corticospinal tract

Option2

[453]
A 21-year-old man presents with progressive hearing loss as well as an on-off ringing affecting both ears. The patient is
otherwise well with no other medical conditions. He does however reports that both his father and grandparents had similar
issues at a young age and eventually required surgical intervention.
External and otoscope examinations are normal however Rinne and Weber's tests reveal a bilateralconductive hearing loss.
What is the most likely diagnosis?
🅐 Alport syndrome
🅑 Cerumen
🅒 Cholesteatoma
🅓 Microtia
🅔 Otosclerosis

Option5
[454]
An 84-year-old female presents to the emergency department with impairment of her speech. She has a background
medical history of type two diabetes mellitus, hypertension, hypercholesterolemia, and a 50-pack year smoking history.
On examination, her speech is fluent. She has no difficulty naming examples within a category, she can name 26 types of
animals in 1 minute. However, she is unable to repeat both simple phrases, such as 'he did it' and 'this is it' and complex
phrases, such as, 'baby hippopotamus' and 'British constitution'. She is able to follow 3 stage commands. She has normal
tone, power, coordination, and reflexes throughout.
Neurological imaging reveals an acute small localised infarct.
Where is this infarct most likely to be found?
🅐 Angular gyrus
🅑 Arcuate fasciculus
🅒 Inferior frontal gyrus
🅓 Transverse temporal gyri (Heschl's gyri)
🅔 Broadmann area 22 in the superior temporal gyrus

Option2

[455]
A 9-year-old boy is referred to a pediatric neurologist for a headache that has been persistent for the past two months. The
mother had initially thought that the boy was having a headache because of school stress but recently the boy has been
increasingly involved in accidents while riding his bicycle. The boy says that he could not see his other friends when they
rode their bicycle next to him. The boy was born via a Caesarean section and had a normal development so far and is doing
well at school. After a thorough examination, the doctor finds that the boy also has a visual defect characterized by an
inability to perceive the two temporal visual fields.
If this boy undergoes surgery for his condition, which part of this child's hypothalamus would be affectedcausing weight gain
after surgery?
🅐 Supraoptic nucleus of the hypothalamus
🅑 Posterior hypothalamus
🅒 Anterior hypothalamus
🅓 Ventromedial area of the hypothalamus
🅔 Paraventricular nucleus of the hypothalamus

Option4

[456]
Which one of the following is least likely to cause a raised protein level in the cerebrospinal fluid?
🅐 Tuberculous meningitis
🅑 Guillain-Barre syndrome
🅒 Fungal meningitis
🅓 Froin's syndrome
🅔 Systemic lupus erythematosus

Option5
[457]
A 55-year-old man is referred to neurology outpatients with reports of a three-month history of weakness in his upper and
lower limbs. He has no past medical history. He is right-handed.
On examination, there were fasciculations and muscle atrophy in the right upper limb. Power was 4/5 in the right upper limb
and left lower limb. Hyperreflexia of the right biceps reflex was noted along with hyporeflexia of the bilateral knee reflexes.
The plantar reflexes were upgoing. The sensation was normal throughout.
Given the likely diagnosis, which of the following treatments will confer a prognostic benefit?
🅐 Intravenous immunoglobulin
🅑 Prednisolone
🅒 Pyridostigmine
🅓 Riluzole
🅔 Surgery

Option4

[458]
A 25-year-old female with a history of bilateral vitreous haemorrhage is referred due to progressive ataxia.
What is the likely diagnosis?
🅐 Neurofibromatosis type 1
🅑 Neurofibromatosis type 2
🅒 Tuberous sclerosis
🅓 Von Hippel-Lindau disease
🅔 Sarcoidosis

Option4

[459]
A 47-year-old female is reviewed in the neurology clinic. She was diagnosed with epilepsy whilst a teenager and her seizures
are well controlled. She is however concerned about increasing numbness of her fingers and soles of her feet.
Which one of the following medications is mostlikely to be responsible?
🅐 Phenytoin
🅑 Lamotrigine
🅒 Sodium valproate
🅓 Ethosuximide
🅔 Levetiracetam

Option1

[460]
An 80-year-old retired female doctor presents to her general practitioner with ongoing sharp stabbing and burning pain on
the left side of her face. Six months ago she was diagnosed with Ramsay Hunt syndrome (herpes zoster oticus), whereby
after treatment with acyclovir, within 72 hours of the shingles lesions being seen via otoscopy, this woman has no residual
facial paralysis or hearing loss.
However, despite 6 months of amitriptyline hydrochloride being titrated up from 10 mg ON to the maximum dose of 75 mg
ON, she is still suffering from moderate post-herpetic neuralgia. The pain is localised to the left side of her face, neck and in
her left ear. She has no other medical history and takes no other medications.
What is the next step to alleviate her neuropathic pain?
🅐 Add on duloxetine
🅑 Add on gabapentin
🅒 Add on pregabalin
🅓 Add on pregabalin and duloxetine
🅔 Stop amitriptyline and commence pregabalin

Option5
[461]
A 35-year-old male presents to the emergency department with a headache, fever and right-hand weakness. There was no
photophobia or neck stiffness. He was currently being treated for a sinus infection by his GP with oral amoxicillin. He had no
significant past medical history and was on no regular medications.
Observations were heart rate 84/min, blood pressure 136/72 mmHg, respiratory rate 18/min, temperature 38.4ºC, oxygen
saturation of 97% on air. He was mildly tender over the right forehead. Neurological examination revealed power was 2/5 in
his right hand.
A CT brain was organized, which demonstrated an intracerebral lesion with an outer hypodense and inner hyperdense rim.
Given the likely diagnosis, what is the most appropriate initial medication treatment for thispatient?
🅐 Chemotherapy
🅑 IV co-amoxiclav
🅒 IV flucloxacillin
🅓 Pyrimethamine + sulfadiazine
🅔 IV 3rd generation cephalosporin + metronidazole

Option5

[462]
A 44-year-old woman presents with a three month history of worsening involuntary movements of the head. These are worse
when she is stressed and improved by alcohol. They are not present when she is sleep. There are no other neurological
symptoms of note and neurological examination is unremarkable other than spotaneous movements of the head which are
worse when she looks to either side. Her father had a similar complaint but never sought medical attention.
What is the most likely diagnosis?
🅐 Parkinson's diseaes
🅑 Cerebellar tremor
🅒 Huntington's disease
🅓 Multiple sclerosis
🅔 Essential tremor

Option5

[463]
A 60-year-old gentleman with a background of lumbar spondylosis and chronic back pain presents with gradually worsening
bilateral upper limb paraesthesias and leg stiffness.
Which one of the investigations below is diagnostic for his likely condition?
🅐 Nerve conduction studies and EMG
🅑 MRI cervical spine
🅒 MRI lumbar spine
🅓 CT C-spine
🅔 AP and lateral C-spine radiographs

Option2
[464]
A 28-year-old woman attends the emergency department with a tonic-clonic seizure. She has a history of epilepsy. She has
switched from levetiracetam to carbamazepine approximately 4 weeks ago. Her epilepsy has been well controlled during
this period. However, over the last 24 hours, she has had three tonic-clonic seizures witnessed by her husband.
Blood tests are as follows:
• Na+: 140 mmol/L (135 - 145)
• K+: 4.2 mmol/L (3.5 - 5.0)
• Urea: 4.2 mmol/L (2.0 - 7.0)
• Creatinine: 61 μmol/L (55 - 120)
• CRP: 2 mg/L (< 5)
A CT brain is performed:
• CT brain: Unremarkable
What mechanism explains why her seizures have returned?
🅐 Auto-induction
🅑 Auto-inhibition
🅒 Michaelis-Menten kinetics
🅓 Tachyphylaxis
🅔 Zero-order kinetics

Option1

[465]
A 34-year-old man with a history of migraine finds that paracetamol taken at the recommend dose often fails to relieve his
acute attacks. He drinks 12 units of alcohol per week and smokes 15 cigarettes per day.
What factor is likely to contribute to this problem?
🅐 Bacterial overgrowth
🅑 Delayed gastric emptying
🅒 P450 enzyme induction
🅓 First pass metabolism
🅔 P450 enzyme inhibition

Option2

[466]
A 62-year-old man presents with left-sided eye pain and diplopia for the past 2 days. Examination of his eyes shows his
pupils equal and reactive to light with no proptosis. There is however an apparent palsy of the 6th cranial nerve associated
with a partial 3rd nerve palsy on the left side. Examining the remaining cranial demonstrates hyperaesthesia of the upper
face on the left side.
Where is the likely lesion?
🅐 Cavernous sinus
🅑 Orbital apex
🅒 Pons
🅓 Cerebropontine angle
🅔 Medulla

Option1
[467]
You are examining a patient who complains of double vision. On looking forward, the patient's right eye turn is deviated
upwards and rotated outwards. On attempting to look to the patient's left the double vision worsens.
What is the most likely underlying problem?
🅐 Right 6th nerve palsy
🅑 Right 4th nerve palsy
🅒 Right 3rd nerve palsy
🅓 Left 6th nerve palsy
🅔 Left 3rd nerve palsy

Option2

[468]
An 86-year-old female presents to the emergency department with impairment of her speech. She has a background
medical history of type two diabetes mellitus, hypertension, hypercholesterolemia, and a 50-pack year smoking history.
On examination, her speech is fluent but incomprehensible. Her sentences are composed of a random assortment of words,
that even she doesn't appear to understand. She is unable to repeat both simple phrases, such as 'he did it' and 'this is it'
and complex phrases, such as 'baby hippopotamus' and 'British constitution'. She is unable to follow 1 stage commands.
She has normal tone, power, coordination, and reflexes throughout.
Neurological imaging reveals an acute small, localised infarct.
Where is this infarct most likely to be found?
🅐 Angular gyrus
🅑 Arcuate fasciculus
🅒 Inferior frontal gyrus
🅓 Transverse temporal gyri (Heschl's gyri)
🅔 Broadmann area 22 in the superior temporal gyrus

Option5

[469]
A 5-year-old boy is referred by his GP to the neurology clinic with abnormal movements. His mother noticed that for the last
year, the boy is starting to fall over more and more frequently. He has also been having increasingly slurred speech. These
have been getting progressively worse. He has had recurrent chest infections in his childhood.
What is the most likely diagnosis?
🅐 Friedreich's ataxia
🅑 Infantile-onset spinocerebellar ataxia
🅒 Di-George syndrome
🅓 Cerebral palsy
🅔 Ataxic telangiectaisa

Option5
[470]
A 54-year-old woman is admitted to the hospital after an acute confusion episode following taking part in a half marathon in
August. She repeatedly asks the same questions and seemsunable to retain any new information.
She has a history of ischaemic heart disease and hyperlipidemia. She has not had any head injuries, and physical
examination, including a full neurological exam, is normal. A toxicology screen, alcohol level, and basic labs including
glucose and electrolytes are normal. CT head is awaited, and it is 13 hours since the onset of symptoms.
What is the most likely diagnosis?
🅐 Basilar artery thrombosis
🅑 Heat stroke
🅒 Lacunar stroke
🅓 Rhabdomyolysis
🅔 Transient global amnesia

Option5

[471]
A 65-year-old man attends the emergency department with increasing weakness in both his legs. He states his muscles are
increasingly tense and his legs feel spastic. He has also noticed are current burning sensation in both his legs.
On examination, there are absent ankle jerks and extensor planters. There is also impaired proprioception in both legs.
When standing up with his eyes closed he loses balance.
What is the likely diagnosis?
🅐 Brown-Sequard syndrome
🅑 Friedreich's ataxia
🅒 Multiple sclerosis
🅓 Subacute combined degeneration of the spinal cord
🅔 Syringomyelia

Option4

[472]
During a routine cranial nerve examination the following findings are observed:
• Rinne's test: Air conduction > bone conduction in both ears
• Weber's test: Localises to the right side
What do these tests imply?
🅐 Left conductive deafness
🅑 Normal hearing
🅒 Right conductive deafness
🅓 Right sensorineural deafness
🅔 Left sensorineural deafness

Option5
[473]
A 24-year-old man presents to his GP with weakness of the left hand.
He recalls a few weeks ago, he tried to climb a tree when he was drunk but fell, hitting a few branches on the way down. He
describes difficulty moving his left hand since then.
On examination, all the fingers of the left hand are flexed into a claw position. There is a loss of sensation along the ulnar
aspect of the whole hand and arm.
What is the most likely cause of his symptoms?
🅐 Carpal tunnel syndrome
🅑 Erb's palsy
🅒 Klumpke's palsy
🅓 Saturday night palsy
🅔 Ulnar nerve compression

Option3

[474]
A 66-year-old man is reviewed in the Neurology clinic after reporting a one-month history of progressive weakness, primarily
affecting his upper limbs. He also reports that his limbs feel very stiff in the mornings.
On examination, his movements are 4+/5 bilaterally in his lower limbs, and 3/5 bilaterally in his upper limbs on the Medical
Research Council (MRC) power scale. He is also noticed to have a nasal quality to his speech, and occasional calf
fasciculations. Peripheral sensation is intact.
Given the likely diagnosis, which of the following treatment options is most likely to extend this patient's survival?
🅐 Interferon alpha
🅑 Interferon beta
🅒 Riluzole
🅓 Non-invasive ventilation
🅔 Chest wall physiotherapy

Option4

[475]
A 74-year-old man presents with sudden onset left arm weakness, dysphasia and a left-sided facial droop which started one
hour ago.
He has a background of hypertension, type 2 diabetes mellitus and hypercholesterolaemia. He lives with his wife and is
usually independent of all activities of daily living, regularly playing golfwith friends.
On examination, he has increased tone and 2/5 power throughout his left arm. His National Institutes of Health Stroke Scale
(NIHSS) score is 9.
CT head shows an ischaemic lesion in the right hemisphere without evidence of haemorrhagic transformation. Magnetic
resonance angiography confirms occlusion of the proximal anterior circulation.
What is the best next step in management?
🅐 Clopidogrel
🅑 Intravenous heparin
🅒 Modified-release dipyridamole
🅓 Thrombolysis
🅔 Thrombolysis and thrombectomy

Option5
[476]
A 67-year-old woman comes for review with her husband. Her husband complains that she is constantly getting up from bed
at night and pacing around the bedroom. She complains of 'antsy' legs and a 'horrible, creeping sensation'. Her symptoms
generally come on in the evening and are only relieved by moving round.
Given the likely diagnosis, what is the most appropriate treatment?
🅐 Ropinirole
🅑 Carbamazepine
🅒 Amitriptyline
🅓 Citalopram
🅔 Quinine

Option1

[477]
A 41-year-old man presented to the emergency department 24 hours ago with a thunderclap headache. A computed
tomography (CT) scan confirmed a spontaneous subarachnoid haemorrhage.
He was admitted to the ward while awaiting neurosurgical intervention and has been commenced on nimodipine.
Despite feeling relatively settled overnight, he reports that he is significantly more nauseous this morning, and has vomited
several times in the last 2 hours despite intravenous anti-emetics.
He is apyrexial and his Glasgow Coma Scale of 14 (E3, V5, M6). His pulse is 84/min and his blood pressure is 148/90 mmHg.
What is the next most appropriate intervention?
🅐 Intravenous labetalol
🅑 Hypertonic saline
🅒 Rapid sequence induction and intubation
🅓 Urgent CT head
🅔 Urgent MRI head

Option4

[478]
A 29-year-old woman presents to her GP with fatigue. She says that for the last few months she has felt weak, particularly
towards the end of the day. She does not report any change in sleeping habit, mood, diet or weight. She also says that she
sometimes gets double vision.
On examination she has a partial ptosis bilaterally but more noticeable on the left, and the rest of her cranial nerves are
normal. Her power is 4/5 all limbs, and her sensation and reflexes are intact.
What is the pathophysiology underlying the likely diagnosis?
🅐 Acetylcholine receptor antibodies
🅑 Cerebral infarction
🅒 Demyelination of the central nervous system
🅓 Demyelination of the peripheral nervous system
🅔 Inclusion bodies in motor neurones

Option1
[479]
A 25 year-old man presents with worsening bilateral sensorineural deafness. A trial of steroid therapy by his GP had been
unsuccessful. He has an MRI scan which demonstrates bilateral lesions at the cerebellopontine angles. He recalls that his
father had some form of brain tumours, but can't remember what. His father also has hearing impairment.
Which of the following chromosomes is likely to have a defect on genetic analysis?
🅐3
🅑 16
🅒 17
🅓X
🅔 22

Option5

[480]
A 48-year-old man presents to his GP with a number of symptoms that have come on over the past few days. His vision is
blurred and his right eye is painful, as well as having a generalised headache.
On examination, the right eye has deviated inferiorly and laterally. There is visible ptosis of the upper right lid, and the pupil
is dilated. It does not respond to light. This eye does not follow movements well, but the left eye appears unaffected and is
normal upon testing.
Where is the most likely location of the lesion?
🅐 Anterior cerebral artery
🅑 Internal carotid artery
🅒 Medulla
🅓 Oculomotor nucleus
🅔 Posterior communicating artery

Option5

[481]
A 16-year-old boy presents having fallen from a tree. He fell from a height of 2 metres before catching himself with his right
arm on a branch just before the ground. He reports his arm was pulled forcibly upwards, causing instant pain both at the
hand and the lower neck. He was able to lower himself to the ground without further injury and get to the hospital.
On examination of the upper limb, there are no open wounds or evidence of fractures. There is an isolated reduction of
movement and power of the intrinsic hand muscles. Examination of all other joints of the upper limb is normal.
Which nerve root injury pattern has the boy sustained?
🅐 C5
🅑 C6
🅒 C7
🅓 L4-5 and S1-S2
🅔 T1

Option5
[482]
A 67-year-old man presents to his GP with numbness in his feet and recurrent falls over the past month. He describes
difficulty walking. His GP recently started him on folate for a macrocytic anaemia. He takes no other regular medications. He
does not drink alcohol. On examination, he has an ataxic gait and Romberg's test is positive. Examination of his lower limbs
demonstrates an increase in tone and bilateral weakness. He has absent ankle and knee jerks with upgoing plantars. Light
touch and vibration sense is reduced bilaterally.
What is the most likely cause of his symptoms?
🅐 Amyotrophic lateral sclerosis
🅑 Guallain-Barre syndrome
🅒 Subacute combined degeneration of the cord
🅓 Myasthenia gravis
🅔 Charcot-Marie-Tooth disease

Option3

[484]
A 33-year-old male is admitted with a subacute headache, confusion, and nausea. On closer questioning, you note that this
gentleman appears irritable, confused, and is hearing voices. Prior to this, his wife reports he has had a recent viral upper
respiratory tract infection. He is otherwise fit and well and takes no regular medications.
On examination, you note that he is neglecting to use his right arm. He is otherwise, neurologically intact. A subsequent CT
head reveals no space occupying lesion, infarct or bleed. Later, an MRI is performed which reveals bilateral asymmetric and
poorly marginated hyperintense T2-weighted and FLAIR lesions within the subcortical white matter. A lumbar puncture
reveals no oligoclonal bands in the CSF.
What is the likely underlying diagnosis?
🅐 Alzheimer's disease
🅑 Multiple sclerosis
🅒 Meningitis
🅓 Herpes simplex encephalitis
🅔 Acute disseminated encephalomyelitis

Option5
[485]
A 26-year-old woman is recovering on the ward following a subarachnoid haemorrhage 6 days ago. She has been managing
to maintain her oral fluid intake above 3 litres per day and her heart rate is 72 bpm at rest and her blood pressure is 146/88
mmHg at rest. Her fluid balance shows that she is net positive 650 ml over the last 6 days. Her daily bloods show the
following:
• Hb: 134 g/l
• Platelets: 253 x 109/l
• WBC: 5.1 x 109/l
• Neuts: 3.9 x 109/l
• Lymphs: 1.2 x 109/l
• Na+: 129 mmol/l
• K+: 4.1 mmol/l
• Urea: 2.3 mmol/l
• Creatinine: 49 μmol/l
• CRP: 12.3 mg/l
Paired serum and urine samples shows the following:
• Serum Osmolality: 263 mosm/l
• Urine Osmolality: 599 mosm/l
• Serum Na+: 129 mmol/l
• Urine Na+: 63 mmol/l
What is the most likely cause for the patient's hyponatraemia?
🅐 Cerebral salt-wasting syndrome
🅑 Iatrogenic fluid overload
🅒 Cranial diabetes insipidus
🅓 Syndrome of inappropriate anti-diuretic hormone secretion (SIADH)
🅔 Adrenal insufficiency

Option4

[486]
A 75-year-old man is seen in the neurology clinic following discharge from hospital with an ischaemic stroke 6 months ago.
He has some persistent altered neurological features associated with the location of his stroke. During one test, he can
correctly identify objects placed into either of his hands with his eyes open but is unable to do so with his eyes closed. In the
latter scenario, he can describe the shapes of objects but is unable to name them or describe their function.
Based on this information, what was the most likely location of the patient's stroke?
🅐 Dominant parietal lobe
🅑 Dominant temporal lobe
🅒 Frontal lobe
🅓 Non-dominant parietal lobe
🅔 Non-dominant temporal lobe

Option1
[487]
A 27-year-old woman with epilepsy manifesting as tonic-clonic seizures has been relatively well-controlled for the last 14
months on combination therapy of sodium valproate and levetiracetam. Two months after giving birth she has had a week of
seizures every day and has now been started on clobazam, in addition to the anti-seizure medications listed above.
She would like to continue to breastfeed her baby and has read that some medications can harm the baby.
What medication change should occur?
🅐 No medication changes
🅑 Stop clobazam
🅒 Stop clobazam and sodium valproate
🅓 Stop levetiracetam
🅔 Stop sodium valproate

Option1

[488]
A 27-year-old woman is reviewed due to sudden loss of vision in her left eye. She is known to have severe rheumatoid
arthritis and is treated currently with methotrexate, infliximab and prednisolone. She has in the past also used sulfasalazine
and hydroxychloroquine. For the past 6 weeks she has developed troublesome headaches. Examination demonstrates
bilateral papilloedema.
Which one of the following is most likely to be responsible for this presentation?
🅐 Chloroquine retinopathy
🅑 Prednisolone
🅒 Infliximab
🅓 Methotrexate
🅔 Keratoconjunctivitis sicca

Option2

[489]
A 54-year-old man presents concerned about leg weakness. On examination he is noted to have increased tone in both
legs, brisk reflexes and weakness in both lower limbs. Examination of his upper limbs is normal.
Which one of the following is least likely to produce these symptoms?
🅐 HIV
🅑 Amyloidosis
🅒 Hereditary spastic paraplegia
🅓 Multiple sclerosis
🅔 Parasagittal meningioma

Option2

[489]
A 40-year-old man presents with a 4-day history of vertigo. This seems to have followed a viral upper respiratory tract
infection in the past week. He is generally fit and well. His symptoms are associated with some nausea but there is no hearing
loss or tinnitus. On examination fine horizontal nystagmus is noted.
What is the most likely diagnosis?
🅐 Vestibular neuronitis
🅑 Viral labyrinthitis
🅒 Transient ischaemic attack
🅓 Acoustic neuroma
🅔 Meniere's disease

Option1
[490]
A 34-year-old man presents to the emergency department with a witnessed tonic-clonic seizure. His partner reports that he
has had intermittent fevers for three days and has been complaining of a progressive headache for two weeks. He has
recently had a sinus infection but is normally fit and well.
You arrange an urgent CT head that shows a frontal ring enhancing lesion.
What is the most appropriate first-line treatment for this patient?
🅐 Amphotericin B and fluconazole
🅑 Ceftriaxone and aciclovir
🅒 Ceftriaxone and metronidazole
🅓 Dexamethasone
🅔 Sulfadiazine and pyrimethamine

Option3

[491]
A 25-year-old woman is started on carbamazepine by her neurologist for partial seizures. She initially has a good response
but 1-month later, her symptoms return.
What is the process through which this medication has lost its effect?
🅐 Acetylator status
🅑 Autoinduction
🅒 Cytochrome P450 enzyme inhibition
🅓 First-pass metabolism
🅔 Zero-order kinetics

Option2

[492]
A 56-year-old woman presents with facial asymmetry. Whilst brushing her teeth this morning she noted that the right hand
corner of her mouth was drooping. She is generally well but noted some pain behind her right ear yesterday and says her
right eye is becoming dry. On examination she has a complete paralysis of the facial nerve on the right side, extending from
the forehead to the mouth. Ear, nose and throat examination is normal. Clinical examination of the peripheral nervous
system is normal.
What is the most likely diagnosis?
🅐 Ramsey-Hunt syndrome
🅑 Bell's palsy
🅒 Stroke
🅓 Multiple sclerosis
🅔 Parotid tumour

Option2

[493]
You are in the joint paediatric and adult neurology clinic. Your next patient is a 10-year-old girl, accompanied by her mother,
who was recently diagnosed with seizures. These were happening around two to four times per day and usually occurred
during episodes of stress. A seizure would last four a few seconds and the mother described her daughter as being vacant
and staring into space.
The mother is concerned about the seizures persisting as she has a brother who also has seizures.
Given the likely diagnosis, what is the probability of the patient becoming seizure-free during adolescence?
🅐 20-25%
🅑 45-50%
🅒 70-75%
🅓 90-95%
🅔1

Option4
[494]
A 55-year-old man presents due to an uncontrollable urge to move his legs during the night-time. He has also experience
the sensation of spiders crawling over his legs. Simple measures such as walking and massaging the affected limb have not
alleviated the problem.
What is the most appropriate medical therapy?
🅐 Selective serotonin reuptake inhibitor
🅑 Low-dose tricyclic antidepressant
🅒 Dopamine agonist
🅓 5-HT3 antagonist
🅔 Dopamine antagonist

Option3

[495]
A 38-year-old woman comes for review. Six months ago she fractured her left wrist whilst skiing. The fracture was treated
using a cast and repeat x-rays showed that the bone had healed well. Unfortunately for the past few weeks she has been
plagued with ongoing 'shooting pains' in her left hand associated with swelling. On examination the left hand is extremely
tender to even light touch. Her left hand is also slightly swollen compared to the right.
What is the most likelydiagnosis?
🅐 Depression
🅑 Conversion disorder
🅒 Complex regional pain syndrome
🅓 Ulnar nerve injury
🅔 Osteomyelitis

Option3

[496]
An 18-year-old girl with a known Chiari 1 malformation presents to her General Practitioner with loss of sensation in both of
her arms and forearms, and on the back of her neck. On testing, she specifically can't feel pain and temperature but can
detect fine touch, proprioception and vibration.
Which of the following abnormalities has this patient most likely have?
🅐 Syringomyelia
🅑 Hydrocephalus
🅒 Brain stem compression
🅓 Cerebellar compression
🅔 Peripheral neuropathy

Option1

[497]
A 65-year-old man with a known movement disorder diagnosed several years ago is seen in a neurology clinic for review
after recently having his medications altered and selegiline added due to limited improvement on first-line therapy.
The patient reports some improvement in his movement symptoms however he has noted several new side effects including
constipation, fatigue and a dry mouth.
What class does this patient's new medication belong to?
🅐 Anticholinergics
🅑 COMT inhibitor
🅒 Dopamine agonist
🅓 MAO-B inhibitor
🅔 Selective serotonin reuptake inhibitor

Option4
[498]
An 84-year-old gentleman is seen in Neurology clinic with an abnormal gait. On questioning, you find that he has
impairment of short-term memory and spatial awareness on the Montreal Cognitive Assessment. He is accompanied by his
wife who reports that he has been having occasional incontinence of urine recently.
On examination, you notice that he has a magnetic gait and a postural tremor.
What is the likely underlying diagnosis?
🅐 Lewy body dementia
🅑 Idiopathic Parkinson's disease
🅒 Multiple system atrophy
🅓 Normal pressure hydrocephalus
🅔 Prion disease

Option4

[499]
A 65-year-old gentleman is referred to neurology outpatients with arm pain, stiffness and imbalance. Following
investigations he is diagnosed with degenerative cervical myelopathy .Unfortunately, he misses his next outpatient clinic due
to admission with acute coronary syndrome. He attends his GP 2 months later and mentions his ongoing neurological
symptoms.
Which of the following is the most important next step in his care?
🅐 Refer to spinal surgery or neurosurgery
🅑 Refer for cervical nerve root injections
🅒 Commence neuropathic analgesia
🅓 Reassure the patient of this diagnosis
🅔 Refer for physiotherapy

Option1

[500]
A 65-year-old man is referred to the neurology outpatient clinic due to a resting tremor of his right hand. A diagnosis of
Parkinson's disease is made. He is otherwise well and is not currently disabled by his symptoms.
What is the most appropriate treatment?
🅐 Amantadine
🅑 No treatment
🅒 New generation dopamine receptor agonist e.g. ropinirole
🅓 Conventional dopamine receptor agonist e.g. bromocriptine
🅔 Antimuscarinics

Option3
[501]
A 19-year-old-man is seen in neurology clinic. He describes two episodes in the last week where he has developed sudden,
jerking motions in both arms. These episodes have both happened shortly after waking, and in one instance, caused him to
drop his breakfast on the floor. On both occasions, he has retained his awareness throughout, and recovered swiftly after the
episode with no post-ictal symptoms.
He is normally fit and well, and does not take any regular medications. He has no known drug allergies.
Neurological examination is unremarkable. He is keen to start on medication, as he wishes to avoid any further disruption to
his day-to-day life.
Which is the most appropriate treatment option for this patient?
🅐 Observe for further seizure activity before initiating treatment
🅑 Sodium valproate
🅒 Carbamazepine
🅓 Lamotrigine
🅔 Levetiracetam

Option2

[502]
A 56-year-old woman comes for review. Around 4 weeks ago she had a blistering rash under her right breast which
extended around to the back. A diagnosis of shingles was made. Unfortunately since that time she has been experiencing
severe 'shooting' pains. The skin is also very tender tot ouch. Neither paracetamol nor ibuprofen have helped her symptoms.
What is the most appropriate next step in management?
🅐 Lidocaine patch
🅑 Tramadol
🅒 Amitriptyline
🅓 Carbamazepine
🅔 Diclofenac

Option3

[503]
A 68-year-old man presents to the emergency department with several hours of loss of balance. He reports that the left side
of his face feels numb and that he is having difficulty swallowing. He has a past medical history of hypertension,
hyperlipidaemia and type 2 diabetes mellitus.
On examination, there is left-sided ptosis and miosis. The left side of the face feels numb when examined, and anhidrosis is
evident, relative to the right side. Some nystagmus is evident and he has an ataxic gait.
What is the most likely vessel affected?
🅐 Anterior inferior cerebellar artery
🅑 Anterior spinal artery
🅒 Basilar artery
🅓 Posterior cerebral artery
🅔 Posterior inferior cerebellar artery

Option5
[504]
A 45-year-old man presents to the Emergency Department with a persistent, generalised headache. He had experienced
several episodes of vomiting over the last two days, and remains nauseated.
His past medical history was notable for rheumatoid arthritis, for which he had recently been started on etanercept.
He had received a course of amoxicillin from his GP two weeks previously for a presumed middle ear infection.
During the consultation, he is irritable and easily distracted. On examination, there is no focal neurology, rash, neck stiffness
or photophobia.
His observations are as follows: temperature 38.8ºC, respiratory rate 21 breaths per minute, heart rate 102 beats per minute,
blood pressure 135/75mmHg, oxygen saturation 97% on air.
A CT head with contrast is performed, which shows a solitary ring-enhancing lesion in the left temporal lobe.
Based on the likely diagnosis, which of the following treatments should be initiated?
🅐 IV piperacillin/tazobactam
🅑 IV cefazolin + metronidazole
🅒 IV aciclovir
🅓 IV ceftriaxone + metronidazole
🅔 IV ceftriaxone

Option4

[505]
In patients with Guillain-Barre syndrome, respiratory function should be monitored with:
🅐 Oxygen saturations
🅑 PEFR
🅒 Flow volume loop
🅓 Arterial blood gases
🅔 Forced vital capacity

Option5

[506]
An 18-year-old man presents for a general health review. He is currently well in himself, although has a complex past medical
history involving recurrent chest infections and prominent difficulties with gait and movement since early childhood.
On examination, gait and limb coordination and movement difficulties are evident. Truncal instability is noted and tendon
reflexes are absent. Distinct blood vessels are visible in both eyes, giving a 'bloodshot' appearance. Similar vessels are also
prominent on the ear lobes.
What is the most likely diagnosis?
🅐 Ataxia telangiectasia (AT)
🅑 Fragile-X tremor-ataxia syndrome (FXTAS)
🅒 Friedrich's ataxia (FA)
🅓 Niemann-Pick disease type C (NP-C)
🅔 Von Hippel-Lindau (VHL) disease

Option1
[507]
A 25-year-old man presents for review. For the past year he has been experiencing headaches. These are now occurring
around 5-6 times per month and typically 'last all day' when they occur. They are not associated with any form of aura. A
typical headache is described as a severe throbbing on both sides of his head associated with nausea and lethargy. When
he gets such a headache he typically goes to bed so he can 'sleep it off' as activity such as walking or climbing the stairs
makes it worse. Before going to bed he typically takes one of his father's diclofenac tablets which seem to help.
Neurological examination is unremarkable.
What is the most likely diagnosis?
🅐 Migraine
🅑 Cluster headache
🅒 Medication-overuse headache
🅓 Tension headache
🅔 Raised intracranial pressure

Option1

[508]
A 10-year-old is referred to neurology due to episodes her GP feels are epileptiform. Her mother reports that she appears to
just 'stop', sometimes even in mid conversation, for several seconds at random times during the day. During these episodes
she can be unresponsive to questioning and has no recollection of them.
Which of these drugs is contraindicated in this condition?
🅐 Ethosuximide
🅑 Sodium valproate
🅒 Lamotrigine
🅓 Carbamazepine
🅔 Clonazepam

Option4

You might also like

pFad - Phonifier reborn

Pfad - The Proxy pFad of © 2024 Garber Painting. All rights reserved.

Note: This service is not intended for secure transactions such as banking, social media, email, or purchasing. Use at your own risk. We assume no liability whatsoever for broken pages.


Alternative Proxies:

Alternative Proxy

pFad Proxy

pFad v3 Proxy

pFad v4 Proxy